SlideShare a Scribd company logo
LAPORAN SIKLUS 1
PENDALAMAN MATERI KOMPETENSI KEAHLIAN
ARITMATIKA DAN BILANGAN
Disusun Oleh:
Arrini Ditta Margarani
NIM. SM-3T. 2016066155
PENDIDIKAN MATEMATIKA
PENDIDIKAN PROFESI GURU SM-3T
UNIVERSITAS PENDIDIKAN INDONESIA
2018
ii
DAFTAR ISI
HALAMAN JUDUL................................................................................................ i
DAFTAR ISI........................................................................................................... ii
BILANGAN BULAT DAN PEMECAHAN MASALAH ..................................... 1
BILANGAN PECAHAN DAN PEMECAHAN MASALAH................................ 8
BARISAN DAN DERET...................................................................................... 14
Barisan dan Deret Aritmatika............................................................................ 15
Barisan dan Deret Geometri.............................................................................. 19
Barisan dan Deret Lain...................................................................................... 22
INDUKSI MATEMATIS ..................................................................................... 27
BILANGAN BERPANGKAT BULAT................................................................ 29
BILANGAN PANGKAT RASIONAL ................................................................ 32
SIFAT-SIFAT KETERBAGIAN.......................................................................... 35
ALGORITMA EUCLID ....................................................................................... 36
FPB dan KPK........................................................................................................ 38
PERSAMAAN DIOPHANTINE LINEAR .......................................................... 39
SIFAT-SIFAT DASAR KONGRUENSI.............................................................. 42
UJI PEMBAGIAN BILANGAN BULAT............................................................ 53
KONGRUENSI LINEAR..................................................................................... 56
LATIHAN SOAL ................................................................................................. 59
DAFTAR PUSTAKA ........................................................................................... 60
1
BILANGAN BULAT DAN PEMECAHAN MASALAH
Definisi:
Bilangan bulat adalah suatu istilah yang hanya ada dalam pikiran, namun akan
memiliki arti jika terdapat banyaknya atau dikaitkan dengan kuantitas. Bilangan
Bulat merupakan gabungan dari himpunan bilangan cacah dan himpunan bilangan
asli.
Fakta Bilangan Bulat
 Himpunan semua bilangan bulat dalam matematika dilambangkan dengan Z
berasal dari Zahlen (bahasa Jerman untuk "bilangan").
Sifat Bilangan Bulat
1. Sifat Tertutup
 Sifat tertutup terhadap penjumlahan ada dengan tunggal yakni untuk setiap
𝑎, 𝑏 𝜖 𝑍 maka (a + b) juga di dalam Z.
 Sifat tertutup terhadap perkalian ada dengan tunggal, yakni untuk setiap
𝑎, 𝑏 𝜖 𝑍 maka a x b juga ada di dalam Z.
2. Sifat Komutatif
 Untuk setiap 𝑎, 𝑏 𝜖 𝑍 berlaku a + b = b + a.
 Untuk setiap bilangan bulat 𝑎, 𝑏 𝜖 𝑍 berlaku a × b = b × a.
3. Sifat Asosiatif
 Sifat asosiatif terhadap penjumlahan yaitu untuk sebarang bilangan bulat a,
b, dan c berlaku sifat (a+b) + c= a + (b+c)
 Sifat asosiatif terhadap perkalian yaitu untuk sebarang bilangan bulat a, b,
dan c berlaku (a × b) × c = a × (b × c)
4. Sifat Distributif
 Sifat distributif kiri perkalian terhadap penjumlahan, yaitu untuk sebarang
bilangan bulat a, b, dan c berlaku sifat a × (b + c) = (a × b) + (a × c)
 Sifat distributif kanan perkalian terhadap penjumlhan yaitu untuk sebarang
bilangan bulat a, b, dan c berlaku sifat (a + b) × c = (a × c) + (b × c)
2
5. Unsur Identitas Penjumlahan
Untuk setiap bilangan bulat a, selalu berlaku a + 0 = 0 + a = a sehingga 0
disebut unsur identitas penjumlahan.
6. Unsur identitas perkalian
Untuk setiap bilangan bulat a, ada dengan tunggal bilangan bulat 1 sehingga
a × 1 = 1 × a = 1 sehingga satu disebut unsur identitas perkalian.
Teorema Operasi Bilangan Bulat
1. Teorema Penjumlahan Bilangan Bulat
 Jika a, b, dan c anggota himpunan blangan bulat Z, dan a = b maka
a + c = b + c.
Bukti:
Ambil a, b, dan c anggoata Z
(a + c) Z (sifat tertutup)
(a + c) = (a + c) (sifat refleksi)
a = b (diberikan)
(a + c) = (b + c) (substitusi, 3 ke 2)
 Jika a, b, dan c anggota dari himpunan bilangan bulat Z, dan a + c = b + c
maka a = b
Bukti:
Ambil a, b, dan c di Z
(a + c) ∈ Z (sifat tertutup)
a + c = b + c
(a+c) + (-c) = b + (c + (-c)) (diberikan – c ∈ Z Invers tambahan)
a + (c + (-c)) = b + (c + (-c))
c + (-c) = 0
a + 0 = b + 0
a + 0 = a dan b + 0 = b
a = b
Teorema diatas biasanya dikenal dengan sifat penghapusan dari
penjumlahan.
3
 Bukti bahwa (-a) + (-b) = - (a + b)
Misalkan a dan b bilangan – bilangan cacah, maka (-a) + (-b) merupakan
jumlah dua bilangan negatif. Misal (-a) + (-b) = c maka c = (-a) + (-b).
c + b = ((-a) + (-b)) + b
c + b = (-a) + ((-b) + b)
c + b = (-a) + 0
(c + b) + a = (-a) + a
(c + b) + a = 0
c + (b + a) = 0
c + (a + b) = 0
(c + (a + b)) + (-(a + b)) = – (a + b)
c +((a + b) + (-(a + b))) = – (a + b)
c + 0 = – (a + b)
sifat kesamaan
sifat asosiatif penjumlahan
invers penjumlahan
sifat kesamaan
invers penjumlahan
sifat asosiatif penjumlahan
sifat komutatif penjumlahan
sifat kesamaan
sifat asosiatif
invers penjumlahan
Jadi kesimpulannya (-a) + (-b) = – (a + b)
2. Teorema Pengurangan Bilangan Bulat
 a – (-b) = a + b untuk sebarang bilangan bulat a dan b
Bukti: ambil bilangan bulat a dan b
a – (-b) = a + (-(-b)
=a+b
Definisi pengurangan
Teorema penjumlahan
 a - b = (a - c) - (b - c) untuk sebarang bilangan bulat a, b, dan c.
Bukti:ambil sebarang bilangan bulat a, b, dan c
a – b = a + (-b) Definisi Pengurangan
= ((a + (-b)) + 0 Identitas Tambahan
= a + (-b) + c + (-c) Invers Tambahan
= (a + (-c)) + ((-b) + c) Asosiatif Tambah
= (a + (-c)) + ((-b) + (-(-c))) Teorema Dalam Penjumlahan
= (a + (-c)) + (- (b + (-c))) Teorema Dalam Penjumlahan
= (a - c) - (b + (-c)) Definisi pengurangan
= (a - c) - (b - c) Definisi pengurangan
4
3. Teorema Perkalian Bilangan Bulat
 Jika a, b, dan c anggota himpunan bilangan bulat Z dan a = b maka
a × c = b × c
Bukti:
Ambil a, b, dan c di Z
(a × c) 𝜖 Z sifat tertutup
a × c = a × c sifat refleksi
a = b diberikan
a × c = b × c substitusi 3 ke 2
 Jika a, b, dan c anggota himpunan bilangan bulat Z maka
(a + b) × c = (a × c) + (b × c)
Bukti:
Ambil a, b, dan c di Z
(a + b) × c 𝜖 Z
(a + b) × c = c × (a + b)
c × (a + b) = (c × a) + (c × b)
(c × a) = (a × c) dan ((c × b) = (b × c)
(a + b) × c = (a × c) + (b × c)
 Jika a anggota bilangan bulat Z maka a × 0 = 0 dan 0 × a = 0
Bukti:
Ambil a, b, dan c di Z.
1) a = a
2) 0 = 0 + 0
3) a x 0 = a x (0 + 0)
4) a x 0 = (a x 0) + (a x 0)
5) 0 + (a x 0) = (a x 0)
6) 0 + (a x 0) = (a x 0) + (a x 0)
7) 0 = (a x 0)
8) (a x 0) = 0
9) (0 x a) = 0
 Jika a dan b anggota bilangan bulat Z maka a x (-b) = -(a x b).
Bukti:
a x (b + (-b)) = a x 0
(a x b) + (a x (-b)) = 0
(a x (-b)) + (a x b) = 0
5
((a x (-b)) + (a x b)) + (-(a x b)) = 0 + (- (a x b))
(a x (-b)) + ((a x b) + (-a (a x b))) = - (a x b)
a x (-b) + 0 = - (a x b)
a x (-b) = - (a x b)
 Buktikan bahwa (-a) (b + (-c)) = ac – ab.
Bukti:
(-a) (b + (-c)) = (-a)(b) + (-a)(-c) sifat distributif perkalian penjumlahan
= (-(ab)) + ac perkalian bilangan bulat
(-a) x b = -ab dan (-a) x (-c) = ac
= ac + (-(ab)) sifat komutatif perkalian
= ac – ab penjumlahan 2 bilangan bulat
(misal: a + (-b) = a – b)
Jadi terbukti bahwa (-a) (b + (-c)) = ac – ab.
4. Teorema Pembagian Bilangan Bulat
 Mengingat bahwa (-a) x (b)= (a) x (-b) = -(ab) dan berdasarkan defnisi
pembagian, kita dapat mengemukakan sifat berikut :
-(ab) ÷ a = (-b)
-(ab) ÷ b = (-a)
-(ab) ÷ (-a) = b
-(ab) ÷ (-b) = a
Demikian pula karena (-a) × (-b) = a × b maka:
ab ÷ (-a) = (-b)
ab ÷ (-b) = (-a)
Formula atau Rumus
1. Jika p dan q bilangan bulat maka berlaku:.
a. p x q = pq
b. (–p) x q = – (p x q) = –pq
c. p x (–q) = – (p x q) = –pq
d. (–p) x (–q) = p x q = pq
2. Untuk setiap p, q, dan r bilangan bulat berlaku sifat:
6
a. Tertutup pada operasi perkalian
b. Komutatif: p x q = q x p
c. Asosiatif: (p x q) x r = p x (q x r)
d. Distributif perkalian terhadap penjumlahan: p x (q + r) = (p x q) + (p x r)
e. Distributif perkalian terhadap pengurangan: p x (q – r) = (p x q) – (p x r).
3. Unsur identitas pada perkalian adalah 1, sehingga untuk setiap bilangan bulat
p berlaku p x 1 = 1 x p = p.
4. Pembagian merupakan operasi kebalikan dari perkalian.
5. Pada operasi pembagian bilangan bulat tidak bersifat tertutup.
Materi Pengayaan
1. Sederhanakan pecahan
200
100
.
Untuk menjawab pertanyaan tersebut, sebaiknya tidak mengarahkan siswa
untuk mencoret dua buah angka nol yang terdapat pada kedua bilangan,
meskipun hasilnya benar, karena hal tersebut akan menimbulkan miskonsepsi
pada siswa yang kurang memahaminya.
Contoh:
101
301
,
16
64
,
26
65
, dst. Meskipun beberapa bilangan bernilai benar saat dicoret
angka yang sama, hal ini tidak berlaku untuk semua bilangan.
2. T: Apakah semua bilangan bulat tertutup terhadap operasi penjumlahan?
J: Ya, karena jika dua buah bilangan bulat dijumlahkan, maka hasilnya adalah
bilangan bulat. Begitu pun juga pengurangan dan perkalian bilangan bulat.
3. T: Apakah semua bilangan bulat tertutup terhadap operasi pembagian?
J: Tidak, hal ini tidak berlaku pada
2
3
, dimana 2 dan 3 merupakan anggota
bilangan bulat, sedangkan hasil baginya bukan merupakan anggota bilangan
bulat.
4. Untuk menjelaskan perkalian sebagai penjumlahan berulang, cukup pada usia
SD kelas I-III.
7
Diskusi Kelompok
1. Apakah 3 x 2 = 2 + 2 + 2 atau 3 x 2 = 3 + 3?
Pilih salah satu konsep, karena keduanya belum tentu benar atau dapat
digunakan. Misalnya kita memilih 3 x 2 = 2 + 2 + 2 maka apabila ada
permasalahan -3 x 2 tidak dapat diselesaikan dengan cara yang sama dengan
3 x 2 = 2 + 2 + 2 tetapi dapat diselesaikan dengan cara
3 x 2 = 3 + 3 yaitu -3 x 2 = -3 x -3. Jadi kedua cara tersebut benar bergantung
dengan permasalahannya.
8
BILANGAN PECAHAN DAN PEMECAHAN MASALAH
Definisi:
Bilangan pecahan adalah bilangan yang dinyatakan dalam bentuk
b
a
, dengan a,b
B dan b bukan faktor dari a dan 𝑏 ≠ 0. a disebut pembilang dan b disebut penyebut.
Konsep
Jika a habis dibagi b maka bilangan itu adalah bilangan bulat (pecahan palsu), jika
tidak maka berupa pecahan.
Jenis-jenis Pecahan
Ditinjau dari perbandingan besar nilai pembilang dan penyebut, pecahan dibedakan
menjadi dua (2) yaitu:
a. Pecahan Sejati (Pecahan Murni)
Pecahan sejati adalah pecahan yang nilai positif pembilang lebih kecil dari nilai
positif penyebut. Contoh 2/3, 5/7, dan 9/10 adalah contoh-contoh bilangan
pecahan sejati
b. Pecahan Tidak Sejati (Pecahan Campuran)
Pecahan tidak sejati adalah pecahan yang nilai positif pembilang lebih besar
dari nilai positif penyebut. Contoh 10/7, 12/9, dan 2 1/4 adalah contoh-contoh
bilangan pecahan tak sejati.
Pecahan tak sejati 10/7 dapat ditulis dalam bentuk 1 3/7 , yang berarti 10/7 = 1
3/7. Pecahan dalam bentuk 1 3/7 disebut pecahan campuran. Jadi pecahan
campuran adalah pecahan yang penulisannya merupakan gabungan dari
bilangan bulat dan pecahan sejati.
Ditinjau dari nilai pembilang atau penyebutnya, dan hubungan antara pembilang
dan penyebut, pecahan dibedakan menjadi:
a. Pecahan Sederhana
Pecahan sederhana adalah pecahan yang FPB (Faktor Persekutuan Terbesar)
dari pembilang dan penyebutnya adalah 1. Contoh 5/7, 2/3, dan 5/3 adalah
contoh-contoh pecahan sederhana karena FPB dari pembilang dan penyebutnya
adalah 1.
9
b. Pecahan Senama
Pecahan senama adalah pecahan yang penyebutnya sama. Contoh 2/4, 3/4, dan
1/4 adalah contoh-contoh pecahan senama karena penyebutnya sama.
c. Pecahan Desimal
Pecahan desimal adalah pecahan yang penyebutnya berbentuk 10n
atau
jumlahan dari pecahan-pecahan yang penyebutnya berbentuk 10n
dengan n
bilangan asli. Contoh 1/10, 1/100, 1/1.000, 2/100, dan 0,03 adalah contoh-
contoh pecahan desimal.
Pecahan Satuan
Pecahan satuan adalah pecahan yang memiliki pembilang 1. Contoh:
1
3
,
1
2
,
1
7
,dst.
Contoh: Nyatakan
3
7
sebagai penjumlahan pecahan satuan.
Penyelesaian:
7
3
≈ 3 =>
3
7
−
1
3
=
9 − 7
21
=
2
21
21
2
≈ 11 =>
2
21
−
1
11
=
22 − 21
231
=
1
231
∴
3
7
=
1
3
+
1
11
+
1
231
Pecahan Kontinu
Pecahan kontinu ditulis sebagai:
𝑥 = 𝑎1 +
1
𝑎2 +
1
𝑎3 +
1
𝑎4 +
…
𝑎 𝑛
Juga dapat direpresentasikan seperti sebuah selang [𝑎1, 𝑎2, 𝑎3, … , 𝑎 𝑛]
Contoh: Tentukan nilai dari [7,4,2,5]
Penyelesaian:
[7,4,2,5] = 7 +
1
4+
1
2+
1
5
= 7 +
1
4+
1
11
5
= 7 +
1
4+
5
11
= 7 +
1
49
11
= 7 +
11
49
=
354
49
10
Selanjutnya, untuk menyatakan suatu pecahan menjadi pecahan kontinu dapat
menggunakan algoritma Euclides seperti contoh berikut.
T : Nyatakan
73
46
sebagai pecahan kontinu.
73 = 46(1) + 27
46 = 27(1) + 19
27 = 19(1) + 8
19 = 8(2) + 3
8 = 3(2) + 2
3 = 2(1) + 1
2 = 1(2)
Teorema 1: Jika 𝑎/𝑐 dan 𝑏/𝑐 pecahan-pecahan dengan c ≠ 0, maka
𝑎
𝑐
+
𝑏
𝑐
=
𝑎+𝑏
𝑐
.
Teorema 2: Jika a/b dan c/d pecahan-pecahan dengan c ≠ 0 maka
𝑎
𝑐
−
𝑏
𝑐
=
𝑎−𝑏
𝑐
.
Teorema 3 (Relasi Urutan):
Diketahui a/c dan b/c adalah pecahan-pecahan dengan c > 0. Pecahan a/c dikatakan
kurang dari b/c, yaitu a/c < b/c jika dan hanya jika a < b.
Teorema 4
Diketahui a/b dan c/d pecahan-pecahan dengan b > 0 dan d > 0
𝑎
𝑏
<
𝑐
𝑑
⇔ 𝑎𝑑 < 𝑏𝑐
Fakta: Bilangan pecahan
b
a
dibaca a dibagi b
Sifat pada Bilangan Pecahan
1. Penjumlahan pecahan memiliki sifat-sifat berikut:
a. Komutatif
b
a
d
c
d
c
b
a

b. Asosiatif













f
e
d
c
b
a
f
e
d
c
b
a
Dari Algoritma di samping, bilangan
73
46
dapat ditulis sebagai pecahan
kontinu [1,1,1,2,2,1,2]
11
2. Perkalian dan Pembagian Pecahan
a. Hasil perkalian dua pecahan diperoleh dengan mengalikan pembilang
dengan pembilang dan penyebut dengan penyebut.
bd
ac
db
ca
d
c
b
a




b. Untuk membagi suatu pecahan dengan pecahan lain sama artinya dengan
mengalikan pecahan pertama dengan kebalikan pecahan kedua
bc
ad
cb
da
c
d
b
a
d
c
b
a



:
Materi pengayaan
1. Nyatakan
13
14
sebagai jumlah pecahan unit yang berbeda.
2. Buktikan
8
11
tidak dapat dinyatakan ke dalam penjumlahan unit yang berbeda
yang banyaknya 4 buah pecahan unit.
3. Tuliskan bilangan rasional yang dinyatakan dengan [1,2,3,4,5,6].
4. Nyatakan dalam bentuk pecahan kontinu
177
233
.
Penyelesaian:
1. Dengan menggunakan pengurangan invers.
14
13
≈ 2 =>
13
14
−
1
2
=
13 − 7
14
=
6
14
14
6
≈ 3 =>
6
14
−
1
3
=
18 − 14
42
=
2
21
21
2
≈ 11 =>
2
21
−
1
11
=
22 − 21
231
=
1
231
∴
13
14
=
1
2
+
1
3
+
1
11
+
1
231
2. Akan dibuktikan,
8
11
tidak dapat dinyatakan ke dalam pecahan unit yang
banyaknya 4 buah.
11
8
≈ 2 =>
8
11
−
1
2
=
16 − 11
22
=
5
22
22
5
≈ 5 =>
5
22
−
1
5
=
25 − 22
110
=
3
110
12
110
3
≈ 37 =>
3
110
−
1
37
=
111 − 110
4070
=
1
4070
∴
8
11
=
1
2
+
1
5
+
1
37
+
1
4070
8
11
dapat dinyatakan ke dalam 4 buah pecahan satuan (tidak terbukti)
3. Bukti:
𝑝 + 𝑞 = 4𝑒𝑓 … … … … . . (𝑖)
𝑝 + 𝑒 = 𝑔𝑞 ↔ 𝑞 =
𝑝 + 𝑒
𝑔
… … … … … . (𝑖𝑖)
Substitusi (ii) ke (i)
↔ 𝑝 +
𝑝 + 𝑒
𝑔
= 4𝑒𝑓 … … … … … . . (𝑘𝑒𝑑𝑢𝑎 𝑟𝑢𝑎𝑠 𝑑𝑖𝑘𝑎𝑙𝑖 𝑔)
↔ 𝑝𝑔 + 𝑝 + 𝑒 = 4𝑒𝑓𝑔 … … … … … (𝑘𝑒𝑑𝑢𝑎 𝑟𝑢𝑎𝑠 𝑑𝑖𝑘𝑎𝑙𝑖
1
𝑒𝑓𝑔
)
↔
𝑝𝑔
𝑒𝑓𝑔
+
𝑝
𝑒𝑓𝑔
+
𝑒
𝑒𝑓𝑔
= 4 … … … … . (𝑘𝑒𝑑𝑢𝑎 𝑟𝑢𝑎𝑠 𝑑𝑖𝑘𝑎𝑙𝑖
1
𝑝
)
↔
𝑝𝑔
𝑒𝑓𝑔𝑝
+
𝑝
𝑒𝑓𝑔𝑝
+
𝑒
𝑒𝑓𝑔𝑝
=
4
𝑝
↔
1
𝑒𝑓
+
1
𝑒𝑓𝑔
+
1
𝑓𝑔𝑝
=
1
𝑝
∴ Terbukti.
4. Gunakan algoritma euclides.
[1,2,3,4,5,6]
↔ 5 +
1
6
=
31
6
↔ 4 +
6
31
=
130
31
↔ 3 +
31
130
=
421
130
↔ 2 +
130
421
=
972
421
↔ 1 +
421
972
=
1393
972
∴ [1,2,3,4,5,6] =
1393
972
13
5. Gunakan algoritma euclides.
177 = 233(0) + 177
233 = 177(1) + 56
177 = 56(3) + 9
56 = 9(6) + 2
9 = 2(4) + 1
2 = 1(2)
∴
177
233
= [0,1,3,6,4,2]
Diskusi Kelompok
Tunjukan bila 𝑝 + 𝑞 = 4𝑒𝑓; 𝑝 + 𝑒 = 𝑔𝑞, maka
4
𝑝
=
1
𝑒𝑓
+
1
𝑒𝑓𝑔
+
1
𝑓𝑔𝑝
.
Bukti:
𝑝 + 𝑞 = 4𝑒𝑓 … … … … . . (𝑖)
𝑝 + 𝑒 = 𝑔𝑞 ↔ 𝑞 =
𝑝 + 𝑒
𝑔
… … … … … . (𝑖𝑖)
Substitusi (ii) ke (i)
↔ 𝑝 +
𝑝 + 𝑒
𝑔
= 4𝑒𝑓 … … … … … . . (𝑘𝑒𝑑𝑢𝑎 𝑟𝑢𝑎𝑠 𝑑𝑖𝑘𝑎𝑙𝑖 𝑔)
↔ 𝑝𝑔 + 𝑝 + 𝑒 = 4𝑒𝑓𝑔 … … … … … (𝑘𝑒𝑑𝑢𝑎 𝑟𝑢𝑎𝑠 𝑑𝑖𝑘𝑎𝑙𝑖
1
𝑒𝑓𝑔
)
↔
𝑝𝑔
𝑒𝑓𝑔
+
𝑝
𝑒𝑓𝑔
+
𝑒
𝑒𝑓𝑔
= 4 … … … … . (𝑘𝑒𝑑𝑢𝑎 𝑟𝑢𝑎𝑠 𝑑𝑖𝑘𝑎𝑙𝑖
1
𝑝
)
↔
𝑝𝑔
𝑒𝑓𝑔𝑝
+
𝑝
𝑒𝑓𝑔𝑝
+
𝑒
𝑒𝑓𝑔𝑝
=
4
𝑝
↔
1
𝑒𝑓
+
1
𝑒𝑓𝑔
+
1
𝑓𝑔𝑝
=
1
𝑝
∴ Terbukti.
14
BARISAN DAN DERET
Barisan Bilangan
Definisi Barisan
Suatu fungsi berharga real yang didefinisikan pada himpunan bilangan bulat postif
disebut suatu barisan. Lazimnya barisan diberi simbol dengan (𝑎 𝑛), (𝑏 𝑛), (𝑈 𝑛) dan
sebagainya.
Selanjutnya:
𝑈1, 𝑈2, 𝑈3, … , 𝑈 𝑛, …
menyatakan barisan tak hingga atau dengan singkat barisan dan 𝑈1 adalah suku
pertama, 𝑈2 adalah suku ke-2, dan 𝑈 𝑛 adalah suku ke-𝑛 dari barisan (𝑈 𝑛). Bentuk
Umum:
Sifat Barisan
Suatu barisan (𝑈 𝑛) dikatakan:
1) Naik, jika dan hanya jika 𝑈 𝑛 < 𝑈 𝑛+1, ∀𝑛 > 0
2) Tidak turun, jika dan hanya jika 𝑈 𝑛 ≤ 𝑈 𝑛+1, ∀𝑛 > 0
3) Turun jika dan hanya jika 𝑈 𝑛 > 𝑈 𝑛+1, ∀𝑛 > 0
4) Tidak naik jika dan hanya jika 𝑈 𝑛 ≥ 𝑈 𝑛+1, ∀𝑛 > 0
Deret Bilangan
Definisi: Jumlah suku-suku dari suatu barisan di sebut deret. Bentuk umumnya
adalah sebagai berikut.
Sifat-sifat Deret
1) Jika setiap suku deret dikalikan dengan konstana yang tidak sama dengan 0
maka kekonvergenan (atau kedivergenan) deret tidak berubah.
𝑈1 + 𝑈2 + 𝑈3 + ⋯ + 𝑈 𝑛 = ∑ 𝑈𝑖
𝑛
𝑖=1
𝑈 𝑛 = 𝑈1, 𝑈2, 𝑈3, …
𝑈1, 𝑈2, 𝑈3,
15
2) Penghapusan (atau penambahan) sejumlah berhingga suku-suku daro (atau
terhadap) suatu deret tidak mengubaj kekonvergenan atau kedivergenan deret.
Barisan dan Deret Aritmatika
Barisan Aritmatika
Definisi: Barisan U1, U2, U3, ..., Un, ... disebut barisan aritmatika jika
Un - Un-1 = konstan,
dengan n= 2, 3, 4, ....
Konstanta pada barisan aritmatika di atas disebut beda dari barisan itu dan sering
dinotasikan dengan b, dan U1 sering dinotasikan dengan a.
Rumus
Jika U1 = a, U2, U3, ..., Un,... merupakan barisan aritmatika, maka unsur ke n
dari barisan itu dapat diturunkan dengan cara berikut.
U1 = a
U2= a + b
U3= U2 + b = (a + b) + b = a + 2b
U4= U3 + b = (a + 2b) + b = a + 3b
Un = a + (n - 1) b
Jadi rumus umum unsur ke n suatu barisan aritmatika dengan unsur pertama a
dan beda b adalah: Un = a + (n -1)b
Bentuk barisan aritmatika
Barisan aritmatika dapat ditulis menjadi dua bentuk, yaitu :
1) Eksplisit (𝑈 𝑛+1 =
1
2
𝑈 𝑛)
2) Rekrusif
Rekrusif adalah suku-suku berikutnya ditentukan oleh suku-suku sebelumnya.
Contoh : 𝑈 𝑛+1 =
𝑈 𝑛+1+𝑈 𝑛
2
16
Sifat-Sifat Barisan Aritmatika
Sifat pertama:
Apabila x, y, dan z merupakan bilangan yang berurutan dari suatu barisan
aritmatika, maka akan berlaku: "Dua kali bilangan yang ditengah sama dengan
jumlah dari kedua bilangan yang ada di sampingnya" (2y = x + z)
Sifat kedua:
Apabila w, x, y, z, empat bilangan yang berurutan dari suatu barisan aritmetika,
maka akan berlaku: "Jumlah dari dua bilangan yang terletak di tengah sama dengan
jumlah dari dua bilangan yang ada di sampingnya" (x + y = w + z)
Sifat Ketiga:
Apabila Un adalah suku ke-n barisan aritmetika maka berlaku: "Selisih antara
jumlah n suku pertama dan jumlah (n-1) suku pertama adalah suku ke-n
Un = Sn – Sn-1.
Barisan Aritmatika Bertingkat
Pada barisan ini tinggal melihat posisi suku sesudahnya
1) Tingkat pertama adalah barisan aritmetika itu sendiri.
2) Tingkat kedua (Kuadrat), jika pada pola selisih tingkat pertama membentuk lagi
selisih tetap seolah- olah sebagai selisih kedua.
Jika 𝑈 𝑛 = 𝑎𝑛2
+ 𝑏𝑛 + 𝑐, maka
a) 𝑎 =
𝑈3+𝑈1−2𝑈2
2
=
𝑠𝑒𝑙𝑖𝑠𝑖ℎ 𝑝𝑎𝑑𝑎 𝑡𝑖𝑛𝑔𝑘𝑎𝑡 𝑘𝑒𝑑𝑢𝑎
2
b) 𝑎 + 𝑏 + 𝑐 = 𝑈1
c) 3𝑎 + 𝑏 = 𝑈2 − 𝑈1
d) 5𝑎 + 𝑏 = 𝑈3 − 𝑈2
e) 2𝑎 = 𝑈1 + 𝑈3 − 2𝑈2
3) Tingkat ketiga
Jika pada pola selisih tingkat kedua membentuk lagi selisih tetap seolah-olah
sebagai selisih ketiga
Jika 𝑈 𝑛 = 𝑎𝑛3
+ 𝑏𝑛2
+ 𝑐𝑛 + 𝑑, dengan
a) 𝑈 𝑛 = rumus suku ke-n
17
b) 𝑎 =
𝑠𝑒𝑙𝑖𝑠𝑖ℎ 𝑡𝑒𝑟𝑎𝑘ℎ𝑖𝑟 𝑝𝑎𝑑𝑎 𝑡𝑖𝑛𝑔𝑘𝑎𝑡 𝑎𝑘ℎ𝑖𝑟
6
c) 𝑎 + 𝑏 + 𝑐 + 𝑑 = 𝑈1 = 𝑠𝑢𝑘𝑢 𝑝𝑒𝑟𝑡𝑎𝑚𝑎
d) 8𝑎 + 4𝑏 + 2𝑐 + 𝑑 = 𝑆2
e) 27𝑎 + 9𝑏 + 3𝑐 + 𝑑 = 𝑆3
4) Tingkat ke-n
Misalkan 𝑈1 = 𝐴 = suku pertama. B, C, D, dan seterusnya selanjutnya adalah
beda tiap tingkatan. Perhatikan bagan berikut:
A A+B A+2B+C A+3B+3C+D A+4B+6C+4D+E
B B+C B+2C+D B+3C+3D+E
C C+D C+2D+E
D D+……
Maka untuk
𝑈1 = 𝐴
𝑈2 = 𝐴 + 𝐵
𝑈3 = 𝐴 + 2𝐵 + 𝐶
𝑈4 = 𝐴 + 3𝐵 + 3𝐶 + 𝐷
……..
𝑈 𝑛 = 𝐴 +
(𝑛 − 1)
1!
𝐵 +
(𝑛 − 1)(𝑛 − 2)
2!
𝐶 +
(𝑛 − 1)(𝑛 − 2)(𝑛 − 3)
3!
𝐷
+ ⋯ …
Pola koefisien A,B, C, D dan seterusnya pada 𝑈 𝑛 mengikuti pola pada binom
Newton.
Contoh soal
1. Tentukan suku pertama, beda, suku kesepuluh dari barisan berikut ini:
a. 1, 4, 7, 10, ….
b. -2, 0, 2, 4, …
Jawab:
18
a. 1, 4, 7, 10, ….
a = 1, b = 3, U10 = 1 + (10 – 1).3
= 1 + 27 = 28
b. -2, 0, 2, 4, …
A = -2, b = 2, U10 = -2 + (10 – 1)2
= -2 + 18 = 16
2. Tentukan formula barisan berikut.
a. 0, 2, 6, 12, 20, ...
Jawab:
0 2 6 12 20
2 4 6 8
2 2 2
𝑈 𝑛 = 𝑎 + (𝑛 − 1)𝑏 +
(𝑛 − 1)(𝑛 − 2)𝑐
2
𝑈 𝑛 = 0 + (𝑛 − 1)2 +
(𝑛 − 1)(𝑛 − 2)2
2
𝑈 𝑛 = 2𝑛 − 2 + 𝑛2
− 3𝑛 + 2
𝑈 𝑛 = 𝑛2
− 𝑛
Jadi formula barisan tersebut adalah 𝑈 𝑛 = 𝑛2
− 𝑛
Deret Aritmatika
Definisi: Jika U1, U2, U3, ..., Un, .... merupakan barisan aritmatka, maka
U1 + U2 + U3 + ... + Un, ...
disebut deret aritmatika. Un disebut suku ke n dari deret itu.
Rumus
Jika Sn menyatakan jumlah n suku pertama deret aritmatika U1 + U2 +
U3 + ... + Un, ...., maka Sn = U1 + U2 + U3 + ... + Un dapat diturunkan
dengan cara sebagai berikut.
Sn = Un + (Un - b) + (Un - 2b) + ... + a
Sn = a + (a - b) + (a + 2b) +..... + Un +
2Sn = (a + Un) + (a + Un) + (a + Un) +... + (a + Un), sebanyak n suku.
𝑎 = 0, 𝑏 = 2, 𝑐 = 2
19
2Sn = n. (a + Un)
Sn = )(
2
1
nUan 
Jadi Sn = )(
2
1
nUan  atau Sn = ))1(2(
2
1
bnan 
Contoh soal
Diketahui deret aritmatika: 1 + 3 + 5 + 7 + 9 + …. Tentukan:
a. Rumus suku ke –n
b. Rumus jumlah n suku pertama
c. Jumlah 50 suku pertama
Jawab:
a. 1 + 3 + 5 + 7 + 9 + ….
Diperoleh a = 1, b = 2
Un = a + (n – 1)b
Un = 1 + ( n – 1)2
Un = 1 + 2n – 2
Un = 2n – 1
b. Sn =  bna
n
)1(2
2

Sn =  2)1(1.2
2
 n
n
=  222
2
 n
n
=  n
n
2
2
= n2
c. Sn = n2
 S50 = (50)2
= 2500
Barisan dan Deret Geometri
Definisi:
Barisan geometri ialah suatu barisan bilangan-bilangan dimana rasio di antara dua
suku berurutan merupakan bilangan tetap. Secara umum dapat dikatakan bahwa
barisan: U1, U2, U3, U4, … , Un disebut barisan geometri jika
32 4
1 2 3 1
... n
n
U UU U
U U U U 
    = konstanta.
20
Rumus
Rumus umum suku ke – n barisan geometri dengan suku pertama a dan rasio r
dapat ditemukan seperti berikut :
U1 = a
U2 = ar
U3 = ar2
U4 = ar3
Un = arn-1
Contoh soal
Suku ke-n dari barisan geometri adalah Un = 2n
. Tentukan :
a. Jumlah 8 suku pertama
b. Rumus jumlah n suku pertama
Jawab:
a. Un = 2n
 U1 = 21
= 2
U2 = 22
= 4
U3 = 23
= 8
2 + 4 + 8 + 16 + …
a = 2, r = 2, n = 8
)1(
)1(



r
ra
S
n
n 
)12(
)12(2 8
8


S 
)12(
)1256(2
8


S  S8 = 2.255 = 510
Deret Geometri
Definisi:
Jika diketahui U1, U2, U3, . . . , Un merupakan suku-suku dari barisan geometri,
maka U1 + U2 + U3 + . . . + Un disebut deret geometri , dengan Un = a rn-1
.
Rumus
Jika Sn merupakan jumlah n suku pertama dari suatu deret geometri, maka rumus
umum untuk Sn dapat ditentukan dengan langkah-langkah sebagai berikut :
Dimana :
a adalah suku pertama / nilai awal
r adalah rasio
21
Sn = U1 + U2 + U3 + . . . + Un maka
Sn = a + ar + ar2
+ . . . + arn-1
Kalikan Sn dengan r
rSn = ar + ar2
+ ar3
+ . . . + arn-1
+ arn
Kurangkan rSn dengan Sn
Sn = U1 + U2 + U3 + . . . + Un
rSn = ar + ar2
+ ar3
+ . . . + arn-1
+ arn
Sn - rSn = a - arn
Sn (1 – r) = a (1 - rn
)
Sn = a
n
(1 - r )
1 - r
Jadi rumus umum jumlah n suku pertama deret geometri adalah :
Sn = a
n
(1 - r )
1 - r
untuk r < 1
Sn = a
n
(r -1 )
r-1
untuk r > 1
Deret Tak Hingga
Jika suku-suku deret geometri bertambah terus mendekati tak hingga, maka:
𝑆 = lim
𝑛→∞
𝑆𝑛 = lim
𝑛→∞
𝑎(1 − 𝑟 𝑛
)
1 − 𝑟
= lim
𝑛→∞
𝑎
1 − 𝑟
− lim
𝑛→∞
𝑎
1 − 𝑟
𝑟 𝑛
=
𝑎
1 − 𝑟
−
𝑎
1 − 𝑟
lim
𝑛→∞
𝑟 𝑛
karena jumlah deret geometri tak hingga adalah 𝑆 =
𝑎
1−𝑟
−
𝑎
1−𝑟
lim
𝑛→∞
𝑟 𝑛
dan nilai
lim
𝑛→∞
𝑆𝑛 tergantung nilai lim
𝑛→∞
𝑟 𝑛
maka aka nada 2 kemungkinan harga S, yaitu:
i. Jika |𝑟| < 1 atau (−1 < 𝑟 < 1) maka nilai lim
𝑛→∞
𝑟 𝑛
= 0. Sehingga
𝑆 = lim
𝑛→∞
𝑆𝑛 =
𝑎
1−𝑟
yang selanjutnya deret ini kita sebut sebagai deret geometri
tak hingga yang konvergen.
ii. Jika |𝑟| ≥ 1 atau (𝑟 ≤ −1 atau 𝑟 ≥ 1), misalkan untuk nilai (𝑟 < −1 𝑎𝑡𝑎𝑢
22
𝑟 > 1) maka nilai lim
𝑛→∞
𝑟 𝑛
= ±∞. Sehingga 𝑆 = lim
𝑛→∞
𝑆𝑛 =
𝑎
1−𝑟
−
𝑎
1−𝑟
(±∞) =
±∞, maka deret geomteri tak hingga seperti ini tidak memiliki limit jumlah atau
divergen.
Barisan dan Deret Lain
Berikut diberikan deret lain selain deret aritmatika dan deret geometri, yaitu deret
teleskopih. Deret Teleskopik adalah deret yang suku – sukunya dapat saling
menghilangkan sehingga hanya ditentukan oleh suku pertama dan terakhir.
Adapun prinsip dari deret teleskopik:
1)
nn
nnk
n
k
k
aa
aaaaaaaaaa






1
1342312
1
1 )(...)()()()(
2) ∏
𝑎 𝑖+1
𝑎𝑖
𝑛
𝑖=1 =
𝑎2
𝑎1
𝑎3
𝑎2
𝑎4
𝑎3
…
𝑎 𝑛+1
𝑎 𝑛
=
𝑎 𝑛+1
𝑎1
3)
)1(
11
)1(
1


 xxxx
4)
1)1(
1
...
4.3
1
3.2
1
2.1
1




n
n
nn
5)
12)12)(12(
1
...
7.5
1
5.3
1
3.1
1




n
n
nn
6)
13)13)(23(
1
...
10.7
1
7.4
1
4.1
1




n
n
nn
7)
14)14)(34(
1
...
13.9
1
9.5
1
5.1
1




n
n
nn
8)
1
2.5
+
1
5.8
+
1
8.11
+ ⋯ +
1
(3𝑛−11)(3𝑛+2)
+=
𝑛
6𝑛+4
9)
1
𝑥.(𝑥+2)
=
1
2
(
1
𝑥
−
1
𝑥+2
)
10)
1
𝑥(𝑥+1)(𝑥+2)
=
1
2
(
1
𝑥(𝑥+1)
−
1
(𝑥+1)(𝑥+2)
)
11)
1
1.2.3
+
1
2.3.4
+
1
3.4.5
+ ⋯ +
1
𝑛.(𝑛+1)(𝑛+2)
+=
𝑛(𝑛+3)
4(𝑛+1)(𝑛+2)
12) 1 +
1
1+2
+
1
1+2+3
+
1
1+2+3+4
+ ⋯ +
1
1+2+3+⋯+𝑛
+=
2𝑛
𝑛+1
23
Materi Pengayaan
1. Jika 𝑋 𝑘+1 = 𝑋 𝑘 +
1
2
, 𝑘 = 1,2,3, …. Tentukan ∑ 𝑋𝑖
400
𝑖=1 dengan 𝑋1 = 1!
Jawaban:
𝑋 𝑘+1 = 𝑋 𝑘 +
1
2
Untuk k = 1 maka 𝑋2 =
3
2
Untuk k = 2 maka 𝑋3 = 2
Untuk k = 3 maka 𝑋4 =
5
2
∑ 𝑋𝑖 = 1 +
3
2
+ 2 +
5
2
+ ⋯ + 𝑋400
400
𝑖=1
𝑎 = 1, 𝑏 =
1
2
𝑆 𝑛 =
𝑛
2
(𝑎 + 𝑈 𝑛)
𝑆400 =
400
2
(1 + 399 ×
1
2
) = 40300
2. Asep memilih suku-suku barisan geometri tak hingga 1,1/2,1/4,1/8, ... untuk
membuat barisan geometri tak hingga baru yang jumlahnya 1/7 , tiga suku
pilihan asep adalah ?
Jawab:
Misal:
𝑈1 = (
1
2
)
𝑎
dan 𝑟 = (
1
2
)
𝑏
, 𝑎, 𝑏 ∈ 𝐵𝑖𝑙. 𝐴𝑠𝑙𝑖 maka,
𝑆∞ =
𝑎
1 − 𝑟
1
7
=
(
1
2
)
𝑎
1−(
1
2
)
𝑏 gunakan Aproksimasi 1 − (
1
2
)
𝑏
maka
1 ≤ 1 − (
1
2
)
𝑏
< 1 akibat 1 <
1
1−(
1
2
)
𝑏 ≤ 2 kalikan dengan (
1
2
)
𝑎
didapat,
(
1
2
)
𝑎
<
1
1 − (
1
2
)
𝑏 ≤ 2 (
1
2
)
𝑎
24
(
1
2
)
𝑎
≤
1
7
< (
1
2
)
𝑎−1
ambil sembarang nilai a anggota bilangan asli. Nilai 𝑎
yang memenuhi adalah 𝑎 = 3. Subtitusikan 𝑎 = 3 pada rumus jumlah tak
hingga di atas.
1
7
=
(
1
2
)
3
1−(
1
2
)
𝑏 didapat nilai 𝑏 = 3.
𝑈1 = (
1
2
)
𝑎
= (
1
2
)
3
=
1
8
dan 𝑟 = (
1
2
)
𝑏
= (
1
2
)
3
=
1
8
𝑈2 =
1
64
𝑈3 =
1
512
Jadi tiga suku pilihan Asep yaitu
1
8
,
1
64
, dan
1
512
3. Tentukan formula barisan berikut.
a. 0, 2, 6, 12, 20, ...
Jawab:
0 2 6 12 20
2 4 6 8
2 2 2
𝑈 𝑛 = 𝑎 + (𝑛 − 1)𝑏 +
(𝑛 − 1)(𝑛 − 2)𝑐
2
𝑈 𝑛 = 0 + (𝑛 − 1)2 +
(𝑛 − 1)(𝑛 − 2)2
2
𝑈 𝑛 = 2𝑛 − 2 + 𝑛2
− 3𝑛 + 2
𝑈 𝑛 = 𝑛2
− 𝑛
Jadi formula barisan tersebut adalah 𝑈 𝑛 = 𝑛2
− 𝑛
b. 1, 5, 19, 49, 101, ....
Jawab:
1 5 19 49 101
4 14 30 52
10 16 22
6 6
𝑎 = 0, 𝑏 = 2, 𝑐 = 2
𝑎 = 1, 𝑏 = 4, 𝑐 = 10, 𝑑 = 6
25
𝑈 𝑛 = 𝑎 + (𝑛 − 1)𝑏 +
(𝑛 − 1)(𝑛 − 2)𝑐
2
+
(𝑛 − 1)(𝑛 − 2)(𝑛 − 3)𝑑
6
𝑈 𝑛 = 1 + (𝑛 − 1)4 +
(𝑛 − 1)(𝑛 − 2)10
2
+
(𝑛 − 1)(𝑛 − 2)(𝑛 − 3)6
6
𝑈 𝑛 = 1 + (4𝑛 − 4) + 5(𝑛2
− 3𝑛 + 2) + (𝑛3
− 3𝑛2
+ 8𝑛 − 6)
𝑈 𝑛 = 𝑛3
+ 2𝑛2
− 3𝑛 + 1
Jadi formula barisan tersebut adalah 𝑈 𝑛 = 𝑛3
+ 2𝑛2
− 3𝑛 + 1
4. Diberikan suatu bilangan desimal 0,5151515151....... ubah kedalam bentuk
pecahan!
Jawab:
Untuk mengubah pecahan desimal diatas menjadi bentuk pecahan kita
gunakan konsep deret geometri. Deret geometri dari bilangan desimal
tersebut adalah sebagai berikut.
51
100
+
51
10000
+
51
10000000
+ ⋯
Didapat 𝑎 =
51
100
dan 𝑟 =
1
100
𝑆∞ =
𝑎
1 − 𝑟
=
51
100
1 −
1
100
𝑆∞ =
51
99
Jadi bentuk pecahan dari bilangan desimal 0,515151 adalah
51
99
5. ∑ (
𝑘−5
𝑘+2
)∞
𝑘=1
Jawab:
𝑆 𝑛 = −
4
3
+ (−
3
4
) + ⋯ +
𝑛 − 5
𝑛 + 2
+ ⋯
lim
𝑛→∞
𝑆 𝑛 = lim
𝑛→∞
𝑛−5
𝑛+2
= 1, karena nilai limitnya ada yaitu 1, maka deret
tersebut konvergen dan nilai deretnya mendekati nilai limitnya yaitu 1.
6. ∑ (
1
𝑘
−
1
𝑘+1
)∞
𝑘=2
Jawab: 𝑆 𝑛 =
1
6
+
1
12
+ ⋯ +
1
𝑛
−
1
𝑛+1
+ ⋯
26
lim
𝑛→∞
𝑆 𝑛 = lim
𝑛→∞
1
𝑛
−
1
𝑛+1
= 0, karena nilai limitnya ada yaitu 0, maka deret
tersebut konvergen dan nilai deretnya mendekati nilai limitnya yaitu 0.
7. ∑
2
(𝑘+2)𝑘
∞
𝑘=1
Jawab:
Cara 1
mengubah bentuk
2
(𝑘+2)𝑘
menjadi bentuk
𝐴
𝑘
+
𝐵
𝑘+2
2
(𝑘 + 2)𝑘
=
1
𝑘
−
1
𝑘 + 2
∑
2
(𝑘 + 2)𝑘
∞
𝑘=1
= ∑
1
𝑘
−
1
𝑘 + 2
∞
𝑘=1
∑
1
𝑘
−
1
𝑘 + 2
∞
𝑘=1
= (
1
1
−
1
3
) + (
1
2
−
1
4
) + (
1
3
−
1
5
) + ⋯ + (
1
𝑛
−
1
𝑛 + 2
)
∑
2
(𝑘 + 2)𝑘
∞
𝑘=1
= (1 +
1
2
) − (
1
𝑘 + 1
+
1
𝑘 + 2
)
Cara 2
𝑆 𝑛 =
2
3
+
1
4
+ ⋯ +
2
(𝑛 + 2)𝑛
+ ⋯
lim
𝑛→∞
𝑆 𝑛 = lim
𝑛→∞
2
(𝑛+2)𝑛
= 0, karena nilai limitnya ada yaitu 0, maka deret
tersebut konvergen dan nilai deretnya mendekati nilai limitnya yaitu 0.
Masalah Yang Dibahas Kelompok
1. Buktikan a =
𝑆 𝑛
𝑛
–
1
2
(n-1)b
Solusi:
Sn =
n
2
(a + Un)
2 Sn = n (2a+(n-1)b)
2 Sn = 2an + bn (n-1)
a =
2Sn-bn (n-1)
2n
27
a =
Sn
n
–
1
2
(n-1)b (terbukti)
INDUKSI MATEMATIS
Induksi Matematika adalah cara standar dalam membuktikan bahwa sebuah
pernyataan tertentu berlaku untuk setiap bilangan asli. Kita akan menotasikan
himpunan bilangan asli dengan ℕ = {1, 2, 3, . . . }, dengan operasi tambah dan
perkalian seperti biasa.
Bilangan Asli ℕ ini memenuhi sifat terurut sempurna (Well-Ordering
Property) yaitu “setiap himpunan bagian yang tidak kosong dari ℕ mempunyai
bilangan terkecil”. Sifat ini menyatakan bahwa S adalah himpunan bagian dari ℕ
dan S ≠ ∅, maka ada bilangan m  S sehingga m ≤ k untuk setiap k  S.
Secara formal Induksi Matematika ini bisa didenisikan sebagai berikut.
Definisi 1.1
Misalkan untuk setiap bilangan asli n kita mempunyai pernyataan P(n) yang
bisa benar atau salah. Misalkan
1. P(1) benar.
2. Jika P(n) benar, maka P(n + 1) benar.
Sehingga P(n) benar untuk setiap bilangan asli n.
Langkah 1 disebut dengan Langkah Dasar, sedangkan Langkah 2 disebut dengan
Langkah Induktif. Jika pada Langkah Induktif yang diasumsikan adalah pernyataan
P(i) benar untuk setiap bilangan i ≤ n, maka perumusan induksi matematika seperti
ini disebut Bentuk Kuat Induksi Matematika.
Tahapan Induksi Matematika
 Basis Step : Tunjukkan bahwa S(1) benar
 Inductive Step : Sumsikan S(k) benar
Akan dibuktikan S(k)  S(k+1) benar
 Conclusion : S(n) adalah benar untuk setiap n bilangan integer
positif
28
Prinsip Induksi Matematika
Misalkan S suatu himpunan bagian dari N yang mempunyai sifat:
1) 1 S
2) jika k  S maka k+1  S
Maka S = N
Prinsip Induksi Matematika ini mengatakan bahwa suatu himpunan bagian S dari
bilangan asli N di mana sifat (1) dan (2) dimiliki oleh himpunan itu, maka himpunan
bagian itu akan merupakan himpunan bilangan asli N atau S = N.
Materi Pengayaan
1. Buktikan dengan induksi matematika bahwa ∑ 𝑖 =
𝑛(𝑛+1)
2
𝑛
𝑖=1
Jawab:
Pilih i= 1 maka 1 =
1(1+1)
2
= 1 (benar)
Asumsikan untuk n = k benar maka Sk =
𝑘(𝑘+1)
2
maka akan ditunjukkan benar
untuk n = k+1
𝑆 𝑘+1 = 𝑆 𝑘 + (𝑘 + 1)
=
𝑘(𝑘+1)
2
+ 𝑘 + 1
=
𝑘(𝑘+1)
2
+
2(𝑘+1)
2
=
𝑘(𝑘+1)+2(𝑘+1)
2
=
(𝑘+1)(𝑘+2)
2
=
(𝑘+1)((𝑘+1)+1)
2
(terbukti)
2. Buktikan dengan induksi matematika bahwa ∑ 𝑖3𝑛
1 =
(𝑛(𝑛+1))
4
2
Jawab:
Misalkan 𝑛 = 1 maka
21
3
1
(1(1 1))
1
4


29
2
(1(2))
4
4
4
1



Benar
Asumsikan 𝑛 = 𝑘 benar
2
3
1
( ( 1))
4
k
k k
i


Akan dibuktikan 𝑛 = 𝑘 + 1 benar
2 31
3
1
( ( 1)) 4( 1)
4 4
k
k k k
i

 
 
2 2
2 2
2
( 1) ( 4 4)
4
( 1) ( 2)
4
(( 1)(( 1) 1))
4
k k k
k k
k k
  

 

  

(terbukti)
BILANGAN BERPANGKAT BULAT
Definisi:
Bentuk an
dengan a, n ∈ Z disebut bentuk pangkat. n disebut pangkat (eksponen)
dan a disebut bilangan pokok (basis). Untuk n bilangan asli an
berarti bilangan a
dikalikan sebanyak n kali.
  
faktorn
aaaa  ...
Contoh: 25
= 2 × 2 × 2 × 2 × 2 dan 52
= 5 × 5.
Untuk n bilangan bulat negatif maka 𝑎 𝑛
=
1
𝑎−𝑛
(n negatif maka – 𝑛 positif).
Pernyataan ini setara dengan 𝑎−𝑛
=
1
𝑎 𝑛
, untuk bilangan asli n. Definisi ini
sebenarnya mudah dipahami dari kenyataan bahwa 𝑎−1
adalah invers dari a pada
operasi perkalian, yakni:
30
0,
1
1 11
 
a
a
aaa sehingga   n
n
nnn
aa
aaa
1111






 
.
Contoh: 4−2
=
1
42 =
1
4×4
=
1
16
.
Sifat-sifat Bilangan Berpangkat
1. Sifat Perkalian Pangkat
Untuk mengalikan bilangan berpangkat dengan basis yang sama jumlahkan
pangkatnya.
Kasus 1: jika n dan m bilangan asli
mn
faktormn
faktormfaktorn
mn
aaaaaaaaaaa 


  
     ......
Kasus 2: jika n bilangan asli dan m bilangan negatif atau m bilangan asli
dan n bilangan negatif. Untuk kasus ini, kita hanya akan menimbulkan
n untuk kasus bilangan asli dan m bilangan negatif, sedangkan yang
lainnya dapat dilakukan dengan cara yang serupa, jika m bilangan
negatif maka (-m) bilangan positif.
mn
faktormn
faktorm
faktornmn
aaaaa
aaaa
aaaa
aa 





   
  
  
)(
...
...
...
Kasus 3: jika n dan m bilangan negatif
mn
mn
faktormfaktorn
mn
a
aaaaaaaaa
aa 




 )(
1
......
1
    
Kasus 4: n bilangan bulat dan m = 0 atau m bilangan bulat dan n = 0.
Untuk kasus ini, kita hanya akan membuktikan untuk kasus n bilangan
bulat dan m = 0, sedangkan yang lainnya dapat dilakukan dengan cara
yang serupa.
𝑎 𝑛
× 𝑎 𝑚
= 𝑎 𝑛
× 𝑎0
= 𝑎 𝑛
× 1 = 𝑎 𝑛
= 𝑎 𝑛+0
= 𝑎 𝑛+𝑚
Tentu saja untuk kasus a0
menyaratkan a ≠ 0.
Dari keempat kasus ini, lengkaplah pembuktian 𝑎 𝑛
× 𝑎 𝑚
= 𝑎 𝑛+𝑚
.
31
Contoh: 53
× 52
= 53+2
= 55
2. Sifat pembagian pangkat
Untuk membagi bilangan berpangkat dengan basis yang sama, kurangkan
pangkatnya.
mn
m
n
a
a
a 

Contoh:
67
64
= 67−4
= 63
.
3. Sifat perpangkatan dari pangkat
Untuk memangkatkan bilangan berpangkat dengan basis yang sama, kalikan
pangkatnya.
  mnmn
aa 

Contoh: (33)4
= 33×4
= 312
.
4. Sifat perpangkatan dari perkalian
Untuk mencari pangkat dari perkalian bilangan berpangkat, cari pangkat
dari masing-masing faktor kemudian kalikan.
  mmm
baba 
Contoh: (3 × 7)2
= 32
× 72
.
5. Sifat perpangkatan dari pembagian
Untuk mencari pangkat dari pembagian bilangan berpangkat, cari hasil
pemangkatan dari pembilang dan hasil pemangkatan dari penyebut
kemudian bagilah.
n
nn
b
a
b
a






Contoh: (
3
7
)
2
=
32
72
=
9
49
.
Perhatikan bahwa (𝑎 + 1)2
≠ 𝑎2
+ 12
, namun
(𝑎 + 1)2
= (𝑎 + 1)(𝑎 + 1) = 𝑎2
+ 2𝑎 + 12
dan perhatikan hal berikut:
(−2𝑤)2
= (−2𝑤)(−2𝑤) = 4𝑤2
−(2𝑤)2
= −(2𝑤 × 2𝑤) = −4𝑤2
32
dua hal yang cukup berbeda. Untuk itu hati-hati dalam melakukan operasi bilangan
berpangkat.
Teorema: Untuk bilangan real a, b, dan n, m bilangan bulat berlaku:
1. mnmn
aaa 

2. mn
m
n
a
a
a 

3.   mnmn
aa 

4.   mmm
baba 
5. m
mm
b
a
b
a






Teorema ini dapat diperluas dengan n dan m bilangan bulat.
BILANGAN PANGKAT RASIONAL
Definisi 1
Misalkan 𝑎 bilangan real dan 𝑎 ≠ 0, 𝑚, 𝑛 bilangan bulat didefinisikan 𝑎
𝑚
𝑛 = (𝑎
1
𝑛)
𝑚
Definisi 2
Misalkan a adalah bilangan real dan 𝑎 ≠ 0 dengan 𝑎 > 0,
𝑝
𝑞
adalah bilangan
pecahan 𝑞 ≠ 0, 𝑞 ≥ 2, 𝑎
𝑝
𝑞 = 𝑐 sehingga 𝑐 = √ 𝑎 𝑝𝑞
atau 𝑎
𝑝
𝑞 = √ 𝑎 𝑝𝑞
Sifat 1
Misalkan 𝑎 adalah bilangan real dan 𝑎 ≠ 0 dengan 𝑎 > 0,
𝑝
𝑛
dan
𝑚
𝑛
adalah
bilangan pecahan 𝑛 ≠ 0. Jika 𝑛, 𝑞 ≥ 2 maka (𝑎
𝑚
𝑛 ) (𝑎
𝑝
𝑛) = (𝑎)
𝑚+𝑝
𝑛 .
Sifat 2
Misalkan 𝑎 adalah bilangan real dan 𝑎 ≠ 0 dengan 𝑎 > 0,
𝑚
𝑛
dan
𝑝
𝑞
adalah
bilangan pecahan 𝑞, 𝑛 ≠ 0. maka (𝑎
𝑚
𝑛 ) (𝑎
𝑝
𝑞) = (𝑎)
𝑚
𝑛
+
𝑝
𝑞.
33
BENTUK AKAR
Definisi:
Untuk bilangan real a, b, dan n bilangan asli maka: abba nn

Untuk n = 2 bentuk 2
a cukup ditulis a . Bentuk a sering disebut akar ketika
a merupakan bilangan irasional.
Sifat-sifat bentuk akar
Untuk a, b, c, dan d bilangan real, berlaku:
1. Penjumlahan dan pengurangan bentuk akar
a. 𝑎√𝑏 + 𝑐√𝑏 = (𝑎 + 𝑐)√𝑏
b. 𝑎√𝑏 − 𝑐√𝑏 = (𝑎 − 𝑐)√𝑏
2. Perkalian dan pembagian bentuk akar
a. √ 𝑎 × √𝑏 = √𝑎 × 𝑏
b.
√ 𝑎
√𝑏
= √
𝑎
𝑏
, 𝑏 ≠ 0
c. 𝑎√𝑏 × 𝑐√𝑑 = (𝑎 × 𝑐)√𝑏 × 𝑑
d.
𝑐√ 𝑎
𝑑√𝑏
=
𝑐
𝑑
√
𝑎
𝑏
, 𝑏 ≠ 0, 𝑑 ≠ 0
Akan diperluas pembahasan tentang bilangan bentuk akar √ 𝑎 dengan
memperkenalkan definisi berikut.
Definisi:
Misalkan a bilangan real dan n bilangan rasional yang berbentuk 𝑛 =
𝑝
𝑞
, dengan
p, q, bilangan asli. Maka bentuk pangkat 𝑎 𝑛
dituliskan sebagai bentuk akar 𝑎 𝑛
=
𝑎
𝑝
𝑞 = √ 𝑎 𝑝𝑞
, dengan a tidak negatif saat p genap. Untuk q = 2, bentuk √ 𝑎 𝑝𝑞
cukup
ditulis sebagai √ 𝑎.
Untuk bilagan real a, b, dan n, m bilangan rasional berbentuk 𝑛 =
𝑝
𝑞
dan 𝑚 =
𝑠
𝑡
,
dengan p, q, s, t bilangan asli berlaku:
1. √ 𝑎 𝑝𝑞
× √ 𝑎 𝑠𝑡
= √𝑎 𝑝𝑡+𝑞𝑠
𝑞𝑡
34
2.
√𝑎 𝑝
𝑞
√𝑎 𝑠𝑡 = √𝑎 𝑝𝑡−𝑞𝑠
𝑞𝑡
3. √ 𝑎 𝑝𝑞
× √𝑏 𝑝
𝑞
= √(𝑎 × 𝑏) 𝑝𝑞
4.
√𝑎 𝑝
𝑞
√𝑏 𝑝
𝑞 = √(
𝑎
𝑏
)
𝑝𝑞
Dengan a dan b tidak negatif saat p atau s genap.
Materi Pengayaan
1. Apa definisi 𝑎 𝑛
?
Ada 2 syarat untuk 𝑎 dan n, yaitu :
𝑎 ∈ ℝ  {0} a anggota bilangan Real yang tidak nol
𝑛 ∈ ℕ n anggota bilangan asli
Jika 𝑎 ∈ ℝ, 𝑎 ≠ 0 akan dibuktikan bahwa 𝑎0
Ambil 𝑚, 𝑛 dengan 𝑚 = 𝑛
Maka
𝑎 𝑚
𝑎 𝑛
= 𝑎 𝑚−𝑛
= 𝑎 𝑚−𝑚
, karena 𝑚 = 𝑛
= 𝑎0
Kemudian dari
𝑎 𝑚
𝑎 𝑛
=
𝑎 𝑚
𝑎 𝑚
= 1
Jika 𝑎 ∈ ℝ, 𝑎 ≠ 0, 𝑛 ∈ ℤ+
∪ {0}
Akan dibuktikan bahwa 𝑎 𝑛
berlaku untuk 𝑛 ∈ ℤ−
𝑎0
𝑎 𝑛
= 𝑎0−𝑛
= 𝑎−𝑛
Untuk 𝑛 ∈ ℤ+
∪ {0} sudah dibuktikan
Misal 𝑛 ∈ ℤ+
, maka −𝑛 ∈ ℤ+
Karena
𝑎0
𝑎 𝑛
= 𝑎0−𝑛
= 𝑎−𝑛
𝑎 𝑛
berlaku untuk 𝑛 ∈ ℤ
35
Asimtot adalah sebuah persamaan yang didekati oleh sebuah garis.
lim
𝑥→𝑐
𝑓(𝑥) = ±∞ (Asimtot tegak)
lim
𝑥→+∞
𝑓(𝑥) = 𝐶 (Asimtot datar)
lim
𝑥→+∞
𝑓(𝑥) = 𝐶(Asimtot datar)
Masalah yang Didiskusikan Kelompok
Word Problems
Dalam sebuah ruangan terdapat seekor amoeba yang bereproduksi dengan
membelah diri menjadi dua setiap 30 menit. Jika diasumsikan tidak ada amoeba
yang mati selama 2 jam, maka berapakah banyak amoeba setelah 2 jam?
Jawab: 𝑈1 = 𝑎 = 1 𝑟 = 2
𝑆4 =
𝑎(𝑟 𝑛
− 1)
𝑟 − 1
=
1(24
− 1)
2 − 1
= 16 − 1
= 15
Jadi banyak amoeba setelah 2 jam yaitu 15 ekor
SIFAT-SIFAT KETERBAGIAN
Definisi 2.1
Suatu bilangan bulat x dikatakan habis dibagi oleh suatu bilangan bulat y ≠ 0, jika
terdapat satu bilangan bulat p sedemikian sehingga x = py. Jika hal ini dipenuhi
maka y dikatakan membagi x dan dinotasikan dengan y│x yang dapat diartikan
sebagai y adalah faktor (pembagi) x, atau x adalah kelipatan y. Jika y tidak membagi
x dinotasikan dengan 𝑦 ∤ 𝑥.
Atau : 𝑏|𝑎 ↔ ∃ 𝑐 ∈ 𝐼 ∋ 𝑎 = 𝑏. 𝑐
Contoh: −3 12⁄ ↔ ∃ − 4 ∈ 𝐼 ∋ 12 = (−4). (−3)
Definisi 3.1 d | n berarti terdapat suatu bilangan bulat k sedemikian sehingga n =
dk, sedangkan d ∤ n berarti bahwa d | n adalah salah. Dicatat bahwa 𝑎|𝑏 ≠
𝑎
𝑏
.
36
Suatu cara lain untuk menyatakan definisi dari d | n adalah seperti di bawah ini.
Definisi 3.2 d | n jika dan hanya jika n= 𝑑𝑘 untuk suatu k.
Teorema 3.3 (Sifat-sifat Keterbagian)
Jika n, m, dan d adalah bilangan-bilangan bulat maka pernyataan-pernyataan
berikut ini benar.
1) n | n (sifat refleksif)
2) d | n dan n | m → d | m (sifat transitif)
3) d | n dan d | m → d | an + bm untuk setiap bilangan bulat a dan b (sifat
linier)
4) d | n → ad | an untuk a ≠ 0 (sifat perkalian)
5) ad | an dan a ≠ 0 → d | n (sifat penghapusan)
6) 1 | n (1 membagi sembarang bilangan)
7) n | 1 →n = ± 1 (hanya 1 dan −1 yang merupakan pembagi dari 1)
8) d | 0 (sembarang nilai membagi nol)
9) 0 | n → n = 0 (nol hanya membagi nol)
10) d, n adalah positif dan d | n → d ≤ n (sifat perbandingan)
11) d | n dan d | (n + m) → d | m
Definisi 3.5 jika c = as + bt untuk suatu bilangan bulat s dan t, dikatakan bahwa c
merupakan suatu kombinasi linier dari a dan b.
Jadi pernyataan (3) dalam Teorema 3.3 mengatakan bahwa jika d membagi a dan
b, maka d membagi semua kombinasi linear dari a dan b. Khususnya, d membagi a
+ b dan a – b.
ALGORITMA EUCLID
Teorema Euclidean
Misalkan m dan n adalah dua buah bilangan bulat dengan syarat n > 0. Jika m
dibagi dengan n maka terdapat dua buah bilangan bulat unik q (quotient) dan r
(remainder), sedemikian sehingga m = nq + r dengan 0 ≤ r < n.
Contoh:
37
a. 1987 dibagi dengan 97. Berdasarkan teorema Euclidean menjadi 1987 = 97.
20 + 47. Memberikan hasil bagi 20 dan sisa 47.
b. −22 dibagi dengan 3 memberikan hasil bagi −8 dan sisa 2. Berdasarkan
teorema Euclidean menjadi −22 = 3 (−8) + 2. Tetapi −22 = 3 (−7) −1 salah
karena r = −1 tidak memenuhi syarat 0 ≤ r < n.
Algoritma Euclid
Jika r1, r2  Z, dan r1 > r2 dan dengan proses algoritma pembagian dibentuk Suatu
barisan menurun bilangan bulat r1, r2, r3, ... , rk-1, rk, rk+1=0
Yaitu:
r1 = q1r2 + r3 , 0 ≤ r3 < r2.
r2 = q2r3 + r4 , 0 ≤ r4 < r2.
r3 = q3r4 + r5
, 0 ≤ r5 < r2.
r4 = q4r5 + r6 , 0 ≤ r6 < r2.
.............................................
rk-2 = qk-2rk-1 + rk , 0 ≤ rk < r2.
rk-1 = qk-1rk + rk+1 , rk+1 = 0
Maka (r1,r2) = rk.
Bukti.
(r1,r2) = (q1r2 + r3 , r2) ....................... (substitusi r1)
= (r3,r2) ........................ (teorema)
= (r3, q2r3 + r4 ) ........................ (substitusi r2)
= (r3,r4)
.......
.......
.......
= (rk,rk+1)
= (rk,0) .......................... (rk+1 = 0)
(r1,r2) = rk
Contoh
1. Tentukan (105,60) dan nyatakan hasilnya sebagai bentuk kombinasi linear
38
ax + by = c, dimana c = (a,b).
Dengan Algoritma Euclides diperoleh:
105 = (1) 60 + 45
60 = (1) 45 + 15
45 = (3) 15 + 0, sehingga diperoleh (105,60) = 15.
Selanjutnya dengan jalan mundur diperoleh:
15 = 60 – 45 (1)]
= 60 – [105 – 60(1)]
= 60 – 105 + 60 (1)
= (-1) 105 + (2) 60.
Akhirnya diperoleh (105,60) = (-1)105 + (2) 60.
FPB dan KPK
- FPB atau faktor persekutuan terbesar dari 𝑎, 𝑏 adalah 𝑑 = 𝑓𝑝𝑏(𝑎, 𝑏) dengan
𝑑 merupakan bilangan terbesar yang memenuhi 𝑑|𝑎 dan 𝑑|𝑏.
- KPK atau kelipatan persekutuan terkecil dari 𝑎, 𝑏 adalah 𝑘 = 𝑘𝑝𝑘(𝑎, 𝑏)
dengan 𝑘 merupakan bilangan terkecil yang memenuhi sehingga 𝑎|𝑘 dan 𝑏|𝑘
Terdapat beberapa sifat yang berkaitan dengan fpb dan kpk di antaranya:
1. 𝑓𝑝𝑏(0,0) = 0
2. 𝑓𝑝𝑏 (𝑎, 1) = 1
3. 𝑓𝑝𝑏(𝑎, 𝑎) = |𝑎|
4. 𝑓𝑝𝑏(𝑎, 𝑎 + 1) = 1
5. Jika 𝑎 = 𝑚𝑝 dan 𝑏 = 𝑚𝑞 maka 𝑓𝑝𝑏(𝑎, 𝑏) = 𝑚. 𝑓𝑝𝑏(𝑝, 𝑞)
6. 𝑘𝑝𝑘(𝑎, 𝑏) =
𝑎.𝑏
𝑑
, dengan 𝑑 = 𝑓𝑝𝑏(𝑎, 𝑏)
Contoh: Tentukan fpb dan kpk dari 36,108, 64
Jawab:
Karena 36 = 22
32
, 108 = 22
33
, dan 64 = 26
, maka fpb adalah 4.
Dan kpk adalah 64.27 = 1728.
39
PERSAMAAN DIOPHANTINE LINEAR
Persamaan Diophantine yang paling sederhana adalah memuat dua variabel
pada umumnya dinyatakan dengan ax + by = c, dengan a,b,c  Z.
Teorema 1: jika FPB (a, b) = d maka ada bilangan bulat x dan y sehingga
ax + by = d
Penjelasan: teorema di atas sama dengan bentuk persamaan Diophantine
ax + by = FPB (a, b).
Bentuk Ax + By = C
Teorema 3.2: diberikan persamaan linier Diophantin secara umum dalam dua
variabel x, y ∈ Z: ax + by = c, dengan a, b , c adalah bilangan bulat misalkan d
= FPB (a, b) maka ax + by = c memiliki sebuah solusi jika dan hanya jika c
habis dibagi oleh d.
Teorema 7.1
Ditentukan a,b,c  Z dan d = ( a,b)
a. Jika d tidak membagi c maka persamaan ax + by = c tidak mempunyai
penyelesaian.
b. Jika d membagi c maka persamaan ax + by = c mempunyai penyelesaian
bulat yang tak hingga banyaknya, yaitu pasangan (x, y) dengan:
x = xo + (b/d) n dan y = yo – (a/d) n
Dengan n  Z dan (xo ,yo) adalah suatu penyelesaian khusus.
Masalah yang Diberikan Instruktur
1. Selesaikan persamaan-persamaan Diophantine 4x + 5y = 10.
Jawab: (4,5) = 1|10
Sesuai dengan Dalil Algoritma Euclides, karena (4,5) = 1 maka ada
11, yx sehingga ada 154 11  yx
Karena 5 = 1.4 + 1 atau 4 (-1) + 5 (1) = 1, maka x1= -1, y1 = -1
4 (-1) + 5 (1) = 1
10 [ 4 (-1) + 5 (1)] = 10 .1
40
4 (-10) + 5 (10) = 10 (ingat 4x + 5y = 10)
Jadi: xo = -10 dan yo = 10
Penyelesaian persamaan adalah x = -10 + 5k dan y = 10 - 4k dengan k  Z.
2. 9x + 12y = 21
Jawab:
(9, 12) = 3 |21, persamaan mempunyai penyelesaian.
Sesuai dengan Dalil Algoritma Euclides, karena (9, 12) = 3 maka tentu ada
x1,y1  Z sehingga 9 x1 + 12 y1 = 3
Karena 12 = 1.9 + 3 atau 9 (-1) + 12 (1) = 3, maka x1= -1, y1 = 1
9 (-1) + 12 (1) = 3
7 [ 9 (-1) + 12 (1)] = 7 .3
9 (-7) + 12 (7) = 21 (ingat 9x + 12y = 21)
Jadi: xo = -7 dan yo = 7
Penyelesaian persamaan adalah
x = xo + (b/d) t
= -7 + (12/3) t
= -7 + 4t , dengan t  Z
y = yo – (a/d) t
= 7 – (9/3) t
= 7 – 3t, dengan t  Z.
3. Selesaikan 4x + 6y = 7
Jawab: (4, 6) = 2. Karena 2 tidak dapat membagi habis 7 maka persamaan
tersebut tidak punya selesaian bilangan bulat.
4. Selesaikan 4x + 5y = 10 dengan cara reduksi!
Jawab:
4
2
2
4
510
5104
y
y
y
x
yx





Misal
4
2 y
t

 , maka
41
ty
yt
42
24


Substitusi y ke persamaan x
tx
tt
y
yx
5
)42(2
4
2
2




Sehingga diperoleh selesaian





ty
tx
42
50
di mana t  Z.
5. Selesaikan 3x + 8y = 11 dengan cara reduksi!
Jawab:
3
22
23
3
811
8113
y
y
y
x
yx





Misal
3
22 y
t

 , maka
2
1
2
32
223
t
t
t
y
yt





Misal
2
t
b

 maka bt 2
Substitusi b ke persamaan y
b
bbt
y 31
2
62
2
)2(32
2
32







Substitusi persamaan y ke persamaan x
b
bbb
x 81
3
243
3
24811
3
)31(811







Sehingga diperoleh selesaian





by
bx
31
81
di mana b  Z.
42
SIFAT-SIFAT DASAR KONGRUENSI
Definisi
Misalkan n ∈ N. a, b ∈ Z dikatakan kongruen modulo n, dinotasikan 𝑎 ≡
𝑏 (𝑚𝑜𝑑 𝑛), jika n membagi a - b. Berarti a - b = kn untuk k ∈ Z. atau a=b+kn
Contoh:
20 ≡ 6 (mod 7)
3 ≡ 38 (mod 7)
 Diberikan 𝑎 ∈ 𝑍, misalkan q dan r adalah hasil bagi dan sisa pembagian oleh
n, jadi 𝑎 = 𝑞𝑛 + 𝑟, dengan 0 ≤ 𝑟 < 𝑛.
 Berdasarkan definisi kongruensi 𝑎 ≡ 𝑟 (𝑚𝑜𝑑 𝑛). Karena ada n pilihan untuk
r, maka setiap bilangan bulat kongruen modulo n dengan tepat satu dari 0, 1,
2, ... , n-1.
 𝑎 ≡ 0 (𝑚𝑜𝑑 𝑛) ⇔ 𝑛|𝑎.
 {0, 1, 2, … , 𝑛 − 1} disebut himpunan sisa positif modulo n.
 koleksi bilangan bulat 𝑎1, 𝑎2, … , 𝑎 𝑛membentuk himpunan lengkap sisa
modulo n jika setiap bilangan bulat kongruen modulo n dengan satu dan
hanya satu 𝑎 𝑘.
 𝑎1, 𝑎2, … , 𝑎 𝑛 kongruen modulo n dengan 0, 1, 2, . . . , 𝑛 − 1.
Contoh:
{-12, -4, 11, 13, 22, 82, 91} adalah himpunan lengkap sisa modulo 7, karena
-12≡2, -4≡3, 11≡4, 13≡6, 22≡1, 82≡5, 91≡0 semua modulo 7.
Teorema 4.1
𝑎, 𝑏 ∈ 𝑍, 𝑎 ≡ 𝑏(𝑚𝑜𝑑 𝑛) ⇔ 𝑎 dan 𝑏 mempunyai sisa tak negatif yang sama jika di
bagi 𝑛.
Bukti:
(⇒)
 𝑎 ≡ 𝑏(𝑚𝑜𝑑 𝑛) ⇒ 𝑎 = 𝑏 + 𝑘𝑛, untuk suatu k∈ 𝑍.
 b dibagi n mempunyai sisa 𝑟 ⇒ 𝑏 = 𝑞𝑛 + 𝑟, dengan 0 ≤ 𝑟 < 𝑛.
43
 ⇒ 𝑎 = 𝑏 + 𝑘𝑛 = (𝑞𝑛 + 𝑟) + 𝑘𝑛 = (𝑞 + 𝑘)𝑛 + 𝑟.
 ⇒ 𝑎 dan b mempunyai sisa yang sama jika dibagi 𝑛
(⇐)
Misalkan 𝑎 = 𝑞1 𝑛 + 𝑟 dan 𝑏 = 𝑞2 𝑛 + 𝑟, dengan 0 ≤ 𝑟 < 𝑛.
⇒ 𝑎 − 𝑏 = (𝑞1 𝑛 + 𝑟) −(𝑞2 𝑛 + 𝑟) = (𝑞1 − 𝑞2)𝑛 ⇒ 𝑛|(𝑎 − 𝑏) ⇒ 𝑎 ≡ 𝑏(𝑚𝑜𝑑 𝑛)
Teorema 4.2
Diberikan 𝑛 ∈ 𝑁 dan a, b, c, d, ∈ 𝑍 sebarang. Maka
1. 𝑎 ≡ 𝑎(𝑚𝑜𝑑 𝑛)
2. 𝑎 ≡ 𝑏(𝑚𝑜𝑑 𝑛) ⇒ 𝑏 ≡ 𝑎(𝑚𝑜𝑑 𝑛)
3. 𝑎 ≡ 𝑏(𝑚𝑜𝑑 𝑛) dan 𝑏 ≡ 𝑐(𝑚𝑜𝑑 𝑛) ⇒ 𝑎 ≡ 𝑐(𝑚𝑜𝑑 𝑛)
4. 𝑎 ≡ 𝑏(𝑚𝑜𝑑 𝑛) dan 𝑐 ≡ 𝑑(𝑚𝑜𝑑 𝑛) ⇒ 𝑎 + 𝑐 ≡ 𝑏 + 𝑑(𝑚𝑜𝑑 𝑛) dan 𝑎𝑐 ≡
𝑏𝑑(𝑚𝑜𝑑 𝑛)
5. 𝑎 ≡ 𝑏(𝑚𝑜𝑑 𝑛) ⇒ 𝑎 + 𝑐 ≡ 𝑏 + 𝑐(𝑚𝑜𝑑 𝑛) dan 𝑎𝑐 ≡ 𝑏𝑐(𝑚𝑜𝑑 𝑛)
6. 𝑎 ≡ 𝑏(𝑚𝑜𝑑 𝑛) ⇒ 𝑎 𝑘
≡ 𝑏 𝑘
(𝑚𝑜𝑑 𝑛) untuk 𝑘 ∈ 𝑍 sebarang.
Bukti:
1. ∀𝑎 ∈ 𝑍 berlaku 𝑎 − 𝑎 = 0. 𝑛 ⇒ 𝑎 ≡ 𝑎(𝑚𝑜𝑑 𝑛)
2. 𝑎 ≡ 𝑏(𝑚𝑜𝑑 𝑛) ⇒ 𝑎 − 𝑏 ≡ 𝑘𝑛 untuk suatu 𝑘 ∈ 𝑍 ⇒ 𝑏 − 𝑎 = −(𝑘𝑛) =
(−𝑘)𝑛.
−𝑘 ∈ 𝑍 ⇒ 𝑏 ≡ 𝑎(𝑚𝑜𝑑 𝑛)
3. 𝑎 ≡ 𝑏(𝑚𝑜𝑑 𝑛), 𝑏 ≡ 𝑐(𝑚𝑜𝑑 𝑛) ⇒ ∃ℎ, 𝑘 ∈ 𝑍 ∋ 𝑎 − 𝑏 = ℎ𝑛 dan
𝑏 − 𝑐 = 𝑘𝑛 ⇒ 𝑎 − 𝑐 = (𝑎 − 𝑏) + (𝑏 − 𝑐) = ℎ𝑛 + 𝑘𝑛 = (ℎ + 𝑘)𝑛 ⇒ 𝑎
≡ 𝑐(𝑚𝑜𝑑 𝑛)
4. 𝑎 ≡ 𝑏(𝑚𝑜𝑑 𝑛) dan 𝑐 ≡ 𝑑(𝑚𝑜𝑑 𝑛) ⇒ ∃ℎ, 𝑘 ∈ 𝑍 ∋ 𝑎 − 𝑏 = ℎ𝑛 dan 𝑐 −
𝑑 = 𝑘𝑛 ⇒ (𝑎 + 𝑐) − (𝑏 + 𝑑) = ℎ𝑛 − 𝑘𝑛 = (ℎ − 𝑘)𝑛 ⇒ 𝑎 + 𝑐𝑏 +
𝑑(𝑚𝑜𝑑 𝑛) dan
𝑎𝑐 = (𝑏 + ℎ𝑛)(𝑑 + 𝑘𝑛) = 𝑏𝑑 + (ℎ𝑑 + 𝑏𝑘 + ℎ𝑘𝑛)𝑛, karena ℎ𝑑 + 𝑏𝑘 +
ℎ𝑘𝑛 ∈ 𝑍 ⇒ 𝑎𝑐 ≡ 𝑏𝑑(𝑚𝑜𝑑 𝑛)
5. 𝑐 ≡ 𝑐(𝑚𝑜𝑑 𝑛) dan (4) berlaku (5)
6. Jelas (6) berlaku untuk 𝑘 = 1.
44
Asumsikan berlaku untuk 𝑘 = 1 ⇒ 𝑎 𝑛
≡ 𝑏 𝑛(𝑚𝑜𝑑 𝑛) ⇒ 𝑎𝑎 𝑛
≡
𝑏𝑏 𝑛(𝑚𝑜𝑑 𝑛) ⇒ 𝑎 𝑛+1
≡ 𝑏 𝑛+1(𝑚𝑜𝑑 𝑛). Jadi ∀𝑘 ∈ 𝑁 berlaku 𝑎 ≡
𝑏(𝑚𝑜𝑑 𝑛)
⇒ 𝑎 𝑛
≡ 𝑏 𝑛(𝑚𝑜𝑑 𝑛)
Teorema 4.3
Jika 𝑐𝑎 ≡ 𝑐𝑏(𝑚𝑜𝑑 𝑛), maka 𝑎 ≡ 𝑏(𝑚𝑜𝑑 𝑛|𝑑), dengan 𝑑 = 𝑝𝑝𝑏(𝑐, 𝑛).
Bukti:
 𝑐(𝑎 − 𝑏) = 𝑐𝑎 − 𝑐𝑏 = 𝑘𝑛 , untuk suatu k ∈ Z
 𝑝𝑝𝑏(𝑐, 𝑛) = 𝑛 ⇒ ∃𝑟, 𝑠 yang relatif prima ∋ 𝑐 = 𝑑𝑟, 𝑛 = 𝑑𝑠
 ⇒ 𝑟(𝑎 − 𝑏) = 𝑘𝑠 ⇒ 𝑠|𝑟(𝑎 − 𝑏) ⇒ 𝑎 ≡ 𝑏(𝑚𝑜𝑑 𝑛|𝑑)
Akibat
Jika 𝑐𝑎 = 𝑐𝑏(𝑚𝑜𝑑 𝑛)dan ppb(𝑐, 𝑛) = 1, maka 𝑎 = 𝑏(𝑚𝑜𝑑 𝑛)
Tuliskan semua masalah yang diberikan instruktur beserta solusinya!
1050
dibagi 7 sisanya berapa?
Jawab:
Cara 1
10 ≡ 3 (𝑚𝑜𝑑 7)
102
≡ 3.10 (𝑚𝑜𝑑 7)
102
≡ 2 (𝑚𝑜𝑑 7)
103
≡ 6 (𝑚𝑜𝑑 7)
104
≡ 4(𝑚𝑜𝑑 7)
105
≡ 5 (𝑚𝑜𝑑 7)
106
≡ 1 (𝑚𝑜𝑑 7)
(106
)8
≡ 18 (𝑚𝑜𝑑 7)
1050
≡ 102 (𝑚𝑜𝑑 7)
𝟏𝟎 𝟓𝟎
≡ 𝟐 (𝒎𝒐𝒅 𝟕)
Cara 2
1050
≡ 350 (𝑚𝑜𝑑 7)
350
≡ (33
)16
. 33(𝑚𝑜𝑑 7)
350
≡ (−1)16
. 9(𝑚𝑜𝑑 7)
350
≡ 9(𝑚𝑜𝑑 7)
𝟏𝟎 𝟓𝟎
≡ 𝟐(𝒎𝒐𝒅 𝟕)
45
Atau cara 3
101
≡ 3 𝑚𝑜𝑑 7
102
≡ 2 𝑚𝑜𝑑 7
103
≡ 6 𝑚𝑜𝑑 7
104
≡ 4 𝑚𝑜𝑑 7
105
≡ 5 𝑚𝑜𝑑 7
106
≡ 1 𝑚𝑜𝑑 7
107
≡ 3 𝑚𝑜𝑑 7
Sisa dari hasil baginya berulang sebanyak 6
kali, maka
50
6
memiliki sisa 2, ada di kelas kedua, yaitu
pembagian dengan sisa 2. Maka sisa dari
1050
7
adalah 2
Jadi sisa dari 1050
di bagi 7 adalah 2.
Masalah yang didiskusikan dalam kelompok
1. Berapakah sisa 52011
dibagi 7?
2. Sisa pembagian 3247
+ 11 akan 17 adalah ...
3. Berapakah nilai digit terakhir dari 2111
?
4. Berapakah nilai dua digit terakhir dari 62018
?
Jawab
1. 52011
≡ 𝑎 (𝑚𝑜𝑑 7)
(57−1
)335+1 (𝑚𝑜𝑑 7) ≡ (1)335
. 51 (𝑚𝑜𝑑 7)
≡ 5 (𝑚𝑜𝑑 7)
Jadi sisa pembagian tersebut adalah 5.
2. (3247
+ 11)(𝑚𝑜𝑑 17)
32
≡ 9 (𝑚𝑜𝑑 17)
33
≡ 10 (𝑚𝑜𝑑 17)
34
≡ 13 (𝑚𝑜𝑑 17)
35
≡ 5 (𝑚𝑜𝑑 17)
36
≡ 15 (𝑚𝑜𝑑 17)
37
≡ 11 (𝑚𝑜𝑑 17)
38
≡ −1 (𝑚𝑜𝑑 17)
(38
)30
≡ (−1)30 (𝑚𝑜𝑑 17)
3240
≡ 1 (𝑚𝑜𝑑 17)
3247
≡ 107 (𝑚𝑜𝑑 17)
3247
≡ 11 (𝑚𝑜𝑑 17)
∴ (3247
+ 11)(𝑚𝑜𝑑 17) ≡ (11 + 11)(𝑚𝑜𝑑 17) ≡ 𝟓 (𝑚𝑜𝑑 17)
Jadi sisanya adalah 5
46
3. Digit terakhir dalam perpangkatan 2:
21
≈ 2
22
≈ 4
23
≈ 8
24
≈ 6
25
≈ 2
Digitnya berulang 4 periode, maka
digit terakhir dari perpangkatan
tersebut adalah
2111
≈ 24.27+3
≈ 23
≈ 𝟖
Jadi digit terakhirnya adalah 8.
4. Dua digit terakhir dari 62018
adalah
Jawab:
6 ≡ 6(𝑚𝑜𝑑100)
62
≡ 36(𝑚𝑜𝑑100)
63
≡ 16(𝑚𝑜𝑑100)
64
≡ 96(𝑚𝑜𝑑100)
65
≡ 76(𝑚𝑜𝑑100)
66
≡ 56(𝑚𝑜𝑑100)
67
≡ 36(𝑚𝑜𝑑100)
68
≡ 16(𝑚𝑜𝑑100)
69
≡ 96(𝑚𝑜𝑑100)
610
≡ 76(𝑚𝑜𝑑100)
611
≡ 56(𝑚𝑜𝑑100)
612
≡ 36(𝑚𝑜𝑑100)
Maka dari diatas dapat disimpulkan
62018
≡ (65
)403
63
(𝑚𝑜𝑑100)
62018
≡ 63
(𝑚𝑜𝑑100)
62018
≡ 16(𝑚𝑜𝑑100)
𝑗𝑎𝑑𝑖 𝑏𝑖𝑙𝑎𝑛𝑔𝑎𝑛 𝑑𝑢𝑎 𝑑𝑖𝑔𝑖𝑡 𝑡𝑒𝑟𝑎ℎ𝑖𝑟 𝑑𝑎𝑟𝑖 62018
𝑎𝑑𝑎𝑙𝑎ℎ 16
Definisi 4.1. Misalkan 𝑛 ∈ 𝑁. 𝑎, 𝑏 ∈ 𝑍 dikatakan kongruen modulo n, dinotasikan
𝑎 ≡ 𝑏(𝑚𝑜𝑑 𝑛), jika n membagi 𝑎 − 𝑏 berarti 𝑎 − 𝑏 = 𝑘𝑛 untuk 𝑘 ∈ 𝑍.
Contoh:
 3 ≡ 24(𝑚𝑜𝑑 7), −31 ≡ 11(𝑚𝑜𝑑 7), −15 ≡ −64 (𝑚𝑜𝑑 7)
Karena 3 − 24 = (−3)7, −31 − 11 = (−6)7, −15 − (−64) = 7.7.
 Setiap dua bilangan adalah kongruen modulo 1.
47
 Jika dua bilangan keduanya ganjil atau keduanya genap maka kedua bilangan
tersebut kongruen modulo 2.
Jika n < (a – b) maka dikatakan a tidak kongruan dengan b modulo n , dinotasikan
a # b.
Contoh: 25 # 12 (mod 7) karena 7 < (25 – 12).
Teorema 4.1. 𝑎, 𝑏 ∈ 𝑧, 𝑎 ≡ 𝑏(𝑚𝑜𝑑 𝑛) ↔ a dan b mempunyai sisa tak negatif sama
jika dibagi dengan n.
Contoh:
 –56 dan –11 dapat dinyatakan dalam bentuk −56 = (−7)9 + 7, −11 =
(−2)9 + 7 → −56 ≡ −11(𝑚𝑜𝑑 9)
 −31 ≡ 11 (𝑚𝑜𝑑 7) → −31 dan 11 mempunyai sisa yang sama jika dibagi 7
yaitu −31 = (−5)7 + 4, 11 = 1.7 + 4.
Teorema 4.2
Diberikan 𝑛 ∈ 𝑁 dan a,b,c,d ∈ ℤ, sebarang. Maka
1. a ≡ a (mod n).
2. a ≡ b (mod n)⇒ b ≡ a (mod n).
3. a ≡ b (mod n) dan b ≡ c (mod n) ⇒ a ≡ c (mod n).
4. a ≡ b (mod n) dan c ≡ d (mod n) ⇒ a+c ≡ b+d (mod n) dan ac ≡ bd (mod
n).
5. a ≡ b (mod n) ⇒ a+c ≡ b+c (mod n) dan ac ≡ bc (mod n).
6. a ≡ b (mod n) ⇒ ak
≡ bk
(mod n) untuk k∈ ℤ, sebarang.
Bukti:
1. ∀ a ∈ ℤ berlaku a-a = 0.n ⇒ a ≡ a (mod n)
2. a ≡ b (mod n) ⇒ a-b = kn untuk suatu k ∈ ℤ ⇒ b-a = -(kn) = (-k)n.
-k ∈ ℤ ⇒ b ≡ a (mod n)
3. a ≡ b (mod n), b ≡ c (mod n) ⇒ ∃ h,k ∈ ℤ ∋ a-b = hn dan b-c = kn ⇒
a-c = (a-b) + (b-c) = hn + kn = (h+k)n ⇒ a ≡ c (mod n).
4. a ≡ b (mod n) dan c ≡ d (mod n) ⇒ ∃ h,k ∈ ℤ ∋ a-b = hn dan c-d = kn ⇒
48
(a+c) – (b+d) = hn – kn = (h-k)n ⇒ a+c ≡ b+d (mod n) dan
ac = (b+hn)(d+kn)= bd+(hd+bk+hkn)n, karena hd+bk+hkn ∈ ℤ ⇒ ac≡ bd
(mod n)
5. c ≡ c (mod n) dan (4) berlaku (5)
6. Jelas (6) berlaku untuk k = 1
Asumsikan berlaku untuk k = n ⇒ an
≡ bn
(mod n) ⇒ aan
≡ bbn
(mod
n) ⇒ an+1
≡ bn+1
(mod n). Jadi ∀ k ∈ ℕ berlaku a ≡ b (mod n) ⇒ ak
≡
bk
(mod n)
Contoh:
Buktikan 220
− 1 dapat dibagi 41.
Bukti:
25
≡ −9(mod 41) ⇒ (25
)4
≡ (−9)5(mod 41) ⇒ 220
≡ 81.81 (mod 41)
81 ≡ −1(mod 41) ⇒ 81.81 ≡ 1 (mod 41) ⇒ 1 ≡ 81.81(mod 41)
⇒ 220
− 1 ≡ (81.81 − 81.81)(mod 41) ⇒ 220
− 1 ≡ 0(mod 41) ⇒
41| 220
− 1
Secara khusus, jika r adalah residu (sisa) pembagian a oleh m, maka a kongruen
dengan
r mod m dengan syarat mr 0 .
Contoh:
)4(mod2610 
)4(mod210 
)4(mod226 
Untuk setiap bilangan bulat adalah kongruen modulo m jika bilangan tersebut
berada diantara 0 dan (m–1). Bilangan m tersebut disebut modulus dari
kekongruenan. Kekongruenan dengan modulus yang sama memiliki sifat yang
hampir serupa dengan persamaan biasa.
49
Jika )(mod11 mba  dan )(mod22 mba  , maka:
i) )(mod2121 mbbaa 
ii) )(mod.. 2121 mbbaa 
Sifat di atas ada kemungkinan tidak berlaku untuk pembagian dalam
kekongruenan. Dengan kata lain, jika )(modmbcac  maka tidak selalu berlaku
)(modmba 
Teorema:
Jika )(modmcbca  , maka )/(mod dmba  , dengan d = fpb(c, m).
Bukti:
 c(a – b) = ca – cb = km, untuk suatu k .
 fpb(c, m) = m  r, s yang relatif prima  c = dr, m = ds.
 r(a – b) = ks s|r(a – b)  )/(mod dmba  .
Akibat:
Jika )(modmcbca  dan fpb(c, m) = 1, maka )(modmba  .
Contoh:
1. )10(mod2.202.15  , tetapi 15 ≢20 (mod 10).
2. Jika )(mod0. mvu  , maka dapat saja u ≢ 0 (mod m) dan v ≢ 0 (mod m).
Contoh:
)8(mod02.4 
4 ≢ 0 (mod 8) dan 2 ≢ 0 (mod 8)
Catatan:
 Kekongruenan yang memuat variabel dapat diselesaikan dengan cara yang
sama seperti menyelesaikan persamaan biasa.
 Dua buah persamaan yang memiliki variabel yang sama disebut ekuivalen jika
kedua persamaan tersebut memiliki penyelesaian yang sama.
50
Materi Pengayaan
1. Carilah penyelesaian dari )8(mod512 x
Jawab:
Berdasarkan sifat-sifat kongruensi linear, berlaku
)8(mod512 x
)8(mod7x
)8(mod1x
Jadi, penyelesaian dari )8(mod512 x yaitu )8(mod1x . Selesaian tersebut
tidak tunggal, selesaian yang lain yaitu {…, -15, -7, 1, 9, 17, 25, …}
2. Carilah penyelesaian dari )19(mod34 x .
Jawab:
Cara 1:
)19(mod34 x
)19(mod224 x
)19(mod11.22.2 x
)19(mod112 x
)19(mod302 x
)19(mod15.22 x
)19(mod15x
Cara 2:
4x – 3 = 19.k (k 
 )
4x = 19k + 3
4
319 

k
x
Pilih k = 3, diperoleh
15
4
33.19


x , sehingga )19(mod15x
Cara 3:
)19(mod34 x
)19(mod3.54.5 x
51
)19(mod1520 x
)19(mod15x
Jadi, penyelesaian dari )8(mod512 x adalah )19(mod15x .
3. Carilah penyelesaian dari )7(mod0122
 xx .
Jawab:
Cara 1:
Menurut definisi, )7(mod0122
 xx dapat ditulis 07122
 kxx ,
dimana k 
 . Ambil sebarang k sedemikian sehingga persamaan kuadrat
tersebut memiliki akar-akar biangan bulat.
Untuk k = 1 7122
 xx
0822
 xx
0)2)(4(  xx
x = -4 atau x = 2
Untuk k = 2 14122
 xx
01522
 xx
(x + 5) (x – 3) = 0
x = -5 atau x = 3
berdasarkan definisi nilai r yang memenuhi yaitu r = {0, 1, 2, …, 6}, sehingga
nilai yang memenuhi adalah 2 dan 3. Penyelesaian dari )7(mod0122
 xx
adalah )19(mod2x dan )19(mod3x .
Cara 2:
)7(mod0122
 xx
)7(mod7122
 xx
)7(mod0822
 xx
)7(mod0)2)(4(  xx
)7(mod3
)7(mod4
)7(mod04



x
x
x atau
)7(mod2
)7(mod02


x
x
52
Jadi, penyelesaian dari )7(mod0122
 xx adalah )7(mod3x dan
)7(mod2x .
4. Carilah penyelesaian dari )10(mod32
x .
Jawab:
Menurut definisi, )10(mod32
x dapat ditulis kx .1032

310
103
2
2


kx
kx
Untuk setiap nilai 
k , tidak ada bilangan k yang memenuhi .3102
 kx
Jadi, )10(mod32
x tidak mempunyai selesaian.
5. Carilah penyelesaian dari )514(mod156 x .
Jawab:
Oleh karena (6, 514) = 2 dan 2 tidak membagi habis 15, maka menurut teorema
)514(mod156 x tidak mempunyai selesaian.
6. Carilah penyelesaian dari )22(mod818 x .
Jawab:
Cara 1:
Menurut definisi, )22(mod818 x dapat ditulis kx .22818  .
kx .22818 
18
822 

k
x
Pilih k = 7 dan 16, sehingga diperoleh nilai x = 9 dan x = 20.
Cara 2:
)22(mod818 x
)22(mod4.29.2 x
)11(mod49 x
)11(mod159 x
)11(mod5.33.3 x
 )11(mod53 x
53
)11(mod5.43.4 x
)11(mod2012 x
)11(mod20x
 )11(mod53 x
)11(mod63 x
)11(mod2x
)11(mod9x
Jadi, penyelesaian dari )22(mod818 x adalah )11(mod20x dan
)11(mod9x
UJI PEMBAGIAN BILANGAN BULAT
Algoritma Pembagian
Teorema 3.22 (Algoritma Pembagian) Jika a dan b adalah bilangan bulat dan b >
0 maka terdapat secara tunggal bilangan bulat q dan r yang memenuhi dua
kondisi:
𝑎 = 𝑏𝑞 + 𝑟 dan 0 ≤ 𝑟 < 𝑏
Dalam situasi ini, q dinamakan hasil bagi (quotient) dan r dinamakan sisa
(remainder) ketika a dibagi oleh b. Dicatat bahwa terdapat dua bagian untuk hasil
ini, yaitu EKSISTENSI dan KETUNGGALAN dari bilangan bulat q dan r yang
memenuhi persamaan 𝑎 = 𝑏𝑞 + 𝑟 dan 0 ≤ 𝑟 < 𝑏.
Contoh (AHSME 1976)
Diambil r adalah sisa ketika 1059, 1417 dan 2312 dibagi oleh b > 1. Tentukan
nilai dari b−r.
Penyelesaian: Berdasarkan Algoritma Pembagian
1059 = q1b + r, 1417 = q2b + r, dan 2312 = q3b + r untuk bilangan-bilangan bulat
q1, q2, q3. Dari sini, 358 = 1417 − 1059 = b (q2 − q1), 1253 = 2312 − 1059 = b (q3−
q1), dan 895 = 2312 − 1417 = b (q3 − q2). Karena itu b | 358 atau b | 2 ∙ 179, b |
1253 atau b | 7 ∙ 179, dan b | 895 atau b | 5 ∙ 179. Karena b > 1, disimpulkan
bahwa b = 179. Jadi (sebagai contoh) 1059 = 5 ∙ 179 + 164, yang berarti bahwa r
= 164. Disimpulkan bahwa b − r = 179 − 164 = 15.
54
Suatu bilangan memiliki sifat tertentu jika dibagi oleh beberapa bilangan tertentu.
Beberapa sifat tersebut adalah:
1. Suatu bilangan dapat dibagi 2 jika dan hanya jika digit terakhirnya adalah
bilangan genap.
2. Suatu bilangan dapat dibagi 3 jika dan hanya jika jumlah digit-digitnya habis
dibagi 3.
3. Suatu bilangan dapat dibagi 4 jika dan hanya jika dua digit terakhir dari bilangan
tersebut habis dibagi 4.
4. Suatu bilangan dapat dibagi 5 jika dan hanya jika digit terakhir dari bilangan
tersbut adalah 0 atau 5.
5. Suatu bilangan dapat dibagi 6 jika dan hanya jika bilangan tersebut habis dibagi
2 dan 3.
6. Suatu bilangan dapat dibagi 7 jika dan hanya jika digit terakhir dikalikan dua,
kemudian dikurangkan dengan angka sebelumnya, hasilnya habis dibagi 7
(termasuk nol).
7. Suatu bilangan dapat dibagi 8 jika dan hanya jika tiga digit terakhirnya dapat
dibagi 8.
8. Suatu bilangan dapat dibagi 9 jika dan hanya jika jumlah digit-digitnya habis
dibagi 9.
9. Suatu bilangan dapat dibagi 10 jika dan hanya jika digit terakhirnya adalah nol.
10. Suatu bilangan habis dibagi 11 jika jumlah digit pada tempat ganjil dikurangi
jumlah digit pada tempat genap habis dibagi 11.
11. Suatu bilangan habis dibagi 12 jika dan hanya jika bilangan tersebut habis
dibagi 3 dan 4.
12. Suatu bilangan habis dibagi 13 jika dan hanya jika menghilangkan digit terakhir,
angka yang tersisa didepannya dikurangi dengan 9 kali digit yang dihilangkan,
hasilnya akan habis dibagi 13.
13. Suatu bilangan habis dibagi 2 𝑛
jika n digit terakhir bilangan tersebut habis
dibagi 2 𝑛
.
55
Contoh:
- 54627819280 habis dibagi 5 karena digit terakhirnya 0
- 2568312 habis dibagi 3 karena 2+5+6+8+3+1+2 = 27 habis dibagi 3
- Lebih jauh, bilangan 2568312 habis dibagi 9 karena jumlah digitnya habis
dibagi 9.
- 123453 habis dibagi 11 karena (1+3+5) - (2+4+3) = 0 habis dibagi 11.
- 2378264 habis dibagi 4 = 22
karena 64 habis dibagi 4.
Contoh 2.
Di antara empat bilangan: 5256, 7018, 18623, 32571, yang habis dibagi 99
adalah….
Jawab: Perhatikan bahwa 99=11.9 sehingga perlu diketahu diantara bilangan
tersebut yang dapat dibagi 11 juga 9.
- 5256 tidak habis dibagi 11 karena (5 + 5) − (2 + 6) = −2 tidak habis dibagi
11.
- 7018 habis dibagi 11 karena (7 + 1) − (0 + 8) = 0 habis dibagi 11.
- 18623 habis dibagi 11 karena (1 + 6 + 3) − (8 − 2) = 0 habis dibagi 11
- 32571 habis dibagi 11.
Di antara 7018, 18623,32571 hanya 32571 yang juga habis dibagi 9, sehingga
bilangan yang dapat dibai 99 adalah 32571
Contoh 3.
N adalah bilangan bulat terbesar dengan semua digitnya berbeda dan N merupakan
bilangan kelipatan 8. Tentukan tiga angka terakhir dari N?
Jawab: Perhatikan bahwa 8 = 23
sehingga 3 digit terakhir dari N harus dapat dibagi
8. Bilangan N terbesar dengan 3 digit terakhir yang dapat dibagi 8 dengan digit
berbeda adalah 120 (mengapa?).
56
KONGRUENSI LINEAR
Definisi 7.2 Suatu kongruensi yang mempunyai bentuk: ax ≡ b (mod m) dengan a,
b, m Z disebut suatu kongruensi linier satu variabel.
Perhatikan bahwa jika x = xo adalah suatu selesaian ax ≡ b (mod m), dan jika
diketahui bahwa x1 ≡ xo (mod m), maka ax1 ≡ axo (mod m), dengan demikian x1
juga suatu selesaian.
Definisi 1: perkongruenan linear 𝑎𝑥 ≡ 𝑏(𝑚𝑜𝑑 𝑚) akan memiliki
penyelesaian/solusi jika dan hanya jika ada bilangan bulat x dan k yang memenuhi
persamaan 𝑎𝑥 ≡ 𝑏 + 𝑘𝑚.
Contoh: 3𝑥 ≡ 4(𝑚𝑜𝑑 5). Jika x kita diganti dengan 3 akan memberikan 3.3 ≡
4(𝑚𝑜𝑑 5) merupakan pernyataan yang benar.
Teorema 1: jika gcd(𝑎, 𝑚) ∤ 𝑏 maka perkongruenan linear 𝑎𝑥 ≡ 𝑏(𝑚𝑜𝑑 𝑚) tidak
memiliki solusi.
Contoh: 6𝑥 ≡ 7(𝑚𝑜𝑑 8) karena gcd(6,8) = 2 dan 2 ∤ 7 maka perkongruenan ini
tidak memiliki solusi.
Teorema 2: jika gcd(𝑎, 𝑚) = 1 maka perkongruenan linear 𝑎𝑥 ≡ 𝑏(𝑚𝑜𝑑 𝑚)
mempunyai tepat satu solusi.
Contoh: 4𝑥 ≡ 1(𝑚𝑜𝑑 15), karena gcd(4, 15) adalah 1 maka tepat memiliki 1
solusi, maka tepat memiliki satu solusi, maka memungkinkan dilakukan konselasi
(penghapusan) pada 4 sehingga diperoleh 𝑥 ≡ 1(𝑚𝑜𝑑 15)
Contoh 7.4
Selesaikan kongruensi-kongruensi linier:
(a) 36x ≡ 8 (mod 102)
(b) 3x ≡ 2 (mod 5)
(c) 15x ≡ 6 (mod 18)
Jawab :
(a) (36,102) = 6 dan 6 tidak membagi 8, maka 36x ≡ 8 (mod 102) tidak mempunyai
selesaian.
57
(b) (3,5) = 1 dan 1│2, maka 3x ≡ 2 (mod 5) mempunyai satu selesaian x ≡ 4 (mod
5)
(c) (15,18) = 3 dan 3│6, maka 15x ≡ 6 (mod 18) mempunyai tiga selesaian, yaitu
x ≡ 4,10,16 (mod 18)
Contoh:
Tentukan bilangan prima yang kongruen dengan 1 modulo 4.
Jawab: Bilangan prima yang kongruen dengan 1 modulo 4 yaitu {5, 13, 17, 29, ...}.
5 = 22
+ 12
13 = 32
+ 22
17 = 42
+ 12
29 = 52
+ 22
Jadi, bilangan prima yang kongruen dengan 1 modulo 4 dapat dinyatakan dalam
penjumlahan 2 bilangan kuadrat.
Teorema: tidak ada bilangan prima p berbentuk 4k + 3 yang merupakan jumlah dua
bilangan kuadrat.
Bukti : 𝑎2
+ 𝑏2
𝑚𝑜𝑑 4 = 2 𝑚𝑜𝑑 4
= 1 𝑚𝑜𝑑 4
= 0 𝑚𝑜𝑑 4
Jadi karena 𝑎2
+ 𝑏2
≠ 3 𝑚𝑜𝑑 4, maka 𝑝 ≡ 3 𝑚𝑜𝑑 4 bukan merupakan jumlah dua
bilangan kuadrat.
Theorema Fermat: Suatu bilangan bulat positif n dapat dinyatakan sebagai jumlah
dua bilangan kuadrat jika dan hanya jika masing-masing faktor primanya
membentuk 4k + 3 terjadi pada pangkat genap
Contoh:
 459 = 33
× 17
 153 = 32
× 17 = 32(42
+ 12) = 122
+ 32
Misal : 𝑛 = 5 × 72
× 13 × 17
= 72(22
+ 12)(32
+ 22)(42
+ 12 )
58
= 72(22
+ 12)((12 + 2)2
+ (3 − 8)2)
= 72(22
+ 12)(142
+ 52)
= 72((28 + 5)5
+ (10 − 14)2)
= 72(332
+ 42)
= 2312
+ 282
Materi Pengayaan
Nyatakan 113, 229, dan 373 ke dalam penjumlahan dua bilangan kuadrat.
Jawab:
 113 = 82
+ 72
 229 = 152
+ 22
 373 = 182
+ 72
59
LATIHAN SOAL
1. Gunakan induksi matematika untuk membuktikan persamaan berikut ini benar
untuk setiap bilangan asli n.
a. 𝑆 𝑛 = 1 + 3 + 5 + 7 + ⋯ + (2𝑛 − 1) = 𝑛2
b. 5 𝑛
− 1 dapat dibagi 4
c. 1.2 + 2.3 + 3.4 + ⋯ + 𝑛(𝑛 + 1) =
𝑛(𝑛+1)(𝑛+2)
3
2. Diantara bilangan berikut: 911253243, 55284471, 73683654 yang habis dibagi
99 adalah….
3. Pada kompetisi sepak bola diikuti 5 klub (a, b, c, d, e). Masing-masing
membawa bendera untuk dikibarkan. Lima bendera diatur dalam 5 tiang
berjajar. Cara menempatkan lima bendera itu agar bendera klub a di tengah-
tengah adalah….
4. Selesaikan persamaan Diophantine 7𝑥 + 11𝑦 = 30!
5. 3𝑥 ≡ 5(𝑚𝑜𝑑11) dan 2𝑥 ≡ 7(𝑚𝑜𝑑11), nilai 𝑥2
+ 𝑦2
adalah….
6. Jika hari ini adalah hari Jumat, maka hari ke 102018
lagi adalah….
60
DAFTAR PUSTAKA
Fahrurrozi. 2007. Tip & Trik: Menyiasati Matematika. Yogyakarta: Teknomedia.
Kemendikbud. 2013. Buku Guru. Jakarta: Politeknik Negeri Media Kreatif.
Nuryadi. 2014. Bahan Ajar Teori Bilangan. Yogyakarta: Universitas Mercu
Buana.
Rinaldi. “Teori Kongruensi”.
http://dinus.ac.id/repository/docs/ajar/6.TeoriBilangan_.ppt (diakses 18 Februari
2018)
Marwati, Rini. “Teori Bilangan”.
http://file.upi.edu/Direktori/FPMIPA/JUR._PEND._MATEMATIKA/1966062519
90012-RINI_MARWATI/25_HandOutTeoriBilangan3.pdf (diakses 20 Februari
2018)
Tasnan, Don. “Sifat-sifat Operasi Himpunan”.
http://repository.binus.ac.id/content/K0064/K006426156.doc (diakses 21 Februari
2018)
https://julanhernadi.files.wordpress.com/2011/05/teori_bilangan_bab3_1.pdf
(diakses 20 Februari 2018)
Irawan, Edy Bambang. “Bilangan 1”.
http://repository.ut.ac.id/4698/2/PEMA4130-M1.pdf (diakses 24 Februari 2018)
Muhsetyo, Gatot. “Modul 4 Kongruensi Linear”.
https://www.slideshare.net/ChikaGidyfa/modul-4-kongruensi-
linier?from_action=save (diakses 25 Februari 2018)
http://jadijuara.com/contoh-soal-induksi-matematika/ (diakses 25 Februari 2018)

More Related Content

What's hot

bilangan bulat (Meidytha puti sabrina)
bilangan bulat (Meidytha puti sabrina)bilangan bulat (Meidytha puti sabrina)
bilangan bulat (Meidytha puti sabrina)
MuhammadAgusridho
 
Keterbagian
KeterbagianKeterbagian
B ab 01 metode numerik secara umum
B ab  01 metode numerik secara umumB ab  01 metode numerik secara umum
B ab 01 metode numerik secara umum
alamsyah88
 
Persamaan dan Pertidaksamaan Kuadrat, Fungsi Kuadrat, dan Diskriminan
Persamaan dan Pertidaksamaan Kuadrat, Fungsi Kuadrat, dan DiskriminanPersamaan dan Pertidaksamaan Kuadrat, Fungsi Kuadrat, dan Diskriminan
Persamaan dan Pertidaksamaan Kuadrat, Fungsi Kuadrat, dan Diskriminan
hari wihana
 
Eksponen
EksponenEksponen
Eksponen
kusnadiyoan
 
Teori bilangan bab ii
Teori bilangan bab iiTeori bilangan bab ii
Teori bilangan bab iiSeptian Amri
 
Jawab soal UNBK matematika SMK 2017 tipe soal A
Jawab soal UNBK matematika SMK 2017 tipe soal AJawab soal UNBK matematika SMK 2017 tipe soal A
Jawab soal UNBK matematika SMK 2017 tipe soal A
Sepriano Sepriano
 
Persamaan Kuadrat Kelas 9
Persamaan Kuadrat Kelas 9Persamaan Kuadrat Kelas 9
Persamaan Kuadrat Kelas 9
Erni Susanti
 
Pertemuan ke ii himpunan
Pertemuan ke ii himpunanPertemuan ke ii himpunan
Pertemuan ke ii himpunan
UNIVERSITAS MUHAMMADIYAH BERAU
 
Kelipatan persekutuan terkecil KPK teobil
Kelipatan persekutuan terkecil KPK teobilKelipatan persekutuan terkecil KPK teobil
Kelipatan persekutuan terkecil KPK teobil
Nailul Hasibuan
 
Polinomial (Suku Banyak)
Polinomial (Suku Banyak)Polinomial (Suku Banyak)
Polinomial (Suku Banyak)
shafirahany22
 
Persamaan dan fungsi kuadrat
Persamaan dan fungsi kuadratPersamaan dan fungsi kuadrat
Persamaan dan fungsi kuadrat
yulika usman
 
Nilai trigonometri
Nilai trigonometriNilai trigonometri
Nilai trigonometri
Muhlisul Anwar
 
Bahan ajar MK Matematika "Sistem Bilangan"
Bahan ajar MK Matematika "Sistem Bilangan"Bahan ajar MK Matematika "Sistem Bilangan"
Bahan ajar MK Matematika "Sistem Bilangan"
Muhammad Lyan Pratama
 
Persamaan & pertidaksamaan kuadrat 1
Persamaan & pertidaksamaan kuadrat 1Persamaan & pertidaksamaan kuadrat 1
Persamaan & pertidaksamaan kuadrat 1
herygumeg
 
Kuncijawaban
KuncijawabanKuncijawaban
Kuncijawaban
fondaessa
 
Persamaan dan fungsi kuadrat
Persamaan dan fungsi kuadratPersamaan dan fungsi kuadrat
Persamaan dan fungsi kuadratArikha Nida
 
Polinomial (Suku Banyak)
Polinomial (Suku Banyak)Polinomial (Suku Banyak)
Polinomial (Suku Banyak)
shafirahany22
 

What's hot (20)

Keterbagian
KeterbagianKeterbagian
Keterbagian
 
bilangan bulat (Meidytha puti sabrina)
bilangan bulat (Meidytha puti sabrina)bilangan bulat (Meidytha puti sabrina)
bilangan bulat (Meidytha puti sabrina)
 
Keterbagian
KeterbagianKeterbagian
Keterbagian
 
B ab 01 metode numerik secara umum
B ab  01 metode numerik secara umumB ab  01 metode numerik secara umum
B ab 01 metode numerik secara umum
 
Persamaan dan Pertidaksamaan Kuadrat, Fungsi Kuadrat, dan Diskriminan
Persamaan dan Pertidaksamaan Kuadrat, Fungsi Kuadrat, dan DiskriminanPersamaan dan Pertidaksamaan Kuadrat, Fungsi Kuadrat, dan Diskriminan
Persamaan dan Pertidaksamaan Kuadrat, Fungsi Kuadrat, dan Diskriminan
 
keterbagian
keterbagianketerbagian
keterbagian
 
Eksponen
EksponenEksponen
Eksponen
 
Teori bilangan bab ii
Teori bilangan bab iiTeori bilangan bab ii
Teori bilangan bab ii
 
Jawab soal UNBK matematika SMK 2017 tipe soal A
Jawab soal UNBK matematika SMK 2017 tipe soal AJawab soal UNBK matematika SMK 2017 tipe soal A
Jawab soal UNBK matematika SMK 2017 tipe soal A
 
Persamaan Kuadrat Kelas 9
Persamaan Kuadrat Kelas 9Persamaan Kuadrat Kelas 9
Persamaan Kuadrat Kelas 9
 
Pertemuan ke ii himpunan
Pertemuan ke ii himpunanPertemuan ke ii himpunan
Pertemuan ke ii himpunan
 
Kelipatan persekutuan terkecil KPK teobil
Kelipatan persekutuan terkecil KPK teobilKelipatan persekutuan terkecil KPK teobil
Kelipatan persekutuan terkecil KPK teobil
 
Polinomial (Suku Banyak)
Polinomial (Suku Banyak)Polinomial (Suku Banyak)
Polinomial (Suku Banyak)
 
Persamaan dan fungsi kuadrat
Persamaan dan fungsi kuadratPersamaan dan fungsi kuadrat
Persamaan dan fungsi kuadrat
 
Nilai trigonometri
Nilai trigonometriNilai trigonometri
Nilai trigonometri
 
Bahan ajar MK Matematika "Sistem Bilangan"
Bahan ajar MK Matematika "Sistem Bilangan"Bahan ajar MK Matematika "Sistem Bilangan"
Bahan ajar MK Matematika "Sistem Bilangan"
 
Persamaan & pertidaksamaan kuadrat 1
Persamaan & pertidaksamaan kuadrat 1Persamaan & pertidaksamaan kuadrat 1
Persamaan & pertidaksamaan kuadrat 1
 
Kuncijawaban
KuncijawabanKuncijawaban
Kuncijawaban
 
Persamaan dan fungsi kuadrat
Persamaan dan fungsi kuadratPersamaan dan fungsi kuadrat
Persamaan dan fungsi kuadrat
 
Polinomial (Suku Banyak)
Polinomial (Suku Banyak)Polinomial (Suku Banyak)
Polinomial (Suku Banyak)
 

Similar to Arrini ditta margarani (2016066155)

Ppt heppi pryitno
Ppt heppi pryitnoPpt heppi pryitno
Ppt heppi pryitno
HeppiPrayitno
 
Rangkuman materi un matematika smp revised
Rangkuman materi un matematika smp revisedRangkuman materi un matematika smp revised
Rangkuman materi un matematika smp revisedSafran Nasoha
 
Mtk Modul 3.2.pptx
Mtk Modul 3.2.pptxMtk Modul 3.2.pptx
Mtk Modul 3.2.pptx
namfyoid
 
Faizatul khayati
Faizatul khayatiFaizatul khayati
Faizatul khayatiyulia94
 
Faizatul khayati
Faizatul khayatiFaizatul khayati
Faizatul khayatiyulia94
 
Sistem Bilangan Lengkap
Sistem Bilangan LengkapSistem Bilangan Lengkap
Sistem Bilangan Lengkap
Putri Rizqi
 
Masbied com-kumpulan-rumus-matematika-smp
Masbied com-kumpulan-rumus-matematika-smpMasbied com-kumpulan-rumus-matematika-smp
Masbied com-kumpulan-rumus-matematika-smpWayan Sudiarta
 
kumpulan-rumus-matematika-smp
kumpulan-rumus-matematika-smpkumpulan-rumus-matematika-smp
kumpulan-rumus-matematika-smp
kandysaputra
 
Annisakhoerunnisya smt1 akuntansi1_bab2+bab3
Annisakhoerunnisya smt1 akuntansi1_bab2+bab3Annisakhoerunnisya smt1 akuntansi1_bab2+bab3
Annisakhoerunnisya smt1 akuntansi1_bab2+bab3
Annisa Khoerunnisya
 
BILANGAN BULAT DAN OPERASINYA 2.pptx
BILANGAN BULAT DAN OPERASINYA 2.pptxBILANGAN BULAT DAN OPERASINYA 2.pptx
BILANGAN BULAT DAN OPERASINYA 2.pptx
Budihermono
 
Makalah matematika
Makalah matematikaMakalah matematika
Makalah matematika
MutiaIranda
 
Bilangan Bulat
Bilangan BulatBilangan Bulat
Bilangan Bulat
Hariyatunnisa Ahmad
 
Matematika kelas-7 (1)
Matematika kelas-7 (1)Matematika kelas-7 (1)
Matematika kelas-7 (1)
Riya Tun PGMI
 
Operasi hitung bilangan bulat dalam pemecahan masalah copy
Operasi hitung bilangan bulat dalam pemecahan masalah   copyOperasi hitung bilangan bulat dalam pemecahan masalah   copy
Operasi hitung bilangan bulat dalam pemecahan masalah copyEddy Cla
 
Pert 1-DASAR-DASAR OPERASI BILANGAN.pptx
Pert 1-DASAR-DASAR OPERASI BILANGAN.pptxPert 1-DASAR-DASAR OPERASI BILANGAN.pptx
Pert 1-DASAR-DASAR OPERASI BILANGAN.pptx
UpayUpiww
 
Sistem bilangan cacah dan bulat Teobil
Sistem bilangan cacah dan bulat TeobilSistem bilangan cacah dan bulat Teobil
Sistem bilangan cacah dan bulat Teobil
Nailul Hasibuan
 
Modul matematika-kelas-xi-trigonometri
Modul matematika-kelas-xi-trigonometriModul matematika-kelas-xi-trigonometri
Modul matematika-kelas-xi-trigonometriAdrian Rama Putra
 
Modul matematika-kelas-xi-trigonometri
Modul matematika-kelas-xi-trigonometriModul matematika-kelas-xi-trigonometri
Modul matematika-kelas-xi-trigonometriAdrian Rama Putra
 
Materi Bilangan Kelas 7 Semester Ganjil.pptx
Materi Bilangan Kelas 7 Semester Ganjil.pptxMateri Bilangan Kelas 7 Semester Ganjil.pptx
Materi Bilangan Kelas 7 Semester Ganjil.pptx
IrwanIrwan785824
 

Similar to Arrini ditta margarani (2016066155) (20)

Ppt heppi pryitno
Ppt heppi pryitnoPpt heppi pryitno
Ppt heppi pryitno
 
Rangkuman materi un matematika smp revised
Rangkuman materi un matematika smp revisedRangkuman materi un matematika smp revised
Rangkuman materi un matematika smp revised
 
Mtk Modul 3.2.pptx
Mtk Modul 3.2.pptxMtk Modul 3.2.pptx
Mtk Modul 3.2.pptx
 
Faizatul khayati
Faizatul khayatiFaizatul khayati
Faizatul khayati
 
Faizatul khayati
Faizatul khayatiFaizatul khayati
Faizatul khayati
 
Sistem Bilangan Lengkap
Sistem Bilangan LengkapSistem Bilangan Lengkap
Sistem Bilangan Lengkap
 
Masbied com-kumpulan-rumus-matematika-smp
Masbied com-kumpulan-rumus-matematika-smpMasbied com-kumpulan-rumus-matematika-smp
Masbied com-kumpulan-rumus-matematika-smp
 
kumpulan-rumus-matematika-smp
kumpulan-rumus-matematika-smpkumpulan-rumus-matematika-smp
kumpulan-rumus-matematika-smp
 
Annisakhoerunnisya smt1 akuntansi1_bab2+bab3
Annisakhoerunnisya smt1 akuntansi1_bab2+bab3Annisakhoerunnisya smt1 akuntansi1_bab2+bab3
Annisakhoerunnisya smt1 akuntansi1_bab2+bab3
 
BILANGAN BULAT DAN OPERASINYA 2.pptx
BILANGAN BULAT DAN OPERASINYA 2.pptxBILANGAN BULAT DAN OPERASINYA 2.pptx
BILANGAN BULAT DAN OPERASINYA 2.pptx
 
Makalah matematika
Makalah matematikaMakalah matematika
Makalah matematika
 
1
11
1
 
Bilangan Bulat
Bilangan BulatBilangan Bulat
Bilangan Bulat
 
Matematika kelas-7 (1)
Matematika kelas-7 (1)Matematika kelas-7 (1)
Matematika kelas-7 (1)
 
Operasi hitung bilangan bulat dalam pemecahan masalah copy
Operasi hitung bilangan bulat dalam pemecahan masalah   copyOperasi hitung bilangan bulat dalam pemecahan masalah   copy
Operasi hitung bilangan bulat dalam pemecahan masalah copy
 
Pert 1-DASAR-DASAR OPERASI BILANGAN.pptx
Pert 1-DASAR-DASAR OPERASI BILANGAN.pptxPert 1-DASAR-DASAR OPERASI BILANGAN.pptx
Pert 1-DASAR-DASAR OPERASI BILANGAN.pptx
 
Sistem bilangan cacah dan bulat Teobil
Sistem bilangan cacah dan bulat TeobilSistem bilangan cacah dan bulat Teobil
Sistem bilangan cacah dan bulat Teobil
 
Modul matematika-kelas-xi-trigonometri
Modul matematika-kelas-xi-trigonometriModul matematika-kelas-xi-trigonometri
Modul matematika-kelas-xi-trigonometri
 
Modul matematika-kelas-xi-trigonometri
Modul matematika-kelas-xi-trigonometriModul matematika-kelas-xi-trigonometri
Modul matematika-kelas-xi-trigonometri
 
Materi Bilangan Kelas 7 Semester Ganjil.pptx
Materi Bilangan Kelas 7 Semester Ganjil.pptxMateri Bilangan Kelas 7 Semester Ganjil.pptx
Materi Bilangan Kelas 7 Semester Ganjil.pptx
 

Recently uploaded

RANCANGAN TINDAKAN UNTUK AKSI NYATA MODUL 1.4 BUDAYA POSITIF.pdf
RANCANGAN TINDAKAN UNTUK AKSI NYATA MODUL 1.4 BUDAYA POSITIF.pdfRANCANGAN TINDAKAN UNTUK AKSI NYATA MODUL 1.4 BUDAYA POSITIF.pdf
RANCANGAN TINDAKAN UNTUK AKSI NYATA MODUL 1.4 BUDAYA POSITIF.pdf
junarpudin36
 
Bahan Sosialisasi PPDB_1 2024/2025 Bandung
Bahan Sosialisasi PPDB_1 2024/2025 BandungBahan Sosialisasi PPDB_1 2024/2025 Bandung
Bahan Sosialisasi PPDB_1 2024/2025 Bandung
Galang Adi Kuncoro
 
NUMERASI KOMPETENSI PENDIDIK TAHAP CAKAP DAN MAHIR.pdf
NUMERASI KOMPETENSI PENDIDIK TAHAP CAKAP DAN MAHIR.pdfNUMERASI KOMPETENSI PENDIDIK TAHAP CAKAP DAN MAHIR.pdf
NUMERASI KOMPETENSI PENDIDIK TAHAP CAKAP DAN MAHIR.pdf
DataSupriatna
 
Tabel 1. 7 Ruang Lingkup Terintegrasi dalam Mata Pelajaran dalam CASEL PSE.pdf
Tabel 1. 7 Ruang Lingkup Terintegrasi dalam Mata Pelajaran dalam CASEL PSE.pdfTabel 1. 7 Ruang Lingkup Terintegrasi dalam Mata Pelajaran dalam CASEL PSE.pdf
Tabel 1. 7 Ruang Lingkup Terintegrasi dalam Mata Pelajaran dalam CASEL PSE.pdf
ppgpriyosetiawan43
 
EVIDENCE BASED DALAM PELAYANAN KB DAN KONTRASEPSI.pdf
EVIDENCE BASED DALAM PELAYANAN KB DAN KONTRASEPSI.pdfEVIDENCE BASED DALAM PELAYANAN KB DAN KONTRASEPSI.pdf
EVIDENCE BASED DALAM PELAYANAN KB DAN KONTRASEPSI.pdf
Rismawati408268
 
2. Kerangka Kompetensi Literasi Guru SD_Rev.pptx
2. Kerangka Kompetensi Literasi Guru SD_Rev.pptx2. Kerangka Kompetensi Literasi Guru SD_Rev.pptx
2. Kerangka Kompetensi Literasi Guru SD_Rev.pptx
arianferdana
 
LK 1 - 5T Keputusan Berdampak PERMATA BUNDA.pdf
LK 1 - 5T Keputusan Berdampak PERMATA BUNDA.pdfLK 1 - 5T Keputusan Berdampak PERMATA BUNDA.pdf
LK 1 - 5T Keputusan Berdampak PERMATA BUNDA.pdf
UditGheozi2
 
Pi-2 AGUS MULYADI. S.Pd (3).pptx visi giru penggerak dan prakrsa perubahan bagja
Pi-2 AGUS MULYADI. S.Pd (3).pptx visi giru penggerak dan prakrsa perubahan bagjaPi-2 AGUS MULYADI. S.Pd (3).pptx visi giru penggerak dan prakrsa perubahan bagja
Pi-2 AGUS MULYADI. S.Pd (3).pptx visi giru penggerak dan prakrsa perubahan bagja
agusmulyadi08
 
LAPORAN TUGAS TAMBAHAN PEMBINA PRAMUKA..
LAPORAN TUGAS TAMBAHAN PEMBINA PRAMUKA..LAPORAN TUGAS TAMBAHAN PEMBINA PRAMUKA..
LAPORAN TUGAS TAMBAHAN PEMBINA PRAMUKA..
widyakusuma99
 
VISI MISI KOMUNITAS BELAJAR SDN 93 KOTA JAMBI
VISI MISI KOMUNITAS BELAJAR SDN 93 KOTA JAMBIVISI MISI KOMUNITAS BELAJAR SDN 93 KOTA JAMBI
VISI MISI KOMUNITAS BELAJAR SDN 93 KOTA JAMBI
gloriosaesy
 
AKSI NYATA TAHAP PERKEMBANGAN PESERTA DIDIK JENJANG SD USIA 6-12 TAHUN.pptx
AKSI NYATA TAHAP PERKEMBANGAN PESERTA DIDIK JENJANG SD USIA 6-12 TAHUN.pptxAKSI NYATA TAHAP PERKEMBANGAN PESERTA DIDIK JENJANG SD USIA 6-12 TAHUN.pptx
AKSI NYATA TAHAP PERKEMBANGAN PESERTA DIDIK JENJANG SD USIA 6-12 TAHUN.pptx
adelsimanjuntak
 
Laporan Kegiatan Pramuka Tugas Tambahan PMM.pdf
Laporan Kegiatan Pramuka Tugas Tambahan PMM.pdfLaporan Kegiatan Pramuka Tugas Tambahan PMM.pdf
Laporan Kegiatan Pramuka Tugas Tambahan PMM.pdf
UmyHasna1
 
Paparan Kurikulum Satuan Pendidikan_LOKAKARYA TPK 2024.pptx.pdf
Paparan Kurikulum Satuan Pendidikan_LOKAKARYA TPK 2024.pptx.pdfPaparan Kurikulum Satuan Pendidikan_LOKAKARYA TPK 2024.pptx.pdf
Paparan Kurikulum Satuan Pendidikan_LOKAKARYA TPK 2024.pptx.pdf
SEMUELSAMBOKARAENG
 
UNTUK DOSEN Materi Sosialisasi Pengelolaan Kinerja Akademik Dosen
UNTUK DOSEN Materi Sosialisasi Pengelolaan Kinerja Akademik DosenUNTUK DOSEN Materi Sosialisasi Pengelolaan Kinerja Akademik Dosen
UNTUK DOSEN Materi Sosialisasi Pengelolaan Kinerja Akademik Dosen
AdrianAgoes9
 
Modul Ajar PAI dan Budi Pekerti Kelas 2 Fase A Kurikulum Merdeka
Modul Ajar PAI dan Budi Pekerti Kelas 2 Fase A Kurikulum MerdekaModul Ajar PAI dan Budi Pekerti Kelas 2 Fase A Kurikulum Merdeka
Modul Ajar PAI dan Budi Pekerti Kelas 2 Fase A Kurikulum Merdeka
Fathan Emran
 
Modul Ajar IPS Kelas 7 Fase D Kurikulum Merdeka
Modul Ajar IPS Kelas 7 Fase D Kurikulum MerdekaModul Ajar IPS Kelas 7 Fase D Kurikulum Merdeka
Modul Ajar IPS Kelas 7 Fase D Kurikulum Merdeka
Fathan Emran
 
INSTRUMEN PENILAIAN PRAKTIK KINERJA KS Dok Rating Observasi (1).docx
INSTRUMEN PENILAIAN PRAKTIK KINERJA KS Dok Rating Observasi (1).docxINSTRUMEN PENILAIAN PRAKTIK KINERJA KS Dok Rating Observasi (1).docx
INSTRUMEN PENILAIAN PRAKTIK KINERJA KS Dok Rating Observasi (1).docx
lindaagina84
 
Karier-Dan-Studi-Lanjut-Di-Bidang-Informatika.pptx
Karier-Dan-Studi-Lanjut-Di-Bidang-Informatika.pptxKarier-Dan-Studi-Lanjut-Di-Bidang-Informatika.pptx
Karier-Dan-Studi-Lanjut-Di-Bidang-Informatika.pptx
adolfnuhujanan101
 
Modul Ajar Bahasa Inggris Kelas 5 Fase C Kurikulum Merdeka
Modul Ajar Bahasa Inggris Kelas 5 Fase C Kurikulum MerdekaModul Ajar Bahasa Inggris Kelas 5 Fase C Kurikulum Merdeka
Modul Ajar Bahasa Inggris Kelas 5 Fase C Kurikulum Merdeka
Fathan Emran
 
SOAL SHB PKN SEMESTER GENAP TAHUN 2023-2024.docx
SOAL SHB PKN SEMESTER GENAP TAHUN 2023-2024.docxSOAL SHB PKN SEMESTER GENAP TAHUN 2023-2024.docx
SOAL SHB PKN SEMESTER GENAP TAHUN 2023-2024.docx
MuhammadBagusAprilia1
 

Recently uploaded (20)

RANCANGAN TINDAKAN UNTUK AKSI NYATA MODUL 1.4 BUDAYA POSITIF.pdf
RANCANGAN TINDAKAN UNTUK AKSI NYATA MODUL 1.4 BUDAYA POSITIF.pdfRANCANGAN TINDAKAN UNTUK AKSI NYATA MODUL 1.4 BUDAYA POSITIF.pdf
RANCANGAN TINDAKAN UNTUK AKSI NYATA MODUL 1.4 BUDAYA POSITIF.pdf
 
Bahan Sosialisasi PPDB_1 2024/2025 Bandung
Bahan Sosialisasi PPDB_1 2024/2025 BandungBahan Sosialisasi PPDB_1 2024/2025 Bandung
Bahan Sosialisasi PPDB_1 2024/2025 Bandung
 
NUMERASI KOMPETENSI PENDIDIK TAHAP CAKAP DAN MAHIR.pdf
NUMERASI KOMPETENSI PENDIDIK TAHAP CAKAP DAN MAHIR.pdfNUMERASI KOMPETENSI PENDIDIK TAHAP CAKAP DAN MAHIR.pdf
NUMERASI KOMPETENSI PENDIDIK TAHAP CAKAP DAN MAHIR.pdf
 
Tabel 1. 7 Ruang Lingkup Terintegrasi dalam Mata Pelajaran dalam CASEL PSE.pdf
Tabel 1. 7 Ruang Lingkup Terintegrasi dalam Mata Pelajaran dalam CASEL PSE.pdfTabel 1. 7 Ruang Lingkup Terintegrasi dalam Mata Pelajaran dalam CASEL PSE.pdf
Tabel 1. 7 Ruang Lingkup Terintegrasi dalam Mata Pelajaran dalam CASEL PSE.pdf
 
EVIDENCE BASED DALAM PELAYANAN KB DAN KONTRASEPSI.pdf
EVIDENCE BASED DALAM PELAYANAN KB DAN KONTRASEPSI.pdfEVIDENCE BASED DALAM PELAYANAN KB DAN KONTRASEPSI.pdf
EVIDENCE BASED DALAM PELAYANAN KB DAN KONTRASEPSI.pdf
 
2. Kerangka Kompetensi Literasi Guru SD_Rev.pptx
2. Kerangka Kompetensi Literasi Guru SD_Rev.pptx2. Kerangka Kompetensi Literasi Guru SD_Rev.pptx
2. Kerangka Kompetensi Literasi Guru SD_Rev.pptx
 
LK 1 - 5T Keputusan Berdampak PERMATA BUNDA.pdf
LK 1 - 5T Keputusan Berdampak PERMATA BUNDA.pdfLK 1 - 5T Keputusan Berdampak PERMATA BUNDA.pdf
LK 1 - 5T Keputusan Berdampak PERMATA BUNDA.pdf
 
Pi-2 AGUS MULYADI. S.Pd (3).pptx visi giru penggerak dan prakrsa perubahan bagja
Pi-2 AGUS MULYADI. S.Pd (3).pptx visi giru penggerak dan prakrsa perubahan bagjaPi-2 AGUS MULYADI. S.Pd (3).pptx visi giru penggerak dan prakrsa perubahan bagja
Pi-2 AGUS MULYADI. S.Pd (3).pptx visi giru penggerak dan prakrsa perubahan bagja
 
LAPORAN TUGAS TAMBAHAN PEMBINA PRAMUKA..
LAPORAN TUGAS TAMBAHAN PEMBINA PRAMUKA..LAPORAN TUGAS TAMBAHAN PEMBINA PRAMUKA..
LAPORAN TUGAS TAMBAHAN PEMBINA PRAMUKA..
 
VISI MISI KOMUNITAS BELAJAR SDN 93 KOTA JAMBI
VISI MISI KOMUNITAS BELAJAR SDN 93 KOTA JAMBIVISI MISI KOMUNITAS BELAJAR SDN 93 KOTA JAMBI
VISI MISI KOMUNITAS BELAJAR SDN 93 KOTA JAMBI
 
AKSI NYATA TAHAP PERKEMBANGAN PESERTA DIDIK JENJANG SD USIA 6-12 TAHUN.pptx
AKSI NYATA TAHAP PERKEMBANGAN PESERTA DIDIK JENJANG SD USIA 6-12 TAHUN.pptxAKSI NYATA TAHAP PERKEMBANGAN PESERTA DIDIK JENJANG SD USIA 6-12 TAHUN.pptx
AKSI NYATA TAHAP PERKEMBANGAN PESERTA DIDIK JENJANG SD USIA 6-12 TAHUN.pptx
 
Laporan Kegiatan Pramuka Tugas Tambahan PMM.pdf
Laporan Kegiatan Pramuka Tugas Tambahan PMM.pdfLaporan Kegiatan Pramuka Tugas Tambahan PMM.pdf
Laporan Kegiatan Pramuka Tugas Tambahan PMM.pdf
 
Paparan Kurikulum Satuan Pendidikan_LOKAKARYA TPK 2024.pptx.pdf
Paparan Kurikulum Satuan Pendidikan_LOKAKARYA TPK 2024.pptx.pdfPaparan Kurikulum Satuan Pendidikan_LOKAKARYA TPK 2024.pptx.pdf
Paparan Kurikulum Satuan Pendidikan_LOKAKARYA TPK 2024.pptx.pdf
 
UNTUK DOSEN Materi Sosialisasi Pengelolaan Kinerja Akademik Dosen
UNTUK DOSEN Materi Sosialisasi Pengelolaan Kinerja Akademik DosenUNTUK DOSEN Materi Sosialisasi Pengelolaan Kinerja Akademik Dosen
UNTUK DOSEN Materi Sosialisasi Pengelolaan Kinerja Akademik Dosen
 
Modul Ajar PAI dan Budi Pekerti Kelas 2 Fase A Kurikulum Merdeka
Modul Ajar PAI dan Budi Pekerti Kelas 2 Fase A Kurikulum MerdekaModul Ajar PAI dan Budi Pekerti Kelas 2 Fase A Kurikulum Merdeka
Modul Ajar PAI dan Budi Pekerti Kelas 2 Fase A Kurikulum Merdeka
 
Modul Ajar IPS Kelas 7 Fase D Kurikulum Merdeka
Modul Ajar IPS Kelas 7 Fase D Kurikulum MerdekaModul Ajar IPS Kelas 7 Fase D Kurikulum Merdeka
Modul Ajar IPS Kelas 7 Fase D Kurikulum Merdeka
 
INSTRUMEN PENILAIAN PRAKTIK KINERJA KS Dok Rating Observasi (1).docx
INSTRUMEN PENILAIAN PRAKTIK KINERJA KS Dok Rating Observasi (1).docxINSTRUMEN PENILAIAN PRAKTIK KINERJA KS Dok Rating Observasi (1).docx
INSTRUMEN PENILAIAN PRAKTIK KINERJA KS Dok Rating Observasi (1).docx
 
Karier-Dan-Studi-Lanjut-Di-Bidang-Informatika.pptx
Karier-Dan-Studi-Lanjut-Di-Bidang-Informatika.pptxKarier-Dan-Studi-Lanjut-Di-Bidang-Informatika.pptx
Karier-Dan-Studi-Lanjut-Di-Bidang-Informatika.pptx
 
Modul Ajar Bahasa Inggris Kelas 5 Fase C Kurikulum Merdeka
Modul Ajar Bahasa Inggris Kelas 5 Fase C Kurikulum MerdekaModul Ajar Bahasa Inggris Kelas 5 Fase C Kurikulum Merdeka
Modul Ajar Bahasa Inggris Kelas 5 Fase C Kurikulum Merdeka
 
SOAL SHB PKN SEMESTER GENAP TAHUN 2023-2024.docx
SOAL SHB PKN SEMESTER GENAP TAHUN 2023-2024.docxSOAL SHB PKN SEMESTER GENAP TAHUN 2023-2024.docx
SOAL SHB PKN SEMESTER GENAP TAHUN 2023-2024.docx
 

Arrini ditta margarani (2016066155)

  • 1. LAPORAN SIKLUS 1 PENDALAMAN MATERI KOMPETENSI KEAHLIAN ARITMATIKA DAN BILANGAN Disusun Oleh: Arrini Ditta Margarani NIM. SM-3T. 2016066155 PENDIDIKAN MATEMATIKA PENDIDIKAN PROFESI GURU SM-3T UNIVERSITAS PENDIDIKAN INDONESIA 2018
  • 2. ii DAFTAR ISI HALAMAN JUDUL................................................................................................ i DAFTAR ISI........................................................................................................... ii BILANGAN BULAT DAN PEMECAHAN MASALAH ..................................... 1 BILANGAN PECAHAN DAN PEMECAHAN MASALAH................................ 8 BARISAN DAN DERET...................................................................................... 14 Barisan dan Deret Aritmatika............................................................................ 15 Barisan dan Deret Geometri.............................................................................. 19 Barisan dan Deret Lain...................................................................................... 22 INDUKSI MATEMATIS ..................................................................................... 27 BILANGAN BERPANGKAT BULAT................................................................ 29 BILANGAN PANGKAT RASIONAL ................................................................ 32 SIFAT-SIFAT KETERBAGIAN.......................................................................... 35 ALGORITMA EUCLID ....................................................................................... 36 FPB dan KPK........................................................................................................ 38 PERSAMAAN DIOPHANTINE LINEAR .......................................................... 39 SIFAT-SIFAT DASAR KONGRUENSI.............................................................. 42 UJI PEMBAGIAN BILANGAN BULAT............................................................ 53 KONGRUENSI LINEAR..................................................................................... 56 LATIHAN SOAL ................................................................................................. 59 DAFTAR PUSTAKA ........................................................................................... 60
  • 3. 1 BILANGAN BULAT DAN PEMECAHAN MASALAH Definisi: Bilangan bulat adalah suatu istilah yang hanya ada dalam pikiran, namun akan memiliki arti jika terdapat banyaknya atau dikaitkan dengan kuantitas. Bilangan Bulat merupakan gabungan dari himpunan bilangan cacah dan himpunan bilangan asli. Fakta Bilangan Bulat  Himpunan semua bilangan bulat dalam matematika dilambangkan dengan Z berasal dari Zahlen (bahasa Jerman untuk "bilangan"). Sifat Bilangan Bulat 1. Sifat Tertutup  Sifat tertutup terhadap penjumlahan ada dengan tunggal yakni untuk setiap 𝑎, 𝑏 𝜖 𝑍 maka (a + b) juga di dalam Z.  Sifat tertutup terhadap perkalian ada dengan tunggal, yakni untuk setiap 𝑎, 𝑏 𝜖 𝑍 maka a x b juga ada di dalam Z. 2. Sifat Komutatif  Untuk setiap 𝑎, 𝑏 𝜖 𝑍 berlaku a + b = b + a.  Untuk setiap bilangan bulat 𝑎, 𝑏 𝜖 𝑍 berlaku a × b = b × a. 3. Sifat Asosiatif  Sifat asosiatif terhadap penjumlahan yaitu untuk sebarang bilangan bulat a, b, dan c berlaku sifat (a+b) + c= a + (b+c)  Sifat asosiatif terhadap perkalian yaitu untuk sebarang bilangan bulat a, b, dan c berlaku (a × b) × c = a × (b × c) 4. Sifat Distributif  Sifat distributif kiri perkalian terhadap penjumlahan, yaitu untuk sebarang bilangan bulat a, b, dan c berlaku sifat a × (b + c) = (a × b) + (a × c)  Sifat distributif kanan perkalian terhadap penjumlhan yaitu untuk sebarang bilangan bulat a, b, dan c berlaku sifat (a + b) × c = (a × c) + (b × c)
  • 4. 2 5. Unsur Identitas Penjumlahan Untuk setiap bilangan bulat a, selalu berlaku a + 0 = 0 + a = a sehingga 0 disebut unsur identitas penjumlahan. 6. Unsur identitas perkalian Untuk setiap bilangan bulat a, ada dengan tunggal bilangan bulat 1 sehingga a × 1 = 1 × a = 1 sehingga satu disebut unsur identitas perkalian. Teorema Operasi Bilangan Bulat 1. Teorema Penjumlahan Bilangan Bulat  Jika a, b, dan c anggota himpunan blangan bulat Z, dan a = b maka a + c = b + c. Bukti: Ambil a, b, dan c anggoata Z (a + c) Z (sifat tertutup) (a + c) = (a + c) (sifat refleksi) a = b (diberikan) (a + c) = (b + c) (substitusi, 3 ke 2)  Jika a, b, dan c anggota dari himpunan bilangan bulat Z, dan a + c = b + c maka a = b Bukti: Ambil a, b, dan c di Z (a + c) ∈ Z (sifat tertutup) a + c = b + c (a+c) + (-c) = b + (c + (-c)) (diberikan – c ∈ Z Invers tambahan) a + (c + (-c)) = b + (c + (-c)) c + (-c) = 0 a + 0 = b + 0 a + 0 = a dan b + 0 = b a = b Teorema diatas biasanya dikenal dengan sifat penghapusan dari penjumlahan.
  • 5. 3  Bukti bahwa (-a) + (-b) = - (a + b) Misalkan a dan b bilangan – bilangan cacah, maka (-a) + (-b) merupakan jumlah dua bilangan negatif. Misal (-a) + (-b) = c maka c = (-a) + (-b). c + b = ((-a) + (-b)) + b c + b = (-a) + ((-b) + b) c + b = (-a) + 0 (c + b) + a = (-a) + a (c + b) + a = 0 c + (b + a) = 0 c + (a + b) = 0 (c + (a + b)) + (-(a + b)) = – (a + b) c +((a + b) + (-(a + b))) = – (a + b) c + 0 = – (a + b) sifat kesamaan sifat asosiatif penjumlahan invers penjumlahan sifat kesamaan invers penjumlahan sifat asosiatif penjumlahan sifat komutatif penjumlahan sifat kesamaan sifat asosiatif invers penjumlahan Jadi kesimpulannya (-a) + (-b) = – (a + b) 2. Teorema Pengurangan Bilangan Bulat  a – (-b) = a + b untuk sebarang bilangan bulat a dan b Bukti: ambil bilangan bulat a dan b a – (-b) = a + (-(-b) =a+b Definisi pengurangan Teorema penjumlahan  a - b = (a - c) - (b - c) untuk sebarang bilangan bulat a, b, dan c. Bukti:ambil sebarang bilangan bulat a, b, dan c a – b = a + (-b) Definisi Pengurangan = ((a + (-b)) + 0 Identitas Tambahan = a + (-b) + c + (-c) Invers Tambahan = (a + (-c)) + ((-b) + c) Asosiatif Tambah = (a + (-c)) + ((-b) + (-(-c))) Teorema Dalam Penjumlahan = (a + (-c)) + (- (b + (-c))) Teorema Dalam Penjumlahan = (a - c) - (b + (-c)) Definisi pengurangan = (a - c) - (b - c) Definisi pengurangan
  • 6. 4 3. Teorema Perkalian Bilangan Bulat  Jika a, b, dan c anggota himpunan bilangan bulat Z dan a = b maka a × c = b × c Bukti: Ambil a, b, dan c di Z (a × c) 𝜖 Z sifat tertutup a × c = a × c sifat refleksi a = b diberikan a × c = b × c substitusi 3 ke 2  Jika a, b, dan c anggota himpunan bilangan bulat Z maka (a + b) × c = (a × c) + (b × c) Bukti: Ambil a, b, dan c di Z (a + b) × c 𝜖 Z (a + b) × c = c × (a + b) c × (a + b) = (c × a) + (c × b) (c × a) = (a × c) dan ((c × b) = (b × c) (a + b) × c = (a × c) + (b × c)  Jika a anggota bilangan bulat Z maka a × 0 = 0 dan 0 × a = 0 Bukti: Ambil a, b, dan c di Z. 1) a = a 2) 0 = 0 + 0 3) a x 0 = a x (0 + 0) 4) a x 0 = (a x 0) + (a x 0) 5) 0 + (a x 0) = (a x 0) 6) 0 + (a x 0) = (a x 0) + (a x 0) 7) 0 = (a x 0) 8) (a x 0) = 0 9) (0 x a) = 0  Jika a dan b anggota bilangan bulat Z maka a x (-b) = -(a x b). Bukti: a x (b + (-b)) = a x 0 (a x b) + (a x (-b)) = 0 (a x (-b)) + (a x b) = 0
  • 7. 5 ((a x (-b)) + (a x b)) + (-(a x b)) = 0 + (- (a x b)) (a x (-b)) + ((a x b) + (-a (a x b))) = - (a x b) a x (-b) + 0 = - (a x b) a x (-b) = - (a x b)  Buktikan bahwa (-a) (b + (-c)) = ac – ab. Bukti: (-a) (b + (-c)) = (-a)(b) + (-a)(-c) sifat distributif perkalian penjumlahan = (-(ab)) + ac perkalian bilangan bulat (-a) x b = -ab dan (-a) x (-c) = ac = ac + (-(ab)) sifat komutatif perkalian = ac – ab penjumlahan 2 bilangan bulat (misal: a + (-b) = a – b) Jadi terbukti bahwa (-a) (b + (-c)) = ac – ab. 4. Teorema Pembagian Bilangan Bulat  Mengingat bahwa (-a) x (b)= (a) x (-b) = -(ab) dan berdasarkan defnisi pembagian, kita dapat mengemukakan sifat berikut : -(ab) ÷ a = (-b) -(ab) ÷ b = (-a) -(ab) ÷ (-a) = b -(ab) ÷ (-b) = a Demikian pula karena (-a) × (-b) = a × b maka: ab ÷ (-a) = (-b) ab ÷ (-b) = (-a) Formula atau Rumus 1. Jika p dan q bilangan bulat maka berlaku:. a. p x q = pq b. (–p) x q = – (p x q) = –pq c. p x (–q) = – (p x q) = –pq d. (–p) x (–q) = p x q = pq 2. Untuk setiap p, q, dan r bilangan bulat berlaku sifat:
  • 8. 6 a. Tertutup pada operasi perkalian b. Komutatif: p x q = q x p c. Asosiatif: (p x q) x r = p x (q x r) d. Distributif perkalian terhadap penjumlahan: p x (q + r) = (p x q) + (p x r) e. Distributif perkalian terhadap pengurangan: p x (q – r) = (p x q) – (p x r). 3. Unsur identitas pada perkalian adalah 1, sehingga untuk setiap bilangan bulat p berlaku p x 1 = 1 x p = p. 4. Pembagian merupakan operasi kebalikan dari perkalian. 5. Pada operasi pembagian bilangan bulat tidak bersifat tertutup. Materi Pengayaan 1. Sederhanakan pecahan 200 100 . Untuk menjawab pertanyaan tersebut, sebaiknya tidak mengarahkan siswa untuk mencoret dua buah angka nol yang terdapat pada kedua bilangan, meskipun hasilnya benar, karena hal tersebut akan menimbulkan miskonsepsi pada siswa yang kurang memahaminya. Contoh: 101 301 , 16 64 , 26 65 , dst. Meskipun beberapa bilangan bernilai benar saat dicoret angka yang sama, hal ini tidak berlaku untuk semua bilangan. 2. T: Apakah semua bilangan bulat tertutup terhadap operasi penjumlahan? J: Ya, karena jika dua buah bilangan bulat dijumlahkan, maka hasilnya adalah bilangan bulat. Begitu pun juga pengurangan dan perkalian bilangan bulat. 3. T: Apakah semua bilangan bulat tertutup terhadap operasi pembagian? J: Tidak, hal ini tidak berlaku pada 2 3 , dimana 2 dan 3 merupakan anggota bilangan bulat, sedangkan hasil baginya bukan merupakan anggota bilangan bulat. 4. Untuk menjelaskan perkalian sebagai penjumlahan berulang, cukup pada usia SD kelas I-III.
  • 9. 7 Diskusi Kelompok 1. Apakah 3 x 2 = 2 + 2 + 2 atau 3 x 2 = 3 + 3? Pilih salah satu konsep, karena keduanya belum tentu benar atau dapat digunakan. Misalnya kita memilih 3 x 2 = 2 + 2 + 2 maka apabila ada permasalahan -3 x 2 tidak dapat diselesaikan dengan cara yang sama dengan 3 x 2 = 2 + 2 + 2 tetapi dapat diselesaikan dengan cara 3 x 2 = 3 + 3 yaitu -3 x 2 = -3 x -3. Jadi kedua cara tersebut benar bergantung dengan permasalahannya.
  • 10. 8 BILANGAN PECAHAN DAN PEMECAHAN MASALAH Definisi: Bilangan pecahan adalah bilangan yang dinyatakan dalam bentuk b a , dengan a,b B dan b bukan faktor dari a dan 𝑏 ≠ 0. a disebut pembilang dan b disebut penyebut. Konsep Jika a habis dibagi b maka bilangan itu adalah bilangan bulat (pecahan palsu), jika tidak maka berupa pecahan. Jenis-jenis Pecahan Ditinjau dari perbandingan besar nilai pembilang dan penyebut, pecahan dibedakan menjadi dua (2) yaitu: a. Pecahan Sejati (Pecahan Murni) Pecahan sejati adalah pecahan yang nilai positif pembilang lebih kecil dari nilai positif penyebut. Contoh 2/3, 5/7, dan 9/10 adalah contoh-contoh bilangan pecahan sejati b. Pecahan Tidak Sejati (Pecahan Campuran) Pecahan tidak sejati adalah pecahan yang nilai positif pembilang lebih besar dari nilai positif penyebut. Contoh 10/7, 12/9, dan 2 1/4 adalah contoh-contoh bilangan pecahan tak sejati. Pecahan tak sejati 10/7 dapat ditulis dalam bentuk 1 3/7 , yang berarti 10/7 = 1 3/7. Pecahan dalam bentuk 1 3/7 disebut pecahan campuran. Jadi pecahan campuran adalah pecahan yang penulisannya merupakan gabungan dari bilangan bulat dan pecahan sejati. Ditinjau dari nilai pembilang atau penyebutnya, dan hubungan antara pembilang dan penyebut, pecahan dibedakan menjadi: a. Pecahan Sederhana Pecahan sederhana adalah pecahan yang FPB (Faktor Persekutuan Terbesar) dari pembilang dan penyebutnya adalah 1. Contoh 5/7, 2/3, dan 5/3 adalah contoh-contoh pecahan sederhana karena FPB dari pembilang dan penyebutnya adalah 1.
  • 11. 9 b. Pecahan Senama Pecahan senama adalah pecahan yang penyebutnya sama. Contoh 2/4, 3/4, dan 1/4 adalah contoh-contoh pecahan senama karena penyebutnya sama. c. Pecahan Desimal Pecahan desimal adalah pecahan yang penyebutnya berbentuk 10n atau jumlahan dari pecahan-pecahan yang penyebutnya berbentuk 10n dengan n bilangan asli. Contoh 1/10, 1/100, 1/1.000, 2/100, dan 0,03 adalah contoh- contoh pecahan desimal. Pecahan Satuan Pecahan satuan adalah pecahan yang memiliki pembilang 1. Contoh: 1 3 , 1 2 , 1 7 ,dst. Contoh: Nyatakan 3 7 sebagai penjumlahan pecahan satuan. Penyelesaian: 7 3 ≈ 3 => 3 7 − 1 3 = 9 − 7 21 = 2 21 21 2 ≈ 11 => 2 21 − 1 11 = 22 − 21 231 = 1 231 ∴ 3 7 = 1 3 + 1 11 + 1 231 Pecahan Kontinu Pecahan kontinu ditulis sebagai: 𝑥 = 𝑎1 + 1 𝑎2 + 1 𝑎3 + 1 𝑎4 + … 𝑎 𝑛 Juga dapat direpresentasikan seperti sebuah selang [𝑎1, 𝑎2, 𝑎3, … , 𝑎 𝑛] Contoh: Tentukan nilai dari [7,4,2,5] Penyelesaian: [7,4,2,5] = 7 + 1 4+ 1 2+ 1 5 = 7 + 1 4+ 1 11 5 = 7 + 1 4+ 5 11 = 7 + 1 49 11 = 7 + 11 49 = 354 49
  • 12. 10 Selanjutnya, untuk menyatakan suatu pecahan menjadi pecahan kontinu dapat menggunakan algoritma Euclides seperti contoh berikut. T : Nyatakan 73 46 sebagai pecahan kontinu. 73 = 46(1) + 27 46 = 27(1) + 19 27 = 19(1) + 8 19 = 8(2) + 3 8 = 3(2) + 2 3 = 2(1) + 1 2 = 1(2) Teorema 1: Jika 𝑎/𝑐 dan 𝑏/𝑐 pecahan-pecahan dengan c ≠ 0, maka 𝑎 𝑐 + 𝑏 𝑐 = 𝑎+𝑏 𝑐 . Teorema 2: Jika a/b dan c/d pecahan-pecahan dengan c ≠ 0 maka 𝑎 𝑐 − 𝑏 𝑐 = 𝑎−𝑏 𝑐 . Teorema 3 (Relasi Urutan): Diketahui a/c dan b/c adalah pecahan-pecahan dengan c > 0. Pecahan a/c dikatakan kurang dari b/c, yaitu a/c < b/c jika dan hanya jika a < b. Teorema 4 Diketahui a/b dan c/d pecahan-pecahan dengan b > 0 dan d > 0 𝑎 𝑏 < 𝑐 𝑑 ⇔ 𝑎𝑑 < 𝑏𝑐 Fakta: Bilangan pecahan b a dibaca a dibagi b Sifat pada Bilangan Pecahan 1. Penjumlahan pecahan memiliki sifat-sifat berikut: a. Komutatif b a d c d c b a  b. Asosiatif              f e d c b a f e d c b a Dari Algoritma di samping, bilangan 73 46 dapat ditulis sebagai pecahan kontinu [1,1,1,2,2,1,2]
  • 13. 11 2. Perkalian dan Pembagian Pecahan a. Hasil perkalian dua pecahan diperoleh dengan mengalikan pembilang dengan pembilang dan penyebut dengan penyebut. bd ac db ca d c b a     b. Untuk membagi suatu pecahan dengan pecahan lain sama artinya dengan mengalikan pecahan pertama dengan kebalikan pecahan kedua bc ad cb da c d b a d c b a    : Materi pengayaan 1. Nyatakan 13 14 sebagai jumlah pecahan unit yang berbeda. 2. Buktikan 8 11 tidak dapat dinyatakan ke dalam penjumlahan unit yang berbeda yang banyaknya 4 buah pecahan unit. 3. Tuliskan bilangan rasional yang dinyatakan dengan [1,2,3,4,5,6]. 4. Nyatakan dalam bentuk pecahan kontinu 177 233 . Penyelesaian: 1. Dengan menggunakan pengurangan invers. 14 13 ≈ 2 => 13 14 − 1 2 = 13 − 7 14 = 6 14 14 6 ≈ 3 => 6 14 − 1 3 = 18 − 14 42 = 2 21 21 2 ≈ 11 => 2 21 − 1 11 = 22 − 21 231 = 1 231 ∴ 13 14 = 1 2 + 1 3 + 1 11 + 1 231 2. Akan dibuktikan, 8 11 tidak dapat dinyatakan ke dalam pecahan unit yang banyaknya 4 buah. 11 8 ≈ 2 => 8 11 − 1 2 = 16 − 11 22 = 5 22 22 5 ≈ 5 => 5 22 − 1 5 = 25 − 22 110 = 3 110
  • 14. 12 110 3 ≈ 37 => 3 110 − 1 37 = 111 − 110 4070 = 1 4070 ∴ 8 11 = 1 2 + 1 5 + 1 37 + 1 4070 8 11 dapat dinyatakan ke dalam 4 buah pecahan satuan (tidak terbukti) 3. Bukti: 𝑝 + 𝑞 = 4𝑒𝑓 … … … … . . (𝑖) 𝑝 + 𝑒 = 𝑔𝑞 ↔ 𝑞 = 𝑝 + 𝑒 𝑔 … … … … … . (𝑖𝑖) Substitusi (ii) ke (i) ↔ 𝑝 + 𝑝 + 𝑒 𝑔 = 4𝑒𝑓 … … … … … . . (𝑘𝑒𝑑𝑢𝑎 𝑟𝑢𝑎𝑠 𝑑𝑖𝑘𝑎𝑙𝑖 𝑔) ↔ 𝑝𝑔 + 𝑝 + 𝑒 = 4𝑒𝑓𝑔 … … … … … (𝑘𝑒𝑑𝑢𝑎 𝑟𝑢𝑎𝑠 𝑑𝑖𝑘𝑎𝑙𝑖 1 𝑒𝑓𝑔 ) ↔ 𝑝𝑔 𝑒𝑓𝑔 + 𝑝 𝑒𝑓𝑔 + 𝑒 𝑒𝑓𝑔 = 4 … … … … . (𝑘𝑒𝑑𝑢𝑎 𝑟𝑢𝑎𝑠 𝑑𝑖𝑘𝑎𝑙𝑖 1 𝑝 ) ↔ 𝑝𝑔 𝑒𝑓𝑔𝑝 + 𝑝 𝑒𝑓𝑔𝑝 + 𝑒 𝑒𝑓𝑔𝑝 = 4 𝑝 ↔ 1 𝑒𝑓 + 1 𝑒𝑓𝑔 + 1 𝑓𝑔𝑝 = 1 𝑝 ∴ Terbukti. 4. Gunakan algoritma euclides. [1,2,3,4,5,6] ↔ 5 + 1 6 = 31 6 ↔ 4 + 6 31 = 130 31 ↔ 3 + 31 130 = 421 130 ↔ 2 + 130 421 = 972 421 ↔ 1 + 421 972 = 1393 972 ∴ [1,2,3,4,5,6] = 1393 972
  • 15. 13 5. Gunakan algoritma euclides. 177 = 233(0) + 177 233 = 177(1) + 56 177 = 56(3) + 9 56 = 9(6) + 2 9 = 2(4) + 1 2 = 1(2) ∴ 177 233 = [0,1,3,6,4,2] Diskusi Kelompok Tunjukan bila 𝑝 + 𝑞 = 4𝑒𝑓; 𝑝 + 𝑒 = 𝑔𝑞, maka 4 𝑝 = 1 𝑒𝑓 + 1 𝑒𝑓𝑔 + 1 𝑓𝑔𝑝 . Bukti: 𝑝 + 𝑞 = 4𝑒𝑓 … … … … . . (𝑖) 𝑝 + 𝑒 = 𝑔𝑞 ↔ 𝑞 = 𝑝 + 𝑒 𝑔 … … … … … . (𝑖𝑖) Substitusi (ii) ke (i) ↔ 𝑝 + 𝑝 + 𝑒 𝑔 = 4𝑒𝑓 … … … … … . . (𝑘𝑒𝑑𝑢𝑎 𝑟𝑢𝑎𝑠 𝑑𝑖𝑘𝑎𝑙𝑖 𝑔) ↔ 𝑝𝑔 + 𝑝 + 𝑒 = 4𝑒𝑓𝑔 … … … … … (𝑘𝑒𝑑𝑢𝑎 𝑟𝑢𝑎𝑠 𝑑𝑖𝑘𝑎𝑙𝑖 1 𝑒𝑓𝑔 ) ↔ 𝑝𝑔 𝑒𝑓𝑔 + 𝑝 𝑒𝑓𝑔 + 𝑒 𝑒𝑓𝑔 = 4 … … … … . (𝑘𝑒𝑑𝑢𝑎 𝑟𝑢𝑎𝑠 𝑑𝑖𝑘𝑎𝑙𝑖 1 𝑝 ) ↔ 𝑝𝑔 𝑒𝑓𝑔𝑝 + 𝑝 𝑒𝑓𝑔𝑝 + 𝑒 𝑒𝑓𝑔𝑝 = 4 𝑝 ↔ 1 𝑒𝑓 + 1 𝑒𝑓𝑔 + 1 𝑓𝑔𝑝 = 1 𝑝 ∴ Terbukti.
  • 16. 14 BARISAN DAN DERET Barisan Bilangan Definisi Barisan Suatu fungsi berharga real yang didefinisikan pada himpunan bilangan bulat postif disebut suatu barisan. Lazimnya barisan diberi simbol dengan (𝑎 𝑛), (𝑏 𝑛), (𝑈 𝑛) dan sebagainya. Selanjutnya: 𝑈1, 𝑈2, 𝑈3, … , 𝑈 𝑛, … menyatakan barisan tak hingga atau dengan singkat barisan dan 𝑈1 adalah suku pertama, 𝑈2 adalah suku ke-2, dan 𝑈 𝑛 adalah suku ke-𝑛 dari barisan (𝑈 𝑛). Bentuk Umum: Sifat Barisan Suatu barisan (𝑈 𝑛) dikatakan: 1) Naik, jika dan hanya jika 𝑈 𝑛 < 𝑈 𝑛+1, ∀𝑛 > 0 2) Tidak turun, jika dan hanya jika 𝑈 𝑛 ≤ 𝑈 𝑛+1, ∀𝑛 > 0 3) Turun jika dan hanya jika 𝑈 𝑛 > 𝑈 𝑛+1, ∀𝑛 > 0 4) Tidak naik jika dan hanya jika 𝑈 𝑛 ≥ 𝑈 𝑛+1, ∀𝑛 > 0 Deret Bilangan Definisi: Jumlah suku-suku dari suatu barisan di sebut deret. Bentuk umumnya adalah sebagai berikut. Sifat-sifat Deret 1) Jika setiap suku deret dikalikan dengan konstana yang tidak sama dengan 0 maka kekonvergenan (atau kedivergenan) deret tidak berubah. 𝑈1 + 𝑈2 + 𝑈3 + ⋯ + 𝑈 𝑛 = ∑ 𝑈𝑖 𝑛 𝑖=1 𝑈 𝑛 = 𝑈1, 𝑈2, 𝑈3, … 𝑈1, 𝑈2, 𝑈3,
  • 17. 15 2) Penghapusan (atau penambahan) sejumlah berhingga suku-suku daro (atau terhadap) suatu deret tidak mengubaj kekonvergenan atau kedivergenan deret. Barisan dan Deret Aritmatika Barisan Aritmatika Definisi: Barisan U1, U2, U3, ..., Un, ... disebut barisan aritmatika jika Un - Un-1 = konstan, dengan n= 2, 3, 4, .... Konstanta pada barisan aritmatika di atas disebut beda dari barisan itu dan sering dinotasikan dengan b, dan U1 sering dinotasikan dengan a. Rumus Jika U1 = a, U2, U3, ..., Un,... merupakan barisan aritmatika, maka unsur ke n dari barisan itu dapat diturunkan dengan cara berikut. U1 = a U2= a + b U3= U2 + b = (a + b) + b = a + 2b U4= U3 + b = (a + 2b) + b = a + 3b Un = a + (n - 1) b Jadi rumus umum unsur ke n suatu barisan aritmatika dengan unsur pertama a dan beda b adalah: Un = a + (n -1)b Bentuk barisan aritmatika Barisan aritmatika dapat ditulis menjadi dua bentuk, yaitu : 1) Eksplisit (𝑈 𝑛+1 = 1 2 𝑈 𝑛) 2) Rekrusif Rekrusif adalah suku-suku berikutnya ditentukan oleh suku-suku sebelumnya. Contoh : 𝑈 𝑛+1 = 𝑈 𝑛+1+𝑈 𝑛 2
  • 18. 16 Sifat-Sifat Barisan Aritmatika Sifat pertama: Apabila x, y, dan z merupakan bilangan yang berurutan dari suatu barisan aritmatika, maka akan berlaku: "Dua kali bilangan yang ditengah sama dengan jumlah dari kedua bilangan yang ada di sampingnya" (2y = x + z) Sifat kedua: Apabila w, x, y, z, empat bilangan yang berurutan dari suatu barisan aritmetika, maka akan berlaku: "Jumlah dari dua bilangan yang terletak di tengah sama dengan jumlah dari dua bilangan yang ada di sampingnya" (x + y = w + z) Sifat Ketiga: Apabila Un adalah suku ke-n barisan aritmetika maka berlaku: "Selisih antara jumlah n suku pertama dan jumlah (n-1) suku pertama adalah suku ke-n Un = Sn – Sn-1. Barisan Aritmatika Bertingkat Pada barisan ini tinggal melihat posisi suku sesudahnya 1) Tingkat pertama adalah barisan aritmetika itu sendiri. 2) Tingkat kedua (Kuadrat), jika pada pola selisih tingkat pertama membentuk lagi selisih tetap seolah- olah sebagai selisih kedua. Jika 𝑈 𝑛 = 𝑎𝑛2 + 𝑏𝑛 + 𝑐, maka a) 𝑎 = 𝑈3+𝑈1−2𝑈2 2 = 𝑠𝑒𝑙𝑖𝑠𝑖ℎ 𝑝𝑎𝑑𝑎 𝑡𝑖𝑛𝑔𝑘𝑎𝑡 𝑘𝑒𝑑𝑢𝑎 2 b) 𝑎 + 𝑏 + 𝑐 = 𝑈1 c) 3𝑎 + 𝑏 = 𝑈2 − 𝑈1 d) 5𝑎 + 𝑏 = 𝑈3 − 𝑈2 e) 2𝑎 = 𝑈1 + 𝑈3 − 2𝑈2 3) Tingkat ketiga Jika pada pola selisih tingkat kedua membentuk lagi selisih tetap seolah-olah sebagai selisih ketiga Jika 𝑈 𝑛 = 𝑎𝑛3 + 𝑏𝑛2 + 𝑐𝑛 + 𝑑, dengan a) 𝑈 𝑛 = rumus suku ke-n
  • 19. 17 b) 𝑎 = 𝑠𝑒𝑙𝑖𝑠𝑖ℎ 𝑡𝑒𝑟𝑎𝑘ℎ𝑖𝑟 𝑝𝑎𝑑𝑎 𝑡𝑖𝑛𝑔𝑘𝑎𝑡 𝑎𝑘ℎ𝑖𝑟 6 c) 𝑎 + 𝑏 + 𝑐 + 𝑑 = 𝑈1 = 𝑠𝑢𝑘𝑢 𝑝𝑒𝑟𝑡𝑎𝑚𝑎 d) 8𝑎 + 4𝑏 + 2𝑐 + 𝑑 = 𝑆2 e) 27𝑎 + 9𝑏 + 3𝑐 + 𝑑 = 𝑆3 4) Tingkat ke-n Misalkan 𝑈1 = 𝐴 = suku pertama. B, C, D, dan seterusnya selanjutnya adalah beda tiap tingkatan. Perhatikan bagan berikut: A A+B A+2B+C A+3B+3C+D A+4B+6C+4D+E B B+C B+2C+D B+3C+3D+E C C+D C+2D+E D D+…… Maka untuk 𝑈1 = 𝐴 𝑈2 = 𝐴 + 𝐵 𝑈3 = 𝐴 + 2𝐵 + 𝐶 𝑈4 = 𝐴 + 3𝐵 + 3𝐶 + 𝐷 …….. 𝑈 𝑛 = 𝐴 + (𝑛 − 1) 1! 𝐵 + (𝑛 − 1)(𝑛 − 2) 2! 𝐶 + (𝑛 − 1)(𝑛 − 2)(𝑛 − 3) 3! 𝐷 + ⋯ … Pola koefisien A,B, C, D dan seterusnya pada 𝑈 𝑛 mengikuti pola pada binom Newton. Contoh soal 1. Tentukan suku pertama, beda, suku kesepuluh dari barisan berikut ini: a. 1, 4, 7, 10, …. b. -2, 0, 2, 4, … Jawab:
  • 20. 18 a. 1, 4, 7, 10, …. a = 1, b = 3, U10 = 1 + (10 – 1).3 = 1 + 27 = 28 b. -2, 0, 2, 4, … A = -2, b = 2, U10 = -2 + (10 – 1)2 = -2 + 18 = 16 2. Tentukan formula barisan berikut. a. 0, 2, 6, 12, 20, ... Jawab: 0 2 6 12 20 2 4 6 8 2 2 2 𝑈 𝑛 = 𝑎 + (𝑛 − 1)𝑏 + (𝑛 − 1)(𝑛 − 2)𝑐 2 𝑈 𝑛 = 0 + (𝑛 − 1)2 + (𝑛 − 1)(𝑛 − 2)2 2 𝑈 𝑛 = 2𝑛 − 2 + 𝑛2 − 3𝑛 + 2 𝑈 𝑛 = 𝑛2 − 𝑛 Jadi formula barisan tersebut adalah 𝑈 𝑛 = 𝑛2 − 𝑛 Deret Aritmatika Definisi: Jika U1, U2, U3, ..., Un, .... merupakan barisan aritmatka, maka U1 + U2 + U3 + ... + Un, ... disebut deret aritmatika. Un disebut suku ke n dari deret itu. Rumus Jika Sn menyatakan jumlah n suku pertama deret aritmatika U1 + U2 + U3 + ... + Un, ...., maka Sn = U1 + U2 + U3 + ... + Un dapat diturunkan dengan cara sebagai berikut. Sn = Un + (Un - b) + (Un - 2b) + ... + a Sn = a + (a - b) + (a + 2b) +..... + Un + 2Sn = (a + Un) + (a + Un) + (a + Un) +... + (a + Un), sebanyak n suku. 𝑎 = 0, 𝑏 = 2, 𝑐 = 2
  • 21. 19 2Sn = n. (a + Un) Sn = )( 2 1 nUan  Jadi Sn = )( 2 1 nUan  atau Sn = ))1(2( 2 1 bnan  Contoh soal Diketahui deret aritmatika: 1 + 3 + 5 + 7 + 9 + …. Tentukan: a. Rumus suku ke –n b. Rumus jumlah n suku pertama c. Jumlah 50 suku pertama Jawab: a. 1 + 3 + 5 + 7 + 9 + …. Diperoleh a = 1, b = 2 Un = a + (n – 1)b Un = 1 + ( n – 1)2 Un = 1 + 2n – 2 Un = 2n – 1 b. Sn =  bna n )1(2 2  Sn =  2)1(1.2 2  n n =  222 2  n n =  n n 2 2 = n2 c. Sn = n2  S50 = (50)2 = 2500 Barisan dan Deret Geometri Definisi: Barisan geometri ialah suatu barisan bilangan-bilangan dimana rasio di antara dua suku berurutan merupakan bilangan tetap. Secara umum dapat dikatakan bahwa barisan: U1, U2, U3, U4, … , Un disebut barisan geometri jika 32 4 1 2 3 1 ... n n U UU U U U U U      = konstanta.
  • 22. 20 Rumus Rumus umum suku ke – n barisan geometri dengan suku pertama a dan rasio r dapat ditemukan seperti berikut : U1 = a U2 = ar U3 = ar2 U4 = ar3 Un = arn-1 Contoh soal Suku ke-n dari barisan geometri adalah Un = 2n . Tentukan : a. Jumlah 8 suku pertama b. Rumus jumlah n suku pertama Jawab: a. Un = 2n  U1 = 21 = 2 U2 = 22 = 4 U3 = 23 = 8 2 + 4 + 8 + 16 + … a = 2, r = 2, n = 8 )1( )1(    r ra S n n  )12( )12(2 8 8   S  )12( )1256(2 8   S  S8 = 2.255 = 510 Deret Geometri Definisi: Jika diketahui U1, U2, U3, . . . , Un merupakan suku-suku dari barisan geometri, maka U1 + U2 + U3 + . . . + Un disebut deret geometri , dengan Un = a rn-1 . Rumus Jika Sn merupakan jumlah n suku pertama dari suatu deret geometri, maka rumus umum untuk Sn dapat ditentukan dengan langkah-langkah sebagai berikut : Dimana : a adalah suku pertama / nilai awal r adalah rasio
  • 23. 21 Sn = U1 + U2 + U3 + . . . + Un maka Sn = a + ar + ar2 + . . . + arn-1 Kalikan Sn dengan r rSn = ar + ar2 + ar3 + . . . + arn-1 + arn Kurangkan rSn dengan Sn Sn = U1 + U2 + U3 + . . . + Un rSn = ar + ar2 + ar3 + . . . + arn-1 + arn Sn - rSn = a - arn Sn (1 – r) = a (1 - rn ) Sn = a n (1 - r ) 1 - r Jadi rumus umum jumlah n suku pertama deret geometri adalah : Sn = a n (1 - r ) 1 - r untuk r < 1 Sn = a n (r -1 ) r-1 untuk r > 1 Deret Tak Hingga Jika suku-suku deret geometri bertambah terus mendekati tak hingga, maka: 𝑆 = lim 𝑛→∞ 𝑆𝑛 = lim 𝑛→∞ 𝑎(1 − 𝑟 𝑛 ) 1 − 𝑟 = lim 𝑛→∞ 𝑎 1 − 𝑟 − lim 𝑛→∞ 𝑎 1 − 𝑟 𝑟 𝑛 = 𝑎 1 − 𝑟 − 𝑎 1 − 𝑟 lim 𝑛→∞ 𝑟 𝑛 karena jumlah deret geometri tak hingga adalah 𝑆 = 𝑎 1−𝑟 − 𝑎 1−𝑟 lim 𝑛→∞ 𝑟 𝑛 dan nilai lim 𝑛→∞ 𝑆𝑛 tergantung nilai lim 𝑛→∞ 𝑟 𝑛 maka aka nada 2 kemungkinan harga S, yaitu: i. Jika |𝑟| < 1 atau (−1 < 𝑟 < 1) maka nilai lim 𝑛→∞ 𝑟 𝑛 = 0. Sehingga 𝑆 = lim 𝑛→∞ 𝑆𝑛 = 𝑎 1−𝑟 yang selanjutnya deret ini kita sebut sebagai deret geometri tak hingga yang konvergen. ii. Jika |𝑟| ≥ 1 atau (𝑟 ≤ −1 atau 𝑟 ≥ 1), misalkan untuk nilai (𝑟 < −1 𝑎𝑡𝑎𝑢
  • 24. 22 𝑟 > 1) maka nilai lim 𝑛→∞ 𝑟 𝑛 = ±∞. Sehingga 𝑆 = lim 𝑛→∞ 𝑆𝑛 = 𝑎 1−𝑟 − 𝑎 1−𝑟 (±∞) = ±∞, maka deret geomteri tak hingga seperti ini tidak memiliki limit jumlah atau divergen. Barisan dan Deret Lain Berikut diberikan deret lain selain deret aritmatika dan deret geometri, yaitu deret teleskopih. Deret Teleskopik adalah deret yang suku – sukunya dapat saling menghilangkan sehingga hanya ditentukan oleh suku pertama dan terakhir. Adapun prinsip dari deret teleskopik: 1) nn nnk n k k aa aaaaaaaaaa       1 1342312 1 1 )(...)()()()( 2) ∏ 𝑎 𝑖+1 𝑎𝑖 𝑛 𝑖=1 = 𝑎2 𝑎1 𝑎3 𝑎2 𝑎4 𝑎3 … 𝑎 𝑛+1 𝑎 𝑛 = 𝑎 𝑛+1 𝑎1 3) )1( 11 )1( 1    xxxx 4) 1)1( 1 ... 4.3 1 3.2 1 2.1 1     n n nn 5) 12)12)(12( 1 ... 7.5 1 5.3 1 3.1 1     n n nn 6) 13)13)(23( 1 ... 10.7 1 7.4 1 4.1 1     n n nn 7) 14)14)(34( 1 ... 13.9 1 9.5 1 5.1 1     n n nn 8) 1 2.5 + 1 5.8 + 1 8.11 + ⋯ + 1 (3𝑛−11)(3𝑛+2) += 𝑛 6𝑛+4 9) 1 𝑥.(𝑥+2) = 1 2 ( 1 𝑥 − 1 𝑥+2 ) 10) 1 𝑥(𝑥+1)(𝑥+2) = 1 2 ( 1 𝑥(𝑥+1) − 1 (𝑥+1)(𝑥+2) ) 11) 1 1.2.3 + 1 2.3.4 + 1 3.4.5 + ⋯ + 1 𝑛.(𝑛+1)(𝑛+2) += 𝑛(𝑛+3) 4(𝑛+1)(𝑛+2) 12) 1 + 1 1+2 + 1 1+2+3 + 1 1+2+3+4 + ⋯ + 1 1+2+3+⋯+𝑛 += 2𝑛 𝑛+1
  • 25. 23 Materi Pengayaan 1. Jika 𝑋 𝑘+1 = 𝑋 𝑘 + 1 2 , 𝑘 = 1,2,3, …. Tentukan ∑ 𝑋𝑖 400 𝑖=1 dengan 𝑋1 = 1! Jawaban: 𝑋 𝑘+1 = 𝑋 𝑘 + 1 2 Untuk k = 1 maka 𝑋2 = 3 2 Untuk k = 2 maka 𝑋3 = 2 Untuk k = 3 maka 𝑋4 = 5 2 ∑ 𝑋𝑖 = 1 + 3 2 + 2 + 5 2 + ⋯ + 𝑋400 400 𝑖=1 𝑎 = 1, 𝑏 = 1 2 𝑆 𝑛 = 𝑛 2 (𝑎 + 𝑈 𝑛) 𝑆400 = 400 2 (1 + 399 × 1 2 ) = 40300 2. Asep memilih suku-suku barisan geometri tak hingga 1,1/2,1/4,1/8, ... untuk membuat barisan geometri tak hingga baru yang jumlahnya 1/7 , tiga suku pilihan asep adalah ? Jawab: Misal: 𝑈1 = ( 1 2 ) 𝑎 dan 𝑟 = ( 1 2 ) 𝑏 , 𝑎, 𝑏 ∈ 𝐵𝑖𝑙. 𝐴𝑠𝑙𝑖 maka, 𝑆∞ = 𝑎 1 − 𝑟 1 7 = ( 1 2 ) 𝑎 1−( 1 2 ) 𝑏 gunakan Aproksimasi 1 − ( 1 2 ) 𝑏 maka 1 ≤ 1 − ( 1 2 ) 𝑏 < 1 akibat 1 < 1 1−( 1 2 ) 𝑏 ≤ 2 kalikan dengan ( 1 2 ) 𝑎 didapat, ( 1 2 ) 𝑎 < 1 1 − ( 1 2 ) 𝑏 ≤ 2 ( 1 2 ) 𝑎
  • 26. 24 ( 1 2 ) 𝑎 ≤ 1 7 < ( 1 2 ) 𝑎−1 ambil sembarang nilai a anggota bilangan asli. Nilai 𝑎 yang memenuhi adalah 𝑎 = 3. Subtitusikan 𝑎 = 3 pada rumus jumlah tak hingga di atas. 1 7 = ( 1 2 ) 3 1−( 1 2 ) 𝑏 didapat nilai 𝑏 = 3. 𝑈1 = ( 1 2 ) 𝑎 = ( 1 2 ) 3 = 1 8 dan 𝑟 = ( 1 2 ) 𝑏 = ( 1 2 ) 3 = 1 8 𝑈2 = 1 64 𝑈3 = 1 512 Jadi tiga suku pilihan Asep yaitu 1 8 , 1 64 , dan 1 512 3. Tentukan formula barisan berikut. a. 0, 2, 6, 12, 20, ... Jawab: 0 2 6 12 20 2 4 6 8 2 2 2 𝑈 𝑛 = 𝑎 + (𝑛 − 1)𝑏 + (𝑛 − 1)(𝑛 − 2)𝑐 2 𝑈 𝑛 = 0 + (𝑛 − 1)2 + (𝑛 − 1)(𝑛 − 2)2 2 𝑈 𝑛 = 2𝑛 − 2 + 𝑛2 − 3𝑛 + 2 𝑈 𝑛 = 𝑛2 − 𝑛 Jadi formula barisan tersebut adalah 𝑈 𝑛 = 𝑛2 − 𝑛 b. 1, 5, 19, 49, 101, .... Jawab: 1 5 19 49 101 4 14 30 52 10 16 22 6 6 𝑎 = 0, 𝑏 = 2, 𝑐 = 2 𝑎 = 1, 𝑏 = 4, 𝑐 = 10, 𝑑 = 6
  • 27. 25 𝑈 𝑛 = 𝑎 + (𝑛 − 1)𝑏 + (𝑛 − 1)(𝑛 − 2)𝑐 2 + (𝑛 − 1)(𝑛 − 2)(𝑛 − 3)𝑑 6 𝑈 𝑛 = 1 + (𝑛 − 1)4 + (𝑛 − 1)(𝑛 − 2)10 2 + (𝑛 − 1)(𝑛 − 2)(𝑛 − 3)6 6 𝑈 𝑛 = 1 + (4𝑛 − 4) + 5(𝑛2 − 3𝑛 + 2) + (𝑛3 − 3𝑛2 + 8𝑛 − 6) 𝑈 𝑛 = 𝑛3 + 2𝑛2 − 3𝑛 + 1 Jadi formula barisan tersebut adalah 𝑈 𝑛 = 𝑛3 + 2𝑛2 − 3𝑛 + 1 4. Diberikan suatu bilangan desimal 0,5151515151....... ubah kedalam bentuk pecahan! Jawab: Untuk mengubah pecahan desimal diatas menjadi bentuk pecahan kita gunakan konsep deret geometri. Deret geometri dari bilangan desimal tersebut adalah sebagai berikut. 51 100 + 51 10000 + 51 10000000 + ⋯ Didapat 𝑎 = 51 100 dan 𝑟 = 1 100 𝑆∞ = 𝑎 1 − 𝑟 = 51 100 1 − 1 100 𝑆∞ = 51 99 Jadi bentuk pecahan dari bilangan desimal 0,515151 adalah 51 99 5. ∑ ( 𝑘−5 𝑘+2 )∞ 𝑘=1 Jawab: 𝑆 𝑛 = − 4 3 + (− 3 4 ) + ⋯ + 𝑛 − 5 𝑛 + 2 + ⋯ lim 𝑛→∞ 𝑆 𝑛 = lim 𝑛→∞ 𝑛−5 𝑛+2 = 1, karena nilai limitnya ada yaitu 1, maka deret tersebut konvergen dan nilai deretnya mendekati nilai limitnya yaitu 1. 6. ∑ ( 1 𝑘 − 1 𝑘+1 )∞ 𝑘=2 Jawab: 𝑆 𝑛 = 1 6 + 1 12 + ⋯ + 1 𝑛 − 1 𝑛+1 + ⋯
  • 28. 26 lim 𝑛→∞ 𝑆 𝑛 = lim 𝑛→∞ 1 𝑛 − 1 𝑛+1 = 0, karena nilai limitnya ada yaitu 0, maka deret tersebut konvergen dan nilai deretnya mendekati nilai limitnya yaitu 0. 7. ∑ 2 (𝑘+2)𝑘 ∞ 𝑘=1 Jawab: Cara 1 mengubah bentuk 2 (𝑘+2)𝑘 menjadi bentuk 𝐴 𝑘 + 𝐵 𝑘+2 2 (𝑘 + 2)𝑘 = 1 𝑘 − 1 𝑘 + 2 ∑ 2 (𝑘 + 2)𝑘 ∞ 𝑘=1 = ∑ 1 𝑘 − 1 𝑘 + 2 ∞ 𝑘=1 ∑ 1 𝑘 − 1 𝑘 + 2 ∞ 𝑘=1 = ( 1 1 − 1 3 ) + ( 1 2 − 1 4 ) + ( 1 3 − 1 5 ) + ⋯ + ( 1 𝑛 − 1 𝑛 + 2 ) ∑ 2 (𝑘 + 2)𝑘 ∞ 𝑘=1 = (1 + 1 2 ) − ( 1 𝑘 + 1 + 1 𝑘 + 2 ) Cara 2 𝑆 𝑛 = 2 3 + 1 4 + ⋯ + 2 (𝑛 + 2)𝑛 + ⋯ lim 𝑛→∞ 𝑆 𝑛 = lim 𝑛→∞ 2 (𝑛+2)𝑛 = 0, karena nilai limitnya ada yaitu 0, maka deret tersebut konvergen dan nilai deretnya mendekati nilai limitnya yaitu 0. Masalah Yang Dibahas Kelompok 1. Buktikan a = 𝑆 𝑛 𝑛 – 1 2 (n-1)b Solusi: Sn = n 2 (a + Un) 2 Sn = n (2a+(n-1)b) 2 Sn = 2an + bn (n-1) a = 2Sn-bn (n-1) 2n
  • 29. 27 a = Sn n – 1 2 (n-1)b (terbukti) INDUKSI MATEMATIS Induksi Matematika adalah cara standar dalam membuktikan bahwa sebuah pernyataan tertentu berlaku untuk setiap bilangan asli. Kita akan menotasikan himpunan bilangan asli dengan ℕ = {1, 2, 3, . . . }, dengan operasi tambah dan perkalian seperti biasa. Bilangan Asli ℕ ini memenuhi sifat terurut sempurna (Well-Ordering Property) yaitu “setiap himpunan bagian yang tidak kosong dari ℕ mempunyai bilangan terkecil”. Sifat ini menyatakan bahwa S adalah himpunan bagian dari ℕ dan S ≠ ∅, maka ada bilangan m  S sehingga m ≤ k untuk setiap k  S. Secara formal Induksi Matematika ini bisa didenisikan sebagai berikut. Definisi 1.1 Misalkan untuk setiap bilangan asli n kita mempunyai pernyataan P(n) yang bisa benar atau salah. Misalkan 1. P(1) benar. 2. Jika P(n) benar, maka P(n + 1) benar. Sehingga P(n) benar untuk setiap bilangan asli n. Langkah 1 disebut dengan Langkah Dasar, sedangkan Langkah 2 disebut dengan Langkah Induktif. Jika pada Langkah Induktif yang diasumsikan adalah pernyataan P(i) benar untuk setiap bilangan i ≤ n, maka perumusan induksi matematika seperti ini disebut Bentuk Kuat Induksi Matematika. Tahapan Induksi Matematika  Basis Step : Tunjukkan bahwa S(1) benar  Inductive Step : Sumsikan S(k) benar Akan dibuktikan S(k)  S(k+1) benar  Conclusion : S(n) adalah benar untuk setiap n bilangan integer positif
  • 30. 28 Prinsip Induksi Matematika Misalkan S suatu himpunan bagian dari N yang mempunyai sifat: 1) 1 S 2) jika k  S maka k+1  S Maka S = N Prinsip Induksi Matematika ini mengatakan bahwa suatu himpunan bagian S dari bilangan asli N di mana sifat (1) dan (2) dimiliki oleh himpunan itu, maka himpunan bagian itu akan merupakan himpunan bilangan asli N atau S = N. Materi Pengayaan 1. Buktikan dengan induksi matematika bahwa ∑ 𝑖 = 𝑛(𝑛+1) 2 𝑛 𝑖=1 Jawab: Pilih i= 1 maka 1 = 1(1+1) 2 = 1 (benar) Asumsikan untuk n = k benar maka Sk = 𝑘(𝑘+1) 2 maka akan ditunjukkan benar untuk n = k+1 𝑆 𝑘+1 = 𝑆 𝑘 + (𝑘 + 1) = 𝑘(𝑘+1) 2 + 𝑘 + 1 = 𝑘(𝑘+1) 2 + 2(𝑘+1) 2 = 𝑘(𝑘+1)+2(𝑘+1) 2 = (𝑘+1)(𝑘+2) 2 = (𝑘+1)((𝑘+1)+1) 2 (terbukti) 2. Buktikan dengan induksi matematika bahwa ∑ 𝑖3𝑛 1 = (𝑛(𝑛+1)) 4 2 Jawab: Misalkan 𝑛 = 1 maka 21 3 1 (1(1 1)) 1 4  
  • 31. 29 2 (1(2)) 4 4 4 1    Benar Asumsikan 𝑛 = 𝑘 benar 2 3 1 ( ( 1)) 4 k k k i   Akan dibuktikan 𝑛 = 𝑘 + 1 benar 2 31 3 1 ( ( 1)) 4( 1) 4 4 k k k k i      2 2 2 2 2 ( 1) ( 4 4) 4 ( 1) ( 2) 4 (( 1)(( 1) 1)) 4 k k k k k k k            (terbukti) BILANGAN BERPANGKAT BULAT Definisi: Bentuk an dengan a, n ∈ Z disebut bentuk pangkat. n disebut pangkat (eksponen) dan a disebut bilangan pokok (basis). Untuk n bilangan asli an berarti bilangan a dikalikan sebanyak n kali.    faktorn aaaa  ... Contoh: 25 = 2 × 2 × 2 × 2 × 2 dan 52 = 5 × 5. Untuk n bilangan bulat negatif maka 𝑎 𝑛 = 1 𝑎−𝑛 (n negatif maka – 𝑛 positif). Pernyataan ini setara dengan 𝑎−𝑛 = 1 𝑎 𝑛 , untuk bilangan asli n. Definisi ini sebenarnya mudah dipahami dari kenyataan bahwa 𝑎−1 adalah invers dari a pada operasi perkalian, yakni:
  • 32. 30 0, 1 1 11   a a aaa sehingga   n n nnn aa aaa 1111         . Contoh: 4−2 = 1 42 = 1 4×4 = 1 16 . Sifat-sifat Bilangan Berpangkat 1. Sifat Perkalian Pangkat Untuk mengalikan bilangan berpangkat dengan basis yang sama jumlahkan pangkatnya. Kasus 1: jika n dan m bilangan asli mn faktormn faktormfaktorn mn aaaaaaaaaaa            ...... Kasus 2: jika n bilangan asli dan m bilangan negatif atau m bilangan asli dan n bilangan negatif. Untuk kasus ini, kita hanya akan menimbulkan n untuk kasus bilangan asli dan m bilangan negatif, sedangkan yang lainnya dapat dilakukan dengan cara yang serupa, jika m bilangan negatif maka (-m) bilangan positif. mn faktormn faktorm faktornmn aaaaa aaaa aaaa aa                 )( ... ... ... Kasus 3: jika n dan m bilangan negatif mn mn faktormfaktorn mn a aaaaaaaaa aa       )( 1 ...... 1      Kasus 4: n bilangan bulat dan m = 0 atau m bilangan bulat dan n = 0. Untuk kasus ini, kita hanya akan membuktikan untuk kasus n bilangan bulat dan m = 0, sedangkan yang lainnya dapat dilakukan dengan cara yang serupa. 𝑎 𝑛 × 𝑎 𝑚 = 𝑎 𝑛 × 𝑎0 = 𝑎 𝑛 × 1 = 𝑎 𝑛 = 𝑎 𝑛+0 = 𝑎 𝑛+𝑚 Tentu saja untuk kasus a0 menyaratkan a ≠ 0. Dari keempat kasus ini, lengkaplah pembuktian 𝑎 𝑛 × 𝑎 𝑚 = 𝑎 𝑛+𝑚 .
  • 33. 31 Contoh: 53 × 52 = 53+2 = 55 2. Sifat pembagian pangkat Untuk membagi bilangan berpangkat dengan basis yang sama, kurangkan pangkatnya. mn m n a a a   Contoh: 67 64 = 67−4 = 63 . 3. Sifat perpangkatan dari pangkat Untuk memangkatkan bilangan berpangkat dengan basis yang sama, kalikan pangkatnya.   mnmn aa   Contoh: (33)4 = 33×4 = 312 . 4. Sifat perpangkatan dari perkalian Untuk mencari pangkat dari perkalian bilangan berpangkat, cari pangkat dari masing-masing faktor kemudian kalikan.   mmm baba  Contoh: (3 × 7)2 = 32 × 72 . 5. Sifat perpangkatan dari pembagian Untuk mencari pangkat dari pembagian bilangan berpangkat, cari hasil pemangkatan dari pembilang dan hasil pemangkatan dari penyebut kemudian bagilah. n nn b a b a       Contoh: ( 3 7 ) 2 = 32 72 = 9 49 . Perhatikan bahwa (𝑎 + 1)2 ≠ 𝑎2 + 12 , namun (𝑎 + 1)2 = (𝑎 + 1)(𝑎 + 1) = 𝑎2 + 2𝑎 + 12 dan perhatikan hal berikut: (−2𝑤)2 = (−2𝑤)(−2𝑤) = 4𝑤2 −(2𝑤)2 = −(2𝑤 × 2𝑤) = −4𝑤2
  • 34. 32 dua hal yang cukup berbeda. Untuk itu hati-hati dalam melakukan operasi bilangan berpangkat. Teorema: Untuk bilangan real a, b, dan n, m bilangan bulat berlaku: 1. mnmn aaa   2. mn m n a a a   3.   mnmn aa   4.   mmm baba  5. m mm b a b a       Teorema ini dapat diperluas dengan n dan m bilangan bulat. BILANGAN PANGKAT RASIONAL Definisi 1 Misalkan 𝑎 bilangan real dan 𝑎 ≠ 0, 𝑚, 𝑛 bilangan bulat didefinisikan 𝑎 𝑚 𝑛 = (𝑎 1 𝑛) 𝑚 Definisi 2 Misalkan a adalah bilangan real dan 𝑎 ≠ 0 dengan 𝑎 > 0, 𝑝 𝑞 adalah bilangan pecahan 𝑞 ≠ 0, 𝑞 ≥ 2, 𝑎 𝑝 𝑞 = 𝑐 sehingga 𝑐 = √ 𝑎 𝑝𝑞 atau 𝑎 𝑝 𝑞 = √ 𝑎 𝑝𝑞 Sifat 1 Misalkan 𝑎 adalah bilangan real dan 𝑎 ≠ 0 dengan 𝑎 > 0, 𝑝 𝑛 dan 𝑚 𝑛 adalah bilangan pecahan 𝑛 ≠ 0. Jika 𝑛, 𝑞 ≥ 2 maka (𝑎 𝑚 𝑛 ) (𝑎 𝑝 𝑛) = (𝑎) 𝑚+𝑝 𝑛 . Sifat 2 Misalkan 𝑎 adalah bilangan real dan 𝑎 ≠ 0 dengan 𝑎 > 0, 𝑚 𝑛 dan 𝑝 𝑞 adalah bilangan pecahan 𝑞, 𝑛 ≠ 0. maka (𝑎 𝑚 𝑛 ) (𝑎 𝑝 𝑞) = (𝑎) 𝑚 𝑛 + 𝑝 𝑞.
  • 35. 33 BENTUK AKAR Definisi: Untuk bilangan real a, b, dan n bilangan asli maka: abba nn  Untuk n = 2 bentuk 2 a cukup ditulis a . Bentuk a sering disebut akar ketika a merupakan bilangan irasional. Sifat-sifat bentuk akar Untuk a, b, c, dan d bilangan real, berlaku: 1. Penjumlahan dan pengurangan bentuk akar a. 𝑎√𝑏 + 𝑐√𝑏 = (𝑎 + 𝑐)√𝑏 b. 𝑎√𝑏 − 𝑐√𝑏 = (𝑎 − 𝑐)√𝑏 2. Perkalian dan pembagian bentuk akar a. √ 𝑎 × √𝑏 = √𝑎 × 𝑏 b. √ 𝑎 √𝑏 = √ 𝑎 𝑏 , 𝑏 ≠ 0 c. 𝑎√𝑏 × 𝑐√𝑑 = (𝑎 × 𝑐)√𝑏 × 𝑑 d. 𝑐√ 𝑎 𝑑√𝑏 = 𝑐 𝑑 √ 𝑎 𝑏 , 𝑏 ≠ 0, 𝑑 ≠ 0 Akan diperluas pembahasan tentang bilangan bentuk akar √ 𝑎 dengan memperkenalkan definisi berikut. Definisi: Misalkan a bilangan real dan n bilangan rasional yang berbentuk 𝑛 = 𝑝 𝑞 , dengan p, q, bilangan asli. Maka bentuk pangkat 𝑎 𝑛 dituliskan sebagai bentuk akar 𝑎 𝑛 = 𝑎 𝑝 𝑞 = √ 𝑎 𝑝𝑞 , dengan a tidak negatif saat p genap. Untuk q = 2, bentuk √ 𝑎 𝑝𝑞 cukup ditulis sebagai √ 𝑎. Untuk bilagan real a, b, dan n, m bilangan rasional berbentuk 𝑛 = 𝑝 𝑞 dan 𝑚 = 𝑠 𝑡 , dengan p, q, s, t bilangan asli berlaku: 1. √ 𝑎 𝑝𝑞 × √ 𝑎 𝑠𝑡 = √𝑎 𝑝𝑡+𝑞𝑠 𝑞𝑡
  • 36. 34 2. √𝑎 𝑝 𝑞 √𝑎 𝑠𝑡 = √𝑎 𝑝𝑡−𝑞𝑠 𝑞𝑡 3. √ 𝑎 𝑝𝑞 × √𝑏 𝑝 𝑞 = √(𝑎 × 𝑏) 𝑝𝑞 4. √𝑎 𝑝 𝑞 √𝑏 𝑝 𝑞 = √( 𝑎 𝑏 ) 𝑝𝑞 Dengan a dan b tidak negatif saat p atau s genap. Materi Pengayaan 1. Apa definisi 𝑎 𝑛 ? Ada 2 syarat untuk 𝑎 dan n, yaitu : 𝑎 ∈ ℝ {0} a anggota bilangan Real yang tidak nol 𝑛 ∈ ℕ n anggota bilangan asli Jika 𝑎 ∈ ℝ, 𝑎 ≠ 0 akan dibuktikan bahwa 𝑎0 Ambil 𝑚, 𝑛 dengan 𝑚 = 𝑛 Maka 𝑎 𝑚 𝑎 𝑛 = 𝑎 𝑚−𝑛 = 𝑎 𝑚−𝑚 , karena 𝑚 = 𝑛 = 𝑎0 Kemudian dari 𝑎 𝑚 𝑎 𝑛 = 𝑎 𝑚 𝑎 𝑚 = 1 Jika 𝑎 ∈ ℝ, 𝑎 ≠ 0, 𝑛 ∈ ℤ+ ∪ {0} Akan dibuktikan bahwa 𝑎 𝑛 berlaku untuk 𝑛 ∈ ℤ− 𝑎0 𝑎 𝑛 = 𝑎0−𝑛 = 𝑎−𝑛 Untuk 𝑛 ∈ ℤ+ ∪ {0} sudah dibuktikan Misal 𝑛 ∈ ℤ+ , maka −𝑛 ∈ ℤ+ Karena 𝑎0 𝑎 𝑛 = 𝑎0−𝑛 = 𝑎−𝑛 𝑎 𝑛 berlaku untuk 𝑛 ∈ ℤ
  • 37. 35 Asimtot adalah sebuah persamaan yang didekati oleh sebuah garis. lim 𝑥→𝑐 𝑓(𝑥) = ±∞ (Asimtot tegak) lim 𝑥→+∞ 𝑓(𝑥) = 𝐶 (Asimtot datar) lim 𝑥→+∞ 𝑓(𝑥) = 𝐶(Asimtot datar) Masalah yang Didiskusikan Kelompok Word Problems Dalam sebuah ruangan terdapat seekor amoeba yang bereproduksi dengan membelah diri menjadi dua setiap 30 menit. Jika diasumsikan tidak ada amoeba yang mati selama 2 jam, maka berapakah banyak amoeba setelah 2 jam? Jawab: 𝑈1 = 𝑎 = 1 𝑟 = 2 𝑆4 = 𝑎(𝑟 𝑛 − 1) 𝑟 − 1 = 1(24 − 1) 2 − 1 = 16 − 1 = 15 Jadi banyak amoeba setelah 2 jam yaitu 15 ekor SIFAT-SIFAT KETERBAGIAN Definisi 2.1 Suatu bilangan bulat x dikatakan habis dibagi oleh suatu bilangan bulat y ≠ 0, jika terdapat satu bilangan bulat p sedemikian sehingga x = py. Jika hal ini dipenuhi maka y dikatakan membagi x dan dinotasikan dengan y│x yang dapat diartikan sebagai y adalah faktor (pembagi) x, atau x adalah kelipatan y. Jika y tidak membagi x dinotasikan dengan 𝑦 ∤ 𝑥. Atau : 𝑏|𝑎 ↔ ∃ 𝑐 ∈ 𝐼 ∋ 𝑎 = 𝑏. 𝑐 Contoh: −3 12⁄ ↔ ∃ − 4 ∈ 𝐼 ∋ 12 = (−4). (−3) Definisi 3.1 d | n berarti terdapat suatu bilangan bulat k sedemikian sehingga n = dk, sedangkan d ∤ n berarti bahwa d | n adalah salah. Dicatat bahwa 𝑎|𝑏 ≠ 𝑎 𝑏 .
  • 38. 36 Suatu cara lain untuk menyatakan definisi dari d | n adalah seperti di bawah ini. Definisi 3.2 d | n jika dan hanya jika n= 𝑑𝑘 untuk suatu k. Teorema 3.3 (Sifat-sifat Keterbagian) Jika n, m, dan d adalah bilangan-bilangan bulat maka pernyataan-pernyataan berikut ini benar. 1) n | n (sifat refleksif) 2) d | n dan n | m → d | m (sifat transitif) 3) d | n dan d | m → d | an + bm untuk setiap bilangan bulat a dan b (sifat linier) 4) d | n → ad | an untuk a ≠ 0 (sifat perkalian) 5) ad | an dan a ≠ 0 → d | n (sifat penghapusan) 6) 1 | n (1 membagi sembarang bilangan) 7) n | 1 →n = ± 1 (hanya 1 dan −1 yang merupakan pembagi dari 1) 8) d | 0 (sembarang nilai membagi nol) 9) 0 | n → n = 0 (nol hanya membagi nol) 10) d, n adalah positif dan d | n → d ≤ n (sifat perbandingan) 11) d | n dan d | (n + m) → d | m Definisi 3.5 jika c = as + bt untuk suatu bilangan bulat s dan t, dikatakan bahwa c merupakan suatu kombinasi linier dari a dan b. Jadi pernyataan (3) dalam Teorema 3.3 mengatakan bahwa jika d membagi a dan b, maka d membagi semua kombinasi linear dari a dan b. Khususnya, d membagi a + b dan a – b. ALGORITMA EUCLID Teorema Euclidean Misalkan m dan n adalah dua buah bilangan bulat dengan syarat n > 0. Jika m dibagi dengan n maka terdapat dua buah bilangan bulat unik q (quotient) dan r (remainder), sedemikian sehingga m = nq + r dengan 0 ≤ r < n. Contoh:
  • 39. 37 a. 1987 dibagi dengan 97. Berdasarkan teorema Euclidean menjadi 1987 = 97. 20 + 47. Memberikan hasil bagi 20 dan sisa 47. b. −22 dibagi dengan 3 memberikan hasil bagi −8 dan sisa 2. Berdasarkan teorema Euclidean menjadi −22 = 3 (−8) + 2. Tetapi −22 = 3 (−7) −1 salah karena r = −1 tidak memenuhi syarat 0 ≤ r < n. Algoritma Euclid Jika r1, r2  Z, dan r1 > r2 dan dengan proses algoritma pembagian dibentuk Suatu barisan menurun bilangan bulat r1, r2, r3, ... , rk-1, rk, rk+1=0 Yaitu: r1 = q1r2 + r3 , 0 ≤ r3 < r2. r2 = q2r3 + r4 , 0 ≤ r4 < r2. r3 = q3r4 + r5 , 0 ≤ r5 < r2. r4 = q4r5 + r6 , 0 ≤ r6 < r2. ............................................. rk-2 = qk-2rk-1 + rk , 0 ≤ rk < r2. rk-1 = qk-1rk + rk+1 , rk+1 = 0 Maka (r1,r2) = rk. Bukti. (r1,r2) = (q1r2 + r3 , r2) ....................... (substitusi r1) = (r3,r2) ........................ (teorema) = (r3, q2r3 + r4 ) ........................ (substitusi r2) = (r3,r4) ....... ....... ....... = (rk,rk+1) = (rk,0) .......................... (rk+1 = 0) (r1,r2) = rk Contoh 1. Tentukan (105,60) dan nyatakan hasilnya sebagai bentuk kombinasi linear
  • 40. 38 ax + by = c, dimana c = (a,b). Dengan Algoritma Euclides diperoleh: 105 = (1) 60 + 45 60 = (1) 45 + 15 45 = (3) 15 + 0, sehingga diperoleh (105,60) = 15. Selanjutnya dengan jalan mundur diperoleh: 15 = 60 – 45 (1)] = 60 – [105 – 60(1)] = 60 – 105 + 60 (1) = (-1) 105 + (2) 60. Akhirnya diperoleh (105,60) = (-1)105 + (2) 60. FPB dan KPK - FPB atau faktor persekutuan terbesar dari 𝑎, 𝑏 adalah 𝑑 = 𝑓𝑝𝑏(𝑎, 𝑏) dengan 𝑑 merupakan bilangan terbesar yang memenuhi 𝑑|𝑎 dan 𝑑|𝑏. - KPK atau kelipatan persekutuan terkecil dari 𝑎, 𝑏 adalah 𝑘 = 𝑘𝑝𝑘(𝑎, 𝑏) dengan 𝑘 merupakan bilangan terkecil yang memenuhi sehingga 𝑎|𝑘 dan 𝑏|𝑘 Terdapat beberapa sifat yang berkaitan dengan fpb dan kpk di antaranya: 1. 𝑓𝑝𝑏(0,0) = 0 2. 𝑓𝑝𝑏 (𝑎, 1) = 1 3. 𝑓𝑝𝑏(𝑎, 𝑎) = |𝑎| 4. 𝑓𝑝𝑏(𝑎, 𝑎 + 1) = 1 5. Jika 𝑎 = 𝑚𝑝 dan 𝑏 = 𝑚𝑞 maka 𝑓𝑝𝑏(𝑎, 𝑏) = 𝑚. 𝑓𝑝𝑏(𝑝, 𝑞) 6. 𝑘𝑝𝑘(𝑎, 𝑏) = 𝑎.𝑏 𝑑 , dengan 𝑑 = 𝑓𝑝𝑏(𝑎, 𝑏) Contoh: Tentukan fpb dan kpk dari 36,108, 64 Jawab: Karena 36 = 22 32 , 108 = 22 33 , dan 64 = 26 , maka fpb adalah 4. Dan kpk adalah 64.27 = 1728.
  • 41. 39 PERSAMAAN DIOPHANTINE LINEAR Persamaan Diophantine yang paling sederhana adalah memuat dua variabel pada umumnya dinyatakan dengan ax + by = c, dengan a,b,c  Z. Teorema 1: jika FPB (a, b) = d maka ada bilangan bulat x dan y sehingga ax + by = d Penjelasan: teorema di atas sama dengan bentuk persamaan Diophantine ax + by = FPB (a, b). Bentuk Ax + By = C Teorema 3.2: diberikan persamaan linier Diophantin secara umum dalam dua variabel x, y ∈ Z: ax + by = c, dengan a, b , c adalah bilangan bulat misalkan d = FPB (a, b) maka ax + by = c memiliki sebuah solusi jika dan hanya jika c habis dibagi oleh d. Teorema 7.1 Ditentukan a,b,c  Z dan d = ( a,b) a. Jika d tidak membagi c maka persamaan ax + by = c tidak mempunyai penyelesaian. b. Jika d membagi c maka persamaan ax + by = c mempunyai penyelesaian bulat yang tak hingga banyaknya, yaitu pasangan (x, y) dengan: x = xo + (b/d) n dan y = yo – (a/d) n Dengan n  Z dan (xo ,yo) adalah suatu penyelesaian khusus. Masalah yang Diberikan Instruktur 1. Selesaikan persamaan-persamaan Diophantine 4x + 5y = 10. Jawab: (4,5) = 1|10 Sesuai dengan Dalil Algoritma Euclides, karena (4,5) = 1 maka ada 11, yx sehingga ada 154 11  yx Karena 5 = 1.4 + 1 atau 4 (-1) + 5 (1) = 1, maka x1= -1, y1 = -1 4 (-1) + 5 (1) = 1 10 [ 4 (-1) + 5 (1)] = 10 .1
  • 42. 40 4 (-10) + 5 (10) = 10 (ingat 4x + 5y = 10) Jadi: xo = -10 dan yo = 10 Penyelesaian persamaan adalah x = -10 + 5k dan y = 10 - 4k dengan k  Z. 2. 9x + 12y = 21 Jawab: (9, 12) = 3 |21, persamaan mempunyai penyelesaian. Sesuai dengan Dalil Algoritma Euclides, karena (9, 12) = 3 maka tentu ada x1,y1  Z sehingga 9 x1 + 12 y1 = 3 Karena 12 = 1.9 + 3 atau 9 (-1) + 12 (1) = 3, maka x1= -1, y1 = 1 9 (-1) + 12 (1) = 3 7 [ 9 (-1) + 12 (1)] = 7 .3 9 (-7) + 12 (7) = 21 (ingat 9x + 12y = 21) Jadi: xo = -7 dan yo = 7 Penyelesaian persamaan adalah x = xo + (b/d) t = -7 + (12/3) t = -7 + 4t , dengan t  Z y = yo – (a/d) t = 7 – (9/3) t = 7 – 3t, dengan t  Z. 3. Selesaikan 4x + 6y = 7 Jawab: (4, 6) = 2. Karena 2 tidak dapat membagi habis 7 maka persamaan tersebut tidak punya selesaian bilangan bulat. 4. Selesaikan 4x + 5y = 10 dengan cara reduksi! Jawab: 4 2 2 4 510 5104 y y y x yx      Misal 4 2 y t   , maka
  • 43. 41 ty yt 42 24   Substitusi y ke persamaan x tx tt y yx 5 )42(2 4 2 2     Sehingga diperoleh selesaian      ty tx 42 50 di mana t  Z. 5. Selesaikan 3x + 8y = 11 dengan cara reduksi! Jawab: 3 22 23 3 811 8113 y y y x yx      Misal 3 22 y t   , maka 2 1 2 32 223 t t t y yt      Misal 2 t b   maka bt 2 Substitusi b ke persamaan y b bbt y 31 2 62 2 )2(32 2 32        Substitusi persamaan y ke persamaan x b bbb x 81 3 243 3 24811 3 )31(811        Sehingga diperoleh selesaian      by bx 31 81 di mana b  Z.
  • 44. 42 SIFAT-SIFAT DASAR KONGRUENSI Definisi Misalkan n ∈ N. a, b ∈ Z dikatakan kongruen modulo n, dinotasikan 𝑎 ≡ 𝑏 (𝑚𝑜𝑑 𝑛), jika n membagi a - b. Berarti a - b = kn untuk k ∈ Z. atau a=b+kn Contoh: 20 ≡ 6 (mod 7) 3 ≡ 38 (mod 7)  Diberikan 𝑎 ∈ 𝑍, misalkan q dan r adalah hasil bagi dan sisa pembagian oleh n, jadi 𝑎 = 𝑞𝑛 + 𝑟, dengan 0 ≤ 𝑟 < 𝑛.  Berdasarkan definisi kongruensi 𝑎 ≡ 𝑟 (𝑚𝑜𝑑 𝑛). Karena ada n pilihan untuk r, maka setiap bilangan bulat kongruen modulo n dengan tepat satu dari 0, 1, 2, ... , n-1.  𝑎 ≡ 0 (𝑚𝑜𝑑 𝑛) ⇔ 𝑛|𝑎.  {0, 1, 2, … , 𝑛 − 1} disebut himpunan sisa positif modulo n.  koleksi bilangan bulat 𝑎1, 𝑎2, … , 𝑎 𝑛membentuk himpunan lengkap sisa modulo n jika setiap bilangan bulat kongruen modulo n dengan satu dan hanya satu 𝑎 𝑘.  𝑎1, 𝑎2, … , 𝑎 𝑛 kongruen modulo n dengan 0, 1, 2, . . . , 𝑛 − 1. Contoh: {-12, -4, 11, 13, 22, 82, 91} adalah himpunan lengkap sisa modulo 7, karena -12≡2, -4≡3, 11≡4, 13≡6, 22≡1, 82≡5, 91≡0 semua modulo 7. Teorema 4.1 𝑎, 𝑏 ∈ 𝑍, 𝑎 ≡ 𝑏(𝑚𝑜𝑑 𝑛) ⇔ 𝑎 dan 𝑏 mempunyai sisa tak negatif yang sama jika di bagi 𝑛. Bukti: (⇒)  𝑎 ≡ 𝑏(𝑚𝑜𝑑 𝑛) ⇒ 𝑎 = 𝑏 + 𝑘𝑛, untuk suatu k∈ 𝑍.  b dibagi n mempunyai sisa 𝑟 ⇒ 𝑏 = 𝑞𝑛 + 𝑟, dengan 0 ≤ 𝑟 < 𝑛.
  • 45. 43  ⇒ 𝑎 = 𝑏 + 𝑘𝑛 = (𝑞𝑛 + 𝑟) + 𝑘𝑛 = (𝑞 + 𝑘)𝑛 + 𝑟.  ⇒ 𝑎 dan b mempunyai sisa yang sama jika dibagi 𝑛 (⇐) Misalkan 𝑎 = 𝑞1 𝑛 + 𝑟 dan 𝑏 = 𝑞2 𝑛 + 𝑟, dengan 0 ≤ 𝑟 < 𝑛. ⇒ 𝑎 − 𝑏 = (𝑞1 𝑛 + 𝑟) −(𝑞2 𝑛 + 𝑟) = (𝑞1 − 𝑞2)𝑛 ⇒ 𝑛|(𝑎 − 𝑏) ⇒ 𝑎 ≡ 𝑏(𝑚𝑜𝑑 𝑛) Teorema 4.2 Diberikan 𝑛 ∈ 𝑁 dan a, b, c, d, ∈ 𝑍 sebarang. Maka 1. 𝑎 ≡ 𝑎(𝑚𝑜𝑑 𝑛) 2. 𝑎 ≡ 𝑏(𝑚𝑜𝑑 𝑛) ⇒ 𝑏 ≡ 𝑎(𝑚𝑜𝑑 𝑛) 3. 𝑎 ≡ 𝑏(𝑚𝑜𝑑 𝑛) dan 𝑏 ≡ 𝑐(𝑚𝑜𝑑 𝑛) ⇒ 𝑎 ≡ 𝑐(𝑚𝑜𝑑 𝑛) 4. 𝑎 ≡ 𝑏(𝑚𝑜𝑑 𝑛) dan 𝑐 ≡ 𝑑(𝑚𝑜𝑑 𝑛) ⇒ 𝑎 + 𝑐 ≡ 𝑏 + 𝑑(𝑚𝑜𝑑 𝑛) dan 𝑎𝑐 ≡ 𝑏𝑑(𝑚𝑜𝑑 𝑛) 5. 𝑎 ≡ 𝑏(𝑚𝑜𝑑 𝑛) ⇒ 𝑎 + 𝑐 ≡ 𝑏 + 𝑐(𝑚𝑜𝑑 𝑛) dan 𝑎𝑐 ≡ 𝑏𝑐(𝑚𝑜𝑑 𝑛) 6. 𝑎 ≡ 𝑏(𝑚𝑜𝑑 𝑛) ⇒ 𝑎 𝑘 ≡ 𝑏 𝑘 (𝑚𝑜𝑑 𝑛) untuk 𝑘 ∈ 𝑍 sebarang. Bukti: 1. ∀𝑎 ∈ 𝑍 berlaku 𝑎 − 𝑎 = 0. 𝑛 ⇒ 𝑎 ≡ 𝑎(𝑚𝑜𝑑 𝑛) 2. 𝑎 ≡ 𝑏(𝑚𝑜𝑑 𝑛) ⇒ 𝑎 − 𝑏 ≡ 𝑘𝑛 untuk suatu 𝑘 ∈ 𝑍 ⇒ 𝑏 − 𝑎 = −(𝑘𝑛) = (−𝑘)𝑛. −𝑘 ∈ 𝑍 ⇒ 𝑏 ≡ 𝑎(𝑚𝑜𝑑 𝑛) 3. 𝑎 ≡ 𝑏(𝑚𝑜𝑑 𝑛), 𝑏 ≡ 𝑐(𝑚𝑜𝑑 𝑛) ⇒ ∃ℎ, 𝑘 ∈ 𝑍 ∋ 𝑎 − 𝑏 = ℎ𝑛 dan 𝑏 − 𝑐 = 𝑘𝑛 ⇒ 𝑎 − 𝑐 = (𝑎 − 𝑏) + (𝑏 − 𝑐) = ℎ𝑛 + 𝑘𝑛 = (ℎ + 𝑘)𝑛 ⇒ 𝑎 ≡ 𝑐(𝑚𝑜𝑑 𝑛) 4. 𝑎 ≡ 𝑏(𝑚𝑜𝑑 𝑛) dan 𝑐 ≡ 𝑑(𝑚𝑜𝑑 𝑛) ⇒ ∃ℎ, 𝑘 ∈ 𝑍 ∋ 𝑎 − 𝑏 = ℎ𝑛 dan 𝑐 − 𝑑 = 𝑘𝑛 ⇒ (𝑎 + 𝑐) − (𝑏 + 𝑑) = ℎ𝑛 − 𝑘𝑛 = (ℎ − 𝑘)𝑛 ⇒ 𝑎 + 𝑐𝑏 + 𝑑(𝑚𝑜𝑑 𝑛) dan 𝑎𝑐 = (𝑏 + ℎ𝑛)(𝑑 + 𝑘𝑛) = 𝑏𝑑 + (ℎ𝑑 + 𝑏𝑘 + ℎ𝑘𝑛)𝑛, karena ℎ𝑑 + 𝑏𝑘 + ℎ𝑘𝑛 ∈ 𝑍 ⇒ 𝑎𝑐 ≡ 𝑏𝑑(𝑚𝑜𝑑 𝑛) 5. 𝑐 ≡ 𝑐(𝑚𝑜𝑑 𝑛) dan (4) berlaku (5) 6. Jelas (6) berlaku untuk 𝑘 = 1.
  • 46. 44 Asumsikan berlaku untuk 𝑘 = 1 ⇒ 𝑎 𝑛 ≡ 𝑏 𝑛(𝑚𝑜𝑑 𝑛) ⇒ 𝑎𝑎 𝑛 ≡ 𝑏𝑏 𝑛(𝑚𝑜𝑑 𝑛) ⇒ 𝑎 𝑛+1 ≡ 𝑏 𝑛+1(𝑚𝑜𝑑 𝑛). Jadi ∀𝑘 ∈ 𝑁 berlaku 𝑎 ≡ 𝑏(𝑚𝑜𝑑 𝑛) ⇒ 𝑎 𝑛 ≡ 𝑏 𝑛(𝑚𝑜𝑑 𝑛) Teorema 4.3 Jika 𝑐𝑎 ≡ 𝑐𝑏(𝑚𝑜𝑑 𝑛), maka 𝑎 ≡ 𝑏(𝑚𝑜𝑑 𝑛|𝑑), dengan 𝑑 = 𝑝𝑝𝑏(𝑐, 𝑛). Bukti:  𝑐(𝑎 − 𝑏) = 𝑐𝑎 − 𝑐𝑏 = 𝑘𝑛 , untuk suatu k ∈ Z  𝑝𝑝𝑏(𝑐, 𝑛) = 𝑛 ⇒ ∃𝑟, 𝑠 yang relatif prima ∋ 𝑐 = 𝑑𝑟, 𝑛 = 𝑑𝑠  ⇒ 𝑟(𝑎 − 𝑏) = 𝑘𝑠 ⇒ 𝑠|𝑟(𝑎 − 𝑏) ⇒ 𝑎 ≡ 𝑏(𝑚𝑜𝑑 𝑛|𝑑) Akibat Jika 𝑐𝑎 = 𝑐𝑏(𝑚𝑜𝑑 𝑛)dan ppb(𝑐, 𝑛) = 1, maka 𝑎 = 𝑏(𝑚𝑜𝑑 𝑛) Tuliskan semua masalah yang diberikan instruktur beserta solusinya! 1050 dibagi 7 sisanya berapa? Jawab: Cara 1 10 ≡ 3 (𝑚𝑜𝑑 7) 102 ≡ 3.10 (𝑚𝑜𝑑 7) 102 ≡ 2 (𝑚𝑜𝑑 7) 103 ≡ 6 (𝑚𝑜𝑑 7) 104 ≡ 4(𝑚𝑜𝑑 7) 105 ≡ 5 (𝑚𝑜𝑑 7) 106 ≡ 1 (𝑚𝑜𝑑 7) (106 )8 ≡ 18 (𝑚𝑜𝑑 7) 1050 ≡ 102 (𝑚𝑜𝑑 7) 𝟏𝟎 𝟓𝟎 ≡ 𝟐 (𝒎𝒐𝒅 𝟕) Cara 2 1050 ≡ 350 (𝑚𝑜𝑑 7) 350 ≡ (33 )16 . 33(𝑚𝑜𝑑 7) 350 ≡ (−1)16 . 9(𝑚𝑜𝑑 7) 350 ≡ 9(𝑚𝑜𝑑 7) 𝟏𝟎 𝟓𝟎 ≡ 𝟐(𝒎𝒐𝒅 𝟕)
  • 47. 45 Atau cara 3 101 ≡ 3 𝑚𝑜𝑑 7 102 ≡ 2 𝑚𝑜𝑑 7 103 ≡ 6 𝑚𝑜𝑑 7 104 ≡ 4 𝑚𝑜𝑑 7 105 ≡ 5 𝑚𝑜𝑑 7 106 ≡ 1 𝑚𝑜𝑑 7 107 ≡ 3 𝑚𝑜𝑑 7 Sisa dari hasil baginya berulang sebanyak 6 kali, maka 50 6 memiliki sisa 2, ada di kelas kedua, yaitu pembagian dengan sisa 2. Maka sisa dari 1050 7 adalah 2 Jadi sisa dari 1050 di bagi 7 adalah 2. Masalah yang didiskusikan dalam kelompok 1. Berapakah sisa 52011 dibagi 7? 2. Sisa pembagian 3247 + 11 akan 17 adalah ... 3. Berapakah nilai digit terakhir dari 2111 ? 4. Berapakah nilai dua digit terakhir dari 62018 ? Jawab 1. 52011 ≡ 𝑎 (𝑚𝑜𝑑 7) (57−1 )335+1 (𝑚𝑜𝑑 7) ≡ (1)335 . 51 (𝑚𝑜𝑑 7) ≡ 5 (𝑚𝑜𝑑 7) Jadi sisa pembagian tersebut adalah 5. 2. (3247 + 11)(𝑚𝑜𝑑 17) 32 ≡ 9 (𝑚𝑜𝑑 17) 33 ≡ 10 (𝑚𝑜𝑑 17) 34 ≡ 13 (𝑚𝑜𝑑 17) 35 ≡ 5 (𝑚𝑜𝑑 17) 36 ≡ 15 (𝑚𝑜𝑑 17) 37 ≡ 11 (𝑚𝑜𝑑 17) 38 ≡ −1 (𝑚𝑜𝑑 17) (38 )30 ≡ (−1)30 (𝑚𝑜𝑑 17) 3240 ≡ 1 (𝑚𝑜𝑑 17) 3247 ≡ 107 (𝑚𝑜𝑑 17) 3247 ≡ 11 (𝑚𝑜𝑑 17) ∴ (3247 + 11)(𝑚𝑜𝑑 17) ≡ (11 + 11)(𝑚𝑜𝑑 17) ≡ 𝟓 (𝑚𝑜𝑑 17) Jadi sisanya adalah 5
  • 48. 46 3. Digit terakhir dalam perpangkatan 2: 21 ≈ 2 22 ≈ 4 23 ≈ 8 24 ≈ 6 25 ≈ 2 Digitnya berulang 4 periode, maka digit terakhir dari perpangkatan tersebut adalah 2111 ≈ 24.27+3 ≈ 23 ≈ 𝟖 Jadi digit terakhirnya adalah 8. 4. Dua digit terakhir dari 62018 adalah Jawab: 6 ≡ 6(𝑚𝑜𝑑100) 62 ≡ 36(𝑚𝑜𝑑100) 63 ≡ 16(𝑚𝑜𝑑100) 64 ≡ 96(𝑚𝑜𝑑100) 65 ≡ 76(𝑚𝑜𝑑100) 66 ≡ 56(𝑚𝑜𝑑100) 67 ≡ 36(𝑚𝑜𝑑100) 68 ≡ 16(𝑚𝑜𝑑100) 69 ≡ 96(𝑚𝑜𝑑100) 610 ≡ 76(𝑚𝑜𝑑100) 611 ≡ 56(𝑚𝑜𝑑100) 612 ≡ 36(𝑚𝑜𝑑100) Maka dari diatas dapat disimpulkan 62018 ≡ (65 )403 63 (𝑚𝑜𝑑100) 62018 ≡ 63 (𝑚𝑜𝑑100) 62018 ≡ 16(𝑚𝑜𝑑100) 𝑗𝑎𝑑𝑖 𝑏𝑖𝑙𝑎𝑛𝑔𝑎𝑛 𝑑𝑢𝑎 𝑑𝑖𝑔𝑖𝑡 𝑡𝑒𝑟𝑎ℎ𝑖𝑟 𝑑𝑎𝑟𝑖 62018 𝑎𝑑𝑎𝑙𝑎ℎ 16 Definisi 4.1. Misalkan 𝑛 ∈ 𝑁. 𝑎, 𝑏 ∈ 𝑍 dikatakan kongruen modulo n, dinotasikan 𝑎 ≡ 𝑏(𝑚𝑜𝑑 𝑛), jika n membagi 𝑎 − 𝑏 berarti 𝑎 − 𝑏 = 𝑘𝑛 untuk 𝑘 ∈ 𝑍. Contoh:  3 ≡ 24(𝑚𝑜𝑑 7), −31 ≡ 11(𝑚𝑜𝑑 7), −15 ≡ −64 (𝑚𝑜𝑑 7) Karena 3 − 24 = (−3)7, −31 − 11 = (−6)7, −15 − (−64) = 7.7.  Setiap dua bilangan adalah kongruen modulo 1.
  • 49. 47  Jika dua bilangan keduanya ganjil atau keduanya genap maka kedua bilangan tersebut kongruen modulo 2. Jika n < (a – b) maka dikatakan a tidak kongruan dengan b modulo n , dinotasikan a # b. Contoh: 25 # 12 (mod 7) karena 7 < (25 – 12). Teorema 4.1. 𝑎, 𝑏 ∈ 𝑧, 𝑎 ≡ 𝑏(𝑚𝑜𝑑 𝑛) ↔ a dan b mempunyai sisa tak negatif sama jika dibagi dengan n. Contoh:  –56 dan –11 dapat dinyatakan dalam bentuk −56 = (−7)9 + 7, −11 = (−2)9 + 7 → −56 ≡ −11(𝑚𝑜𝑑 9)  −31 ≡ 11 (𝑚𝑜𝑑 7) → −31 dan 11 mempunyai sisa yang sama jika dibagi 7 yaitu −31 = (−5)7 + 4, 11 = 1.7 + 4. Teorema 4.2 Diberikan 𝑛 ∈ 𝑁 dan a,b,c,d ∈ ℤ, sebarang. Maka 1. a ≡ a (mod n). 2. a ≡ b (mod n)⇒ b ≡ a (mod n). 3. a ≡ b (mod n) dan b ≡ c (mod n) ⇒ a ≡ c (mod n). 4. a ≡ b (mod n) dan c ≡ d (mod n) ⇒ a+c ≡ b+d (mod n) dan ac ≡ bd (mod n). 5. a ≡ b (mod n) ⇒ a+c ≡ b+c (mod n) dan ac ≡ bc (mod n). 6. a ≡ b (mod n) ⇒ ak ≡ bk (mod n) untuk k∈ ℤ, sebarang. Bukti: 1. ∀ a ∈ ℤ berlaku a-a = 0.n ⇒ a ≡ a (mod n) 2. a ≡ b (mod n) ⇒ a-b = kn untuk suatu k ∈ ℤ ⇒ b-a = -(kn) = (-k)n. -k ∈ ℤ ⇒ b ≡ a (mod n) 3. a ≡ b (mod n), b ≡ c (mod n) ⇒ ∃ h,k ∈ ℤ ∋ a-b = hn dan b-c = kn ⇒ a-c = (a-b) + (b-c) = hn + kn = (h+k)n ⇒ a ≡ c (mod n). 4. a ≡ b (mod n) dan c ≡ d (mod n) ⇒ ∃ h,k ∈ ℤ ∋ a-b = hn dan c-d = kn ⇒
  • 50. 48 (a+c) – (b+d) = hn – kn = (h-k)n ⇒ a+c ≡ b+d (mod n) dan ac = (b+hn)(d+kn)= bd+(hd+bk+hkn)n, karena hd+bk+hkn ∈ ℤ ⇒ ac≡ bd (mod n) 5. c ≡ c (mod n) dan (4) berlaku (5) 6. Jelas (6) berlaku untuk k = 1 Asumsikan berlaku untuk k = n ⇒ an ≡ bn (mod n) ⇒ aan ≡ bbn (mod n) ⇒ an+1 ≡ bn+1 (mod n). Jadi ∀ k ∈ ℕ berlaku a ≡ b (mod n) ⇒ ak ≡ bk (mod n) Contoh: Buktikan 220 − 1 dapat dibagi 41. Bukti: 25 ≡ −9(mod 41) ⇒ (25 )4 ≡ (−9)5(mod 41) ⇒ 220 ≡ 81.81 (mod 41) 81 ≡ −1(mod 41) ⇒ 81.81 ≡ 1 (mod 41) ⇒ 1 ≡ 81.81(mod 41) ⇒ 220 − 1 ≡ (81.81 − 81.81)(mod 41) ⇒ 220 − 1 ≡ 0(mod 41) ⇒ 41| 220 − 1 Secara khusus, jika r adalah residu (sisa) pembagian a oleh m, maka a kongruen dengan r mod m dengan syarat mr 0 . Contoh: )4(mod2610  )4(mod210  )4(mod226  Untuk setiap bilangan bulat adalah kongruen modulo m jika bilangan tersebut berada diantara 0 dan (m–1). Bilangan m tersebut disebut modulus dari kekongruenan. Kekongruenan dengan modulus yang sama memiliki sifat yang hampir serupa dengan persamaan biasa.
  • 51. 49 Jika )(mod11 mba  dan )(mod22 mba  , maka: i) )(mod2121 mbbaa  ii) )(mod.. 2121 mbbaa  Sifat di atas ada kemungkinan tidak berlaku untuk pembagian dalam kekongruenan. Dengan kata lain, jika )(modmbcac  maka tidak selalu berlaku )(modmba  Teorema: Jika )(modmcbca  , maka )/(mod dmba  , dengan d = fpb(c, m). Bukti:  c(a – b) = ca – cb = km, untuk suatu k .  fpb(c, m) = m  r, s yang relatif prima  c = dr, m = ds.  r(a – b) = ks s|r(a – b)  )/(mod dmba  . Akibat: Jika )(modmcbca  dan fpb(c, m) = 1, maka )(modmba  . Contoh: 1. )10(mod2.202.15  , tetapi 15 ≢20 (mod 10). 2. Jika )(mod0. mvu  , maka dapat saja u ≢ 0 (mod m) dan v ≢ 0 (mod m). Contoh: )8(mod02.4  4 ≢ 0 (mod 8) dan 2 ≢ 0 (mod 8) Catatan:  Kekongruenan yang memuat variabel dapat diselesaikan dengan cara yang sama seperti menyelesaikan persamaan biasa.  Dua buah persamaan yang memiliki variabel yang sama disebut ekuivalen jika kedua persamaan tersebut memiliki penyelesaian yang sama.
  • 52. 50 Materi Pengayaan 1. Carilah penyelesaian dari )8(mod512 x Jawab: Berdasarkan sifat-sifat kongruensi linear, berlaku )8(mod512 x )8(mod7x )8(mod1x Jadi, penyelesaian dari )8(mod512 x yaitu )8(mod1x . Selesaian tersebut tidak tunggal, selesaian yang lain yaitu {…, -15, -7, 1, 9, 17, 25, …} 2. Carilah penyelesaian dari )19(mod34 x . Jawab: Cara 1: )19(mod34 x )19(mod224 x )19(mod11.22.2 x )19(mod112 x )19(mod302 x )19(mod15.22 x )19(mod15x Cara 2: 4x – 3 = 19.k (k   ) 4x = 19k + 3 4 319   k x Pilih k = 3, diperoleh 15 4 33.19   x , sehingga )19(mod15x Cara 3: )19(mod34 x )19(mod3.54.5 x
  • 53. 51 )19(mod1520 x )19(mod15x Jadi, penyelesaian dari )8(mod512 x adalah )19(mod15x . 3. Carilah penyelesaian dari )7(mod0122  xx . Jawab: Cara 1: Menurut definisi, )7(mod0122  xx dapat ditulis 07122  kxx , dimana k   . Ambil sebarang k sedemikian sehingga persamaan kuadrat tersebut memiliki akar-akar biangan bulat. Untuk k = 1 7122  xx 0822  xx 0)2)(4(  xx x = -4 atau x = 2 Untuk k = 2 14122  xx 01522  xx (x + 5) (x – 3) = 0 x = -5 atau x = 3 berdasarkan definisi nilai r yang memenuhi yaitu r = {0, 1, 2, …, 6}, sehingga nilai yang memenuhi adalah 2 dan 3. Penyelesaian dari )7(mod0122  xx adalah )19(mod2x dan )19(mod3x . Cara 2: )7(mod0122  xx )7(mod7122  xx )7(mod0822  xx )7(mod0)2)(4(  xx )7(mod3 )7(mod4 )7(mod04    x x x atau )7(mod2 )7(mod02   x x
  • 54. 52 Jadi, penyelesaian dari )7(mod0122  xx adalah )7(mod3x dan )7(mod2x . 4. Carilah penyelesaian dari )10(mod32 x . Jawab: Menurut definisi, )10(mod32 x dapat ditulis kx .1032  310 103 2 2   kx kx Untuk setiap nilai  k , tidak ada bilangan k yang memenuhi .3102  kx Jadi, )10(mod32 x tidak mempunyai selesaian. 5. Carilah penyelesaian dari )514(mod156 x . Jawab: Oleh karena (6, 514) = 2 dan 2 tidak membagi habis 15, maka menurut teorema )514(mod156 x tidak mempunyai selesaian. 6. Carilah penyelesaian dari )22(mod818 x . Jawab: Cara 1: Menurut definisi, )22(mod818 x dapat ditulis kx .22818  . kx .22818  18 822   k x Pilih k = 7 dan 16, sehingga diperoleh nilai x = 9 dan x = 20. Cara 2: )22(mod818 x )22(mod4.29.2 x )11(mod49 x )11(mod159 x )11(mod5.33.3 x  )11(mod53 x
  • 55. 53 )11(mod5.43.4 x )11(mod2012 x )11(mod20x  )11(mod53 x )11(mod63 x )11(mod2x )11(mod9x Jadi, penyelesaian dari )22(mod818 x adalah )11(mod20x dan )11(mod9x UJI PEMBAGIAN BILANGAN BULAT Algoritma Pembagian Teorema 3.22 (Algoritma Pembagian) Jika a dan b adalah bilangan bulat dan b > 0 maka terdapat secara tunggal bilangan bulat q dan r yang memenuhi dua kondisi: 𝑎 = 𝑏𝑞 + 𝑟 dan 0 ≤ 𝑟 < 𝑏 Dalam situasi ini, q dinamakan hasil bagi (quotient) dan r dinamakan sisa (remainder) ketika a dibagi oleh b. Dicatat bahwa terdapat dua bagian untuk hasil ini, yaitu EKSISTENSI dan KETUNGGALAN dari bilangan bulat q dan r yang memenuhi persamaan 𝑎 = 𝑏𝑞 + 𝑟 dan 0 ≤ 𝑟 < 𝑏. Contoh (AHSME 1976) Diambil r adalah sisa ketika 1059, 1417 dan 2312 dibagi oleh b > 1. Tentukan nilai dari b−r. Penyelesaian: Berdasarkan Algoritma Pembagian 1059 = q1b + r, 1417 = q2b + r, dan 2312 = q3b + r untuk bilangan-bilangan bulat q1, q2, q3. Dari sini, 358 = 1417 − 1059 = b (q2 − q1), 1253 = 2312 − 1059 = b (q3− q1), dan 895 = 2312 − 1417 = b (q3 − q2). Karena itu b | 358 atau b | 2 ∙ 179, b | 1253 atau b | 7 ∙ 179, dan b | 895 atau b | 5 ∙ 179. Karena b > 1, disimpulkan bahwa b = 179. Jadi (sebagai contoh) 1059 = 5 ∙ 179 + 164, yang berarti bahwa r = 164. Disimpulkan bahwa b − r = 179 − 164 = 15.
  • 56. 54 Suatu bilangan memiliki sifat tertentu jika dibagi oleh beberapa bilangan tertentu. Beberapa sifat tersebut adalah: 1. Suatu bilangan dapat dibagi 2 jika dan hanya jika digit terakhirnya adalah bilangan genap. 2. Suatu bilangan dapat dibagi 3 jika dan hanya jika jumlah digit-digitnya habis dibagi 3. 3. Suatu bilangan dapat dibagi 4 jika dan hanya jika dua digit terakhir dari bilangan tersebut habis dibagi 4. 4. Suatu bilangan dapat dibagi 5 jika dan hanya jika digit terakhir dari bilangan tersbut adalah 0 atau 5. 5. Suatu bilangan dapat dibagi 6 jika dan hanya jika bilangan tersebut habis dibagi 2 dan 3. 6. Suatu bilangan dapat dibagi 7 jika dan hanya jika digit terakhir dikalikan dua, kemudian dikurangkan dengan angka sebelumnya, hasilnya habis dibagi 7 (termasuk nol). 7. Suatu bilangan dapat dibagi 8 jika dan hanya jika tiga digit terakhirnya dapat dibagi 8. 8. Suatu bilangan dapat dibagi 9 jika dan hanya jika jumlah digit-digitnya habis dibagi 9. 9. Suatu bilangan dapat dibagi 10 jika dan hanya jika digit terakhirnya adalah nol. 10. Suatu bilangan habis dibagi 11 jika jumlah digit pada tempat ganjil dikurangi jumlah digit pada tempat genap habis dibagi 11. 11. Suatu bilangan habis dibagi 12 jika dan hanya jika bilangan tersebut habis dibagi 3 dan 4. 12. Suatu bilangan habis dibagi 13 jika dan hanya jika menghilangkan digit terakhir, angka yang tersisa didepannya dikurangi dengan 9 kali digit yang dihilangkan, hasilnya akan habis dibagi 13. 13. Suatu bilangan habis dibagi 2 𝑛 jika n digit terakhir bilangan tersebut habis dibagi 2 𝑛 .
  • 57. 55 Contoh: - 54627819280 habis dibagi 5 karena digit terakhirnya 0 - 2568312 habis dibagi 3 karena 2+5+6+8+3+1+2 = 27 habis dibagi 3 - Lebih jauh, bilangan 2568312 habis dibagi 9 karena jumlah digitnya habis dibagi 9. - 123453 habis dibagi 11 karena (1+3+5) - (2+4+3) = 0 habis dibagi 11. - 2378264 habis dibagi 4 = 22 karena 64 habis dibagi 4. Contoh 2. Di antara empat bilangan: 5256, 7018, 18623, 32571, yang habis dibagi 99 adalah…. Jawab: Perhatikan bahwa 99=11.9 sehingga perlu diketahu diantara bilangan tersebut yang dapat dibagi 11 juga 9. - 5256 tidak habis dibagi 11 karena (5 + 5) − (2 + 6) = −2 tidak habis dibagi 11. - 7018 habis dibagi 11 karena (7 + 1) − (0 + 8) = 0 habis dibagi 11. - 18623 habis dibagi 11 karena (1 + 6 + 3) − (8 − 2) = 0 habis dibagi 11 - 32571 habis dibagi 11. Di antara 7018, 18623,32571 hanya 32571 yang juga habis dibagi 9, sehingga bilangan yang dapat dibai 99 adalah 32571 Contoh 3. N adalah bilangan bulat terbesar dengan semua digitnya berbeda dan N merupakan bilangan kelipatan 8. Tentukan tiga angka terakhir dari N? Jawab: Perhatikan bahwa 8 = 23 sehingga 3 digit terakhir dari N harus dapat dibagi 8. Bilangan N terbesar dengan 3 digit terakhir yang dapat dibagi 8 dengan digit berbeda adalah 120 (mengapa?).
  • 58. 56 KONGRUENSI LINEAR Definisi 7.2 Suatu kongruensi yang mempunyai bentuk: ax ≡ b (mod m) dengan a, b, m Z disebut suatu kongruensi linier satu variabel. Perhatikan bahwa jika x = xo adalah suatu selesaian ax ≡ b (mod m), dan jika diketahui bahwa x1 ≡ xo (mod m), maka ax1 ≡ axo (mod m), dengan demikian x1 juga suatu selesaian. Definisi 1: perkongruenan linear 𝑎𝑥 ≡ 𝑏(𝑚𝑜𝑑 𝑚) akan memiliki penyelesaian/solusi jika dan hanya jika ada bilangan bulat x dan k yang memenuhi persamaan 𝑎𝑥 ≡ 𝑏 + 𝑘𝑚. Contoh: 3𝑥 ≡ 4(𝑚𝑜𝑑 5). Jika x kita diganti dengan 3 akan memberikan 3.3 ≡ 4(𝑚𝑜𝑑 5) merupakan pernyataan yang benar. Teorema 1: jika gcd(𝑎, 𝑚) ∤ 𝑏 maka perkongruenan linear 𝑎𝑥 ≡ 𝑏(𝑚𝑜𝑑 𝑚) tidak memiliki solusi. Contoh: 6𝑥 ≡ 7(𝑚𝑜𝑑 8) karena gcd(6,8) = 2 dan 2 ∤ 7 maka perkongruenan ini tidak memiliki solusi. Teorema 2: jika gcd(𝑎, 𝑚) = 1 maka perkongruenan linear 𝑎𝑥 ≡ 𝑏(𝑚𝑜𝑑 𝑚) mempunyai tepat satu solusi. Contoh: 4𝑥 ≡ 1(𝑚𝑜𝑑 15), karena gcd(4, 15) adalah 1 maka tepat memiliki 1 solusi, maka tepat memiliki satu solusi, maka memungkinkan dilakukan konselasi (penghapusan) pada 4 sehingga diperoleh 𝑥 ≡ 1(𝑚𝑜𝑑 15) Contoh 7.4 Selesaikan kongruensi-kongruensi linier: (a) 36x ≡ 8 (mod 102) (b) 3x ≡ 2 (mod 5) (c) 15x ≡ 6 (mod 18) Jawab : (a) (36,102) = 6 dan 6 tidak membagi 8, maka 36x ≡ 8 (mod 102) tidak mempunyai selesaian.
  • 59. 57 (b) (3,5) = 1 dan 1│2, maka 3x ≡ 2 (mod 5) mempunyai satu selesaian x ≡ 4 (mod 5) (c) (15,18) = 3 dan 3│6, maka 15x ≡ 6 (mod 18) mempunyai tiga selesaian, yaitu x ≡ 4,10,16 (mod 18) Contoh: Tentukan bilangan prima yang kongruen dengan 1 modulo 4. Jawab: Bilangan prima yang kongruen dengan 1 modulo 4 yaitu {5, 13, 17, 29, ...}. 5 = 22 + 12 13 = 32 + 22 17 = 42 + 12 29 = 52 + 22 Jadi, bilangan prima yang kongruen dengan 1 modulo 4 dapat dinyatakan dalam penjumlahan 2 bilangan kuadrat. Teorema: tidak ada bilangan prima p berbentuk 4k + 3 yang merupakan jumlah dua bilangan kuadrat. Bukti : 𝑎2 + 𝑏2 𝑚𝑜𝑑 4 = 2 𝑚𝑜𝑑 4 = 1 𝑚𝑜𝑑 4 = 0 𝑚𝑜𝑑 4 Jadi karena 𝑎2 + 𝑏2 ≠ 3 𝑚𝑜𝑑 4, maka 𝑝 ≡ 3 𝑚𝑜𝑑 4 bukan merupakan jumlah dua bilangan kuadrat. Theorema Fermat: Suatu bilangan bulat positif n dapat dinyatakan sebagai jumlah dua bilangan kuadrat jika dan hanya jika masing-masing faktor primanya membentuk 4k + 3 terjadi pada pangkat genap Contoh:  459 = 33 × 17  153 = 32 × 17 = 32(42 + 12) = 122 + 32 Misal : 𝑛 = 5 × 72 × 13 × 17 = 72(22 + 12)(32 + 22)(42 + 12 )
  • 60. 58 = 72(22 + 12)((12 + 2)2 + (3 − 8)2) = 72(22 + 12)(142 + 52) = 72((28 + 5)5 + (10 − 14)2) = 72(332 + 42) = 2312 + 282 Materi Pengayaan Nyatakan 113, 229, dan 373 ke dalam penjumlahan dua bilangan kuadrat. Jawab:  113 = 82 + 72  229 = 152 + 22  373 = 182 + 72
  • 61. 59 LATIHAN SOAL 1. Gunakan induksi matematika untuk membuktikan persamaan berikut ini benar untuk setiap bilangan asli n. a. 𝑆 𝑛 = 1 + 3 + 5 + 7 + ⋯ + (2𝑛 − 1) = 𝑛2 b. 5 𝑛 − 1 dapat dibagi 4 c. 1.2 + 2.3 + 3.4 + ⋯ + 𝑛(𝑛 + 1) = 𝑛(𝑛+1)(𝑛+2) 3 2. Diantara bilangan berikut: 911253243, 55284471, 73683654 yang habis dibagi 99 adalah…. 3. Pada kompetisi sepak bola diikuti 5 klub (a, b, c, d, e). Masing-masing membawa bendera untuk dikibarkan. Lima bendera diatur dalam 5 tiang berjajar. Cara menempatkan lima bendera itu agar bendera klub a di tengah- tengah adalah…. 4. Selesaikan persamaan Diophantine 7𝑥 + 11𝑦 = 30! 5. 3𝑥 ≡ 5(𝑚𝑜𝑑11) dan 2𝑥 ≡ 7(𝑚𝑜𝑑11), nilai 𝑥2 + 𝑦2 adalah…. 6. Jika hari ini adalah hari Jumat, maka hari ke 102018 lagi adalah….
  • 62. 60 DAFTAR PUSTAKA Fahrurrozi. 2007. Tip & Trik: Menyiasati Matematika. Yogyakarta: Teknomedia. Kemendikbud. 2013. Buku Guru. Jakarta: Politeknik Negeri Media Kreatif. Nuryadi. 2014. Bahan Ajar Teori Bilangan. Yogyakarta: Universitas Mercu Buana. Rinaldi. “Teori Kongruensi”. http://dinus.ac.id/repository/docs/ajar/6.TeoriBilangan_.ppt (diakses 18 Februari 2018) Marwati, Rini. “Teori Bilangan”. http://file.upi.edu/Direktori/FPMIPA/JUR._PEND._MATEMATIKA/1966062519 90012-RINI_MARWATI/25_HandOutTeoriBilangan3.pdf (diakses 20 Februari 2018) Tasnan, Don. “Sifat-sifat Operasi Himpunan”. http://repository.binus.ac.id/content/K0064/K006426156.doc (diakses 21 Februari 2018) https://julanhernadi.files.wordpress.com/2011/05/teori_bilangan_bab3_1.pdf (diakses 20 Februari 2018) Irawan, Edy Bambang. “Bilangan 1”. http://repository.ut.ac.id/4698/2/PEMA4130-M1.pdf (diakses 24 Februari 2018) Muhsetyo, Gatot. “Modul 4 Kongruensi Linear”. https://www.slideshare.net/ChikaGidyfa/modul-4-kongruensi- linier?from_action=save (diakses 25 Februari 2018) http://jadijuara.com/contoh-soal-induksi-matematika/ (diakses 25 Februari 2018)